Решение задач по физике

Физика - это наука, изучающая взаимодействие материи в окружающем нас материальном мире и энергии. Классическая, или ньютоновская, физика развивалась до появления квантового принципа. В современной физике материя и энергия рассматриваются как дискретные единицы измерения - кванты.

Физика одна из основных естественных наук (наук о природе), основа современного естествознания. Она изучает наиболее общие свойства материи и формы ее движения (механическое, тепловое, электромагнитное, атомное, ядерное). Делится на множество разделов (механика, молекулярная физика, электромагнетизм, оптика, атомная физика, ядерная физика, физика элементарных частиц) и видов. Фундаментальными теориями Ф. являются: Ньютоновская механика, теория электромагнитного поля, специальная и общая теории относительности (см. Относительности теория), квантовая механика. Физика - наиболее развитая в методологическом отношении наука. Ее законы, методы, методологические принципы, теории и концепции широко используются в других науках.

Примеры решений задач по физике выпускных экзаменов, вступительных экзаменов, олимпиад, контрольных и т.д., для школьников и студентов.

Страница может использоваться для  самостоятельной работы школьников и студентов по решению физических задач (выполнению контрольных работ и индивидуальных домашних заданий).

Решение задач по физике – важнейший вид учебной деятельности в процессе обучения точным наукам.

Задачи по физике с решениями по разделам:

Содержание:

  1. Кинематика
  2. Динамика
  3. Динамика поступательного движения
  4. Статика
  5. Работа и энергия
  6. Работа и мощность. Закон сохранения импульса. Закон сохранения энергии
  7. Динамика вращательного движения
  8. Закон всемирного тяготения
  9. Классическая теория тяготения Ньютона
  10. Колебательные движения
  11. Колебания и волны
  12. Звук и звуковые волны
  13. Механика жидкостей и газов
  14. Теплотехника и термодинамика
  15. Теплота и молекулярная физика. Тепловое расширение твердых и жидких тел
  16. Свойства газов и паров
  17. Свойства жидкостей
  18. Теплота и капиллярные явления
  19. Электричество и электростатика
  20. Постоянный электрический тoк
  21. Переменный ток, электромагнитные колебания и волны
  22. Электричество и магнетизм
  23. Оптика. Отражение и преломление света
  24. Линзы. Оптические приборы
  25. Фотометрия
  26. Волновые свойства и действие света
  27. Оптика
  28. Атомная физика и квантовая механика
  29. Строение атома
  1. Основные физические величины
  2. Некоторые астрономические величины
  3. Математические постоянные
  4. Плотности
  5. Газы (г/м3 при нормальных условиях)
  6. Коэффициент трения скольжения k
  7. Удельная теплоемкость (дж/кг-град)
  8. Теплота сгорания топлива (106 дж/кг)
  9. Удельная теплота плавления (кдж/кг)
  10. Температура плавления и отвердевания (oC)
  11. Температура кипения (при 760 мм pm. ст.)
  12. Удельная теплота парообразования (кдж/кг)
  13. Теплопроводность некоторых твердых тел (вт/м-град)
  14. Коэффициент линейного расширения (град -I)
  15. Коэффициент объемного расширения жидкостей β (град -I)
  16. Коэффициент поверхностного натяжения при 20o C (н/м)
  17. Модуль упругости E (н/m2)
  18. Упругость насыщающих водяных паров (мм pm. ст.) и количество их в 1 м3 (кг)
  19. Психрометрическая таблица относительной влажности воздуха (%)
  20. Диэлектрическая проницаемость диэлектриков
  21. Удельное сопротивление (ом ∙ мм2/м или 10-6 ом ∙ м)
  22. Температурный коэффициент сопротивлений (град -I)
  23. Электрохимический эквивалент (кг/к)
  24. Показатель преломления
  25. Массы некоторых изотопов и элементарных частиц
  26. Таблица значений синусов и тангенсов углов
  1. Дополнительные рассмотренные темы задач с решением
  2. Механика. Кинематика материальной точки
  3. Динамика материальной точки
  4. Динамика твердого тела. Системе отсчета
  5. Законы сохранения импульса, момента импульса и энергии
  6. Механические колебания
  7. Упругие волны
  8. Молекулярно-кинетическая теория газов. Явления переноса
  9. Физические основы термодинамики
  10. Электростатика. Закон Кулона. Напряженность и потенциал электрического поля
  11. Проводники и диэлектрики в электрическом поле. Энергия электрического поля

Кинематика

Задача № 1. Автомобиль первую половину пути двигался со скоростью  Решение задач по физике а вторую — со скоростью Решение задач по физике Найти среднюю скорость движения автомобиля. 
Дано: 
Решение задач по физике

Решение:

Если общий путь, пройденный автомобилем, обозначим через Решение задач по физикето

Решение задач по физике
где Решение задач по физике — время, в течение которого автомобиль прошел первую   половину пути;Решение задач по физике — время, в течение которого автомобиль прошел вторую половину пути. 
Среднюю скорость движения автомобиля найдем из уравнения 
Решение задач по физике
или после подстановки значений Решение задач по физике и Решение задач по физике 
Решение задач по физике

Задача № 2. Велосипедист едет по пересеченной местности Решение задач по физике  в гору он двигался со скоростью Решение задач по физике остальной путь Решение задач по физике с горы — со скоростью Решение задач по физике Какова его средняя скорость? 
Дано: 
Решение задач по физике

Решение:
Чтобы получить среднюю скорость, нужно полный путь   разделить на время движения, т. е. 
Решение задач по физике
где Решение задач по физике — полный путь, пройденный велосипедистом; Решение задач по физике — время,  затраченное велосипедистом на преодоление подъема со скоростью  Решение задач по физике — время, затраченное на спуск велосипедиста со скоростью Решение задач по физике 
Среднюю скорость велосипедиста можно найти, вычислив Решение задач по физике и Решение задач по физике из соотношения Решение задач по физике
Решение задач по физике
Подставив в формулу для средней скорости значения Решение задач по физике и Решение задач по физике   получим 
Решение задач по физике

Задача № 3. Будет ли одинаково время проезда одного и того же расстояния на катере туда и обратно по реке и по озеру? Скорость течения реки Решение задач по физике скорость катера относительно воды в обоих 
случаях составляет Решение задач по физике

Дано: 
Решение задач по физике

Решение:
Время движения по реке против течения 
Решение задач по физике
Время движения по течению 
Решение задач по физике
Время движения по реке туда и обратно 
Решение задач по физике
Время движения катера туда и обратно по озеру 
Решение задач по физике
Отношение времен 
Решение задач по физике
т. е. время проезда по реке в 1,1 раза больше, чем время проезда по озеру. 

Задача № 4. Каким курсом должен лететь самолет, чтобы трасса его полета проходила строго с юга на север, если собственная скорость самолета Решение задач по физике а скорость ветра, дующего с востока, составляет Решение задач по физике 
Дано: 
Решение задач по физике

Решение:
Самолет участвует в двух движениях: со скоростью Решение задач по физике относительно воздуха и со скоростью Решение задач по физике  вместе с воздухом относительно Земли (рис. 1).

Решение задач по физике

Рис.1.

Поэтому истинная скорость самолета относительно Земли 
Решение задач по физике

Из прямоугольного треугольника Решение задач по физике находим, что 

Решение задач по физике
 

Задача № 5. С какой скоростью и по какому курсу должен лететь самолет, чтобы за время, равное Решение задач по физике пролететь точно на север путь Решение задач по физике если во время полета дует северо-западный ветер под  углом Решение задач по физике к меридиану со скоростью Решение задач по физике
Дано: 
Решение задач по физике

Решение:
Самолет участвует в двух движениях: со скоростью Решение задач по физике относительно воздуха и со скоростью Решение задач по физикев вместе с воздухом относительно Земли. По условию задачи скорость Решение задач по физике направлена на юго-восток под углом Решение задач по физике к меридиану. 
Скорость самолета относительно Земли Решение задач по физике По условию задачи скорость Решение задач по физикенаправлена по меридиану на север. 

По теореме сложения скоростей 
Решение задач по физике
Изобразим это равенство графически (рис. 2). Из чертежа видно, что самолет должен держать курс на северо-запад под углом Решение задач по физикек меридиану. 

Решение задач по физике

Рис.2.


Пользуясь теоремой косинусов, из Решение задач по физике по двум сторонам Решение задач по физике и Решение задач по физике и углу Решение задач по физике определим сторону Решение задач по физике т, е. численное значение скорости Решение задач по физике
Решение задач по физике

После подстановки численных значении получим 
Решение задач по физике
Вычислим Решение задач по физике (курс самолета). 
По теореме синусов 
Решение задач по физике
откуда 
Решение задач по физике

Прямолинейное равномерное движение

Задача № 6. Два катера, идущие с одинаковыми скоростями, одновременно отошли от причалов Решение задач по физике и Решение задач по физике находящихся на противоположных берегах реки, и идут все время по прямой Решение задач по физике соединяющей причалы. Расстояние между причалами Решение задач по физике Прямая Решение задач по физике образует с направлением течения воды в реке угол Решение задач по физике Скорость течения воды по всей ширине реки одинакова и равна Решение задач по физике Определить: место встречи катеров; угол, под каким они должны двигаться по отношению к прямой Решение задач по физике скорость движения катеров относительно воды, если известно, что катера встретились через Решение задач по физике после их отхода от причалов. 
Дано: 
Решение задач по физике

Решение:
Оба катера одновременно участвуют в двух движениях: в движении относительно воды, если бы она была в покое, и в движении вместе с водой. Следовательно, движение катеров относительно берегов реки, т. е, по прямой Решение задач по физике является результирующим. Очевидно, катера будут двигаться по прямой Решение задач по физике только в том случае, если диагонали параллелограммов, построенных на слагаемых скоростях Решение задач по физике и Решение задач по физике движения катеров для любого момента времени, будут совпадать с прямой Решение задач по физике (рис. 3).  

Для определения места встречи катеров необходимо найти пути Решение задач по физике и Решение задач по физике проходимые катерами от причалов по прямой Решение задач по физике до встречи. Так как время движения катеров до встречи известно, то для определения путей Решение задач по физике и Решение задач по физике надо знать результирующие скорости Решение задач по физике и Решение задач по физике движения катеров по прямой Решение задач по физике Эти скорости равны диагоналям параллелограммов и, как видно из рис, 3, будут; 
Решение задач по физике

Решение задач по физике

Рис.3.


Умножая предыдущие два уравнения на Решение задач по физике получим: 
Решение задач по физике
По условию задачи 
Решение задач по физике
Отсюда: 
Решение задач по физике
Подставляя численные значения физических величин и их размерности, получим: 
Решение задач по физике
Определим Решение задач по физике и Решение задач по физике (см. рис. 3): 
Решение задач по физике
Решая систему двух последних уравнений, найдем- 
Решение задач по физике
Подставив численные значения, получим 
Решение задач по физике

Пользуясь выражением Решение задач по физике можно определять скорость движения катеров относительно воды Решение задач по физике 
Решение задач по физике
Подставив численные значения, найдем 
Решение задач по физике

Задача № 7. Автомобиль движется в течение некоторого времени с постоянной скоростью Решение задач по физике Затем его движение становится равноускоренным, и он за Решение задач по физике проходит путь Решение задач по физике Как велики ускорение и конечная скорость автомобиля? 
Дано: 
Решение задач по физике 

Решение:
Из формул равноускоренного движения 
Решение задач по физике
находим: 
Решение задач по физике

Задача № 8. Тело, двигаясь равноускоренно, проходит два одинаковых отрезка пути в Решение задач по физике соответственно в течение Решение задач по физике и Решение задач по физике Определить ускорение и скорость тела в начале первого отрезка пути. 
Дано: 
Решение задач по физике

Решение:
Обозначив скорость тела в начале первого отрезка пути через Решение задач по физике, получим 
Решение задач по физике
В конце этого пути тело имело скорость 
Решение задач по физике
Второй участок пути определяется следующим уравнением: 
Решение задач по физике
Из этих двух уравнений получим: 
Решение задач по физике

Прямолинейное равнопеременное движение 

Задача № 9. Поезд двигался со скоростью Решение задач по физике При торможении до полной остановки он прошел Решение задач по физике Определить ускорение и время торможения. 
Дано: 
Решение задач по физике

Решение:
Так как конечная скорость поезда равна нулю, то ускорение и время торможения находим по формулам: 
Решение задач по физике

Задача № 10. С какой высоты упало тело, если за последнюю секунду своего падения оно прошло путь Решение задач по физике 

Дано: 
Решение задач по физике

Решение:
За время падения Решение задач по физике тело прошло путь 

Решение задач по физике

Путь, проходимый телом за время Решение задач по физике

Решение задач по физике
Пройденный путь в последнюю секунду равен 
Решение задач по физике
откуда 

Решение задач по физике
Тогда 
Решение задач по физике

Задача № 11. С какой начальной скоростью с высоты Решение задач по физике нужно бросить вертикально вниз тело, чтобы оно упало на Решение задач по физике раньше, чем при свободном падении? 
Дано: 
Решение задач по физике 

Решение:
Пусть время свободного падения тела Решение задач по физике Тогда высота падения 
Решение задач по физике
С другой стороны, при падении тела с начальной скоростью

Решение задач по физике

Приравнивая правые части полученных уравнении и учитывая, что Решение задач по физике найдем 

Решение задач по физике
или 
Решение задач по физике

Свободное падение тел. Движение тела, брошенного вертикально

Задача № 12. Тело падает с высоты Решение задач по физике Спустя время Решение задач по физике с меньшей  высоты начинает падать второе тело. Какова должна быть высота, с которой падает второе тело, чтобы они одновременно упали на 
землю? 
 

Решение:
Пусть второе тело находилось в полете время Решение задач по физике тогда время, в течение которого падало первое тело, равно Решение задач по физике Пути, пройденные первым и вторым телами, соответственно равны 
Решение задач по физике 
Выразим из второго уравнения Решение задач по физике и подставим в первое: 
Решение задач по физике
После решения относительно Решение задач по физике найдем 

Решение задач по физике

Движение тела, брошенного горизонтально

Задача № 13. Баллон поднимается вверх с постоянной скоростью Решение задач по физике В определенный момент времени с баллона сброшен камень,  падающий на землю в течение Решение задач по физике Как высоко находился баллон (рис. 4), когда камень упал на землю? 

Решение задач по физике

Рис.4.

Дано:

Решение задач по физике

Решение:
Так как камень двигался равнопеременно и начал свое движение с начальной скоростью Решение задач по физике то путь, пройденный за время Решение задач по физике будет 

Решение задач по физике
Через тот же промежуток времени Решение задач по физике баллон окажется на высоте Решение задач по физике равной 
Решение задач по физике
Подставляя значение Решение задач по физике получим 
Решение задач по физике 

Задача № 14. Два парашютиста Решение задач по физике одновременно прыгнули с двух самолетов, находящихся на разных высотах, причем парашютист Решение задач по физике находился на высоте Решение задач по физике — на  
высоте Решение задач по физике над землей Решение задач по физике Падение  парашютистов после раскрытия парашютов 
примем за равномерное со скоростью Решение задач по физике Определить высоту Решение задач по физике на которой парашютист Решение задач по физике должен раскрыть свой парашют, чтобы приземлиться одновременно с парашютистом Решение задач по физике Сопротивлением  воздуха до раскрытия парашютов  пренебречь. 

Решение:
Рассмотрим два случая. Первый случай, когда парашютист Решение задач по физике сразу  раскрыл свой парашют. Пусть парашютист Решение задач по физике раскрыл свой парашют в точке Решение задач по физике (рис. 5) на высоте Решение задач по физике когда парашютист Решение задач по физике был в точке Решение задач по физике тоже на высоте Решение задач по физике в этом случае они приземлятся 
одновременно. Время падения парашютиста Решение задач по физике от точки Решение задач по физике до точки Решение задач по физикеРешение задач по физикебудет время падения парашютиста Решение задач по физике с раскрытым парашютом будет Решение задач по физике Следовательно, 
Решение задач по физике
Решая это квадратное уравнение, находим 
Решение задач по физике 
Второй случай, когда парашютист Решение задач по физике не сразу раскрыл парашют, предлагаем рассмотреть читателям самостоятельно. 

Задача № 15. Тело, двигаясь прямолинейно с ускорением Решение задач по физике  достигло скорости Решение задач по физике а затем, двигаясь равнозамедленно,  остановилось через 10 сек. Определить путь, пройденный телом за все время движения. 
Дано: 
Решение задач по физике 

Решение:
Задача просто решается графически (рис. 6). Из графика  скорости следует, что путь численно равен площади треугольника Решение задач по физике 
Решение задач по физике

Решение задач по физике

Рис.6.

Но Решение задач по физике
Следовательно, 
Решение задач по физике

Задача № 16. Какой угол наклона должна иметь крыша, чтобы вода стекала за минимальное время? Ширина крыши Решение задач по физике
 

Решение:
Определим Решение задач по физике (рис. 7): 
Решение задач по физике 
где 
Решение задач по физике 

Решение задач по физике

Рис.7.


Ускорение движения капли воды 
Решение задач по физике 
откуда 
Решение задач по физике
или 
Решение задач по физике
Время стекания капли будет наименьшим, когда Решение задач по физике будет иметь наибольшее значение, т.е. при Решение задач по физике

Задача № 17. Снаряд вылетает из орудия с начальной скоростью Решение задач по физике под углом Решение задач по физике к горизонту. Найти высоту, дальность и время полета снаряда, не учитывая вращения снаряда и сопротивления воздушной среды. 
Дано: 
Решение задач по физике

Решение:
Составляющие скорости по осям Решение задач по физике и Решение задач по физике (рис. 8) в начальный момент времени равны: 
Решение задач по физике

Составляющая Решение задач по физике остается неизменной в течение всего времени полета снаряда. Составляющая же Решение задач по физике изменяется, согласно  
уравнению скорости равнопеременного движения: 
Решение задач по физике

Решение задач по физике
Рис.8. 


В наивысшей точке траектории Решение задач по физике т. е. Решение задач по физике  откуда время подъема снаряда до наивысшей точки 
Решение задач по физике 
Время полета снаряда 
Решение задач по физике
Для определения высоты Решение задач по физике снаряда воспользуемся формулой пути равпозамедленного движения: 
Решение задач по физике
Для определения дальности полета снаряда воспользуемся формулой прямолинейного равномерного движения: 

Решение задач по физике

Задача № 18. Из точки Решение задач по физике свободно падает тело. Одновременно из точки Решение задач по физике под углом Решение задач по физике к горизонту бросают- другое тело так, чтобы оба тела столкнулись в воздухе. 

Показать, что угол Решение задач по физике не зависит от начальной скорости Решение задач по физике тела, брошенного из точки Решение задач по физике и определить этот угол, если Решение задач по физике
Сопротивлением воздуха пренебречь. 
Дано: 
Решение задач по физике

Решение:
Оба тела могут встретиться на линии Решение задач по физике (рис. 9) в точке Решение задач по физике Разложим скорость Решение задач по физике тела, брошенного из точки Решение задач по физике на горизонтальную Решение задач по физике и вертикальную Решение задач по физике составляющие: 
Решение задач по физике
От начала движения до момента встречи пройдет время 
Решение задач по физике
За это время тело из точки Решение задач по физике опустится на величину 

Решение задач по физике

а тело из точки Решение задач по физике поднимется на высоту 
Решение задач по физике
Решая совместно последние два уравнения, находим 
Решение задач по физике
Подставляя значение Решение задач по физике имеем 

Решение задач по физике
т. е. угол_ бросания а не зависит от начальной скорости Решение задач по физике отсюда Решение задач по физике

Задача № 19. С башни брошено тело в горизонтальном направлении со скоростью Решение задач по физике Какова скорость тела через Решение задач по физике после начала движения? Какой угол образует с плоскостью горизонта вектор скорости тела в этот момент? 

Дано: 
Решение задач по физике

Решение:
Тело одновременно движется равномерно в горизонтальном направлении со скоростью Решение задач по физике и находится в свободном падении со скоростью Решение задач по физике (рис. 10). Численное значение скорости тела через время Решение задач по физике после начала движения будет 
Решение задач по физике
Направление вектора скорости определяется углом Решение задач по физике Из рис. 10 
видно, что 
Решение задач по физике

Движение тела, брошенного под углом к горизонту

Задача № 20. С самолета, летящего со скоростью Решение задач по физике отделяется Решение задач по физике тело. Найти радиус кривизны в точке траектории, где оно  будет через 5 сек после начала движения. Сопротивлением воздуха пренебречь. 
Дано: 
Решение задач по физике 

Решение:
Для определения радиуса кривизны в точке траектории через время Решение задач по физике после начала движения воспользуемся формулой центростремительного ускорения 
Решение задач по физике

откуда 
Решение задач по физике
где Решение задач по физике — скорость тела (см. рис. 10) через время Решение задач по физике после начала движения. Из рис. 10 следует, что . 
Решение задач по физике
или 

Решение задач по физике
Направление вектора и определяется углом Решение задач по физике

Решение задач по физике
Центростремительное ускорение численно равно (см. рис. 10): 
Решение задач по физике
Подставив в формулу радиуса кривизны вместо Решение задач по физике найденные значения, получим 
Решение задач по физике 

Задача № 21. С вершины горы бросают под углом Решение задач по физике к горизонту камень с начальной скоростью Решение задач по физике (рис. 11). Угол наклона горы к. горизонту также составляет Решение задач по физике На каком расстоянии от точки  бросания упадет камень? 
Дано: 
Решение задач по физике

Решение:
Движение камня по параболе можно рассматривать как сумму двух независимых прямолинейных движений. В данном случае в качестве составляющих движений удобно брать движения вдоль наклонной плоскости и перпендикулярно к ней. При таком  рассмотрении оба составляющие движения прямолинейны и  равнопеременные. 

Поместим начало координат в точке бросания, ось Решение задач по физике направим параллельно наклонной плоскости вниз, ось Решение задач по физике —  перпендикулярно плоскости вверх. 
Как видно из рис. 11, 
Решение задач по физике

Решение задач по физике

Рис.11.


На основании предыдущих выражений закон движения камня в выбранной системе координат можно записать так: 
Решение задач по физике
Отсюда следует, что в точке падения камня — точке Решение задач по физике — при Решение задач по физикеРешение задач по физике 
Решение задач по физике
Подставив в уравнение 
Решение задач по физике
значения Решение задач по физике и заменив Решение задач по физике предыдущим выражением, получим 
Решение задач по физике
Произведем вычисления; 
Решение задач по физике 

Задача № 22. Найти угловую скорость блока (диаметр 10 см) машины Атвуда через Решение задач по физике после начала движения грузов, если более тяжелый груз опустился за это время на Решение задач по физике (рис. 12). 

Решение задач по физике

Рис.12.


Дано: 
Решение задач по физике 

Решение:
Линейная скорость Решение задач по физике какой-либо точки вращающегося тела, кратчайшее расстояние этой  
точки от оси вращения Решение задач по физике и его угловая скорость Решение задач по физике связаны формулой 
Решение задач по физике 
где Решение задач по физике — кратчайшее расстояние от оси вращения до точки вращающегося тела. 
Для определения Решение задач по физике необходимо найти только линейную скорость какой-либо точки обода блока. 
Очевидно, линейная скорость точки обода блока в любой  момент времени будет равна скорости движения грузов при отсутствии скольжения нити. 
Так как грузы движутся под действием постоянной силы, равной разности весов грузов, то движение их будет равноускоренным. 
Поэтому скорость их, а следовательно, и скорость точек обода блока через Решение задач по физике секунд после начала движения составит 
Решение задач по физике 
где Решение задач по физике — ускорение, с которым движутся грузы. 
Из выражения 
Решение задач по физике
найдем ускорение Решение задач по физике
Решение задач по физике

Подставив в  уравнение Решение задач по физике вместо Решение задач по физике его значение, получим 
Решение задач по физике
Подставив это выражение в первое уравнение, найдем 
Решение задач по физике
Произведем вычисления: 
Решение задач по физике

Задача № 23. Определить скорость и ускорение точки, находящейся па поверхности Земли (в Минске), принимая во внимание только вращение Земли вокруг оси. Широта Минска — Решение задач по физике радиус Земли принять равным Решение задач по физике 
Дано: 
Решение задач по физике

Решение:
Для определения скорости Решение задач по физике точки Решение задач по физике воспользуемся формулой 
Решение задач по физике 
- где Решение задач по физике — радиус окружности сечения  земной сферы плоскостью, проходящей через точку Решение задач по физике с широтой в Решение задач по физике перпендикулярно к оси вращения Земли; Решение задач по физике — время одного полного оборота Земли вокруг своей оси. Из рис. 13 видно, что Решение задач по физике( Решение задач по физике— радиус Земли; Решение задач по физике — угол, образованный радиусом Земли, проходящим через указанную точку Решение задач по физике с плоскостью экватора).

 Решение задач по физике

Ускорение определяется по формуле 
Решение задач по физике

Задача № 24. Вал начинает вращаться равноускоренно. За первые Решение задач по физике он делает Решение задач по физике оборотов. Определить его угловую скорость к концу десятой секунды. 
Дано: 
Решение задач по физике 

Решение:
Для равноускоренного вращения угловая скорость Решение задач по физике где Решение задач по физике — угловое ускорение. Угол поворота вала за время Решение задач по физике будет 

Решение задач по физике
Исключая ускорение Решение задач по физике получаем 
Решение задач по физике
После подстановки численных значений 
Решение задач по физике

Задача № 1.1. Два пассажира, имея секундомеры, решили найти скорость поезда: первый по стуку колес на стыках рельсов (зная, что длина рельса равна 10 м), а второй по числу телеграфных столбов, мелькавших в окне (зная, что расстояние между столбами равно 50 м). Первый пассажир при первом стуке колес пустил в ход свой секундомер и на 156-м стуке его остановил. Оказалось, что прошло 3 мин. Второй пассажир пустил в ход свой секундомер при появлении в окне 1-го столба и остановил секундомер при появлении 32-го столба. Оказалось, что и его опыт длился 3 мин. Первый пассажир нашел, что скорость поезда равна 31,2 км/ч, а второй — 32 км/ч. Кто из них ошибся и почему? Какова скорость поезда в действительности?

Решение:

Так как первый пассажир определил, что скорость поезда равна 31,2 км/ч, то, следовательно, за 3 мин = 0,05 ч поезд прошел расстояние 31,2 · 0,05 = 1,56 км = 1560 м. Ясно, что это расстояние пассажир получил, умножив 10 на 156, а ему следовало умножить 10 на 155, ибо первый рельс пройден к моменту второго стука. Второй пассажир определил скорость поезда в 32 км/ч. Значит, по ero подсчету за 3 мин поезд прошел расстояние 32 · 0,05 = 1,6 км = 1600 м. Видимо, второй пассажир умножил 50 на 32, тоrда как ему следовало умножить на 31, ибо первый отрезок пути в 50 м пройден к моменту появления второго столба. Значит, оба пассажира ошиблись, начав счет рельсов и столбов не от нуля, а от единицы. Истинная скорость поезда была 155 х 10/0,05 = 31000 м/ч = 31 км/ч или 50 · 31/0,05 = 31000 м/ч = 31 км/ч.

Задача № 1.2. Переход пароходов из порта А в порт В длится ровно 12 суток. Каждый полдень из А в В и из В в А отходит по пароходу. Сколько пароходов встретит в открытом море каждый из
вышедших пароходов?
Решение задач по физике
Рис. 1

Решение:

Задача № 1.2. Пароход, вышедший из порта А, встретит, во-первых, те вышедшие из порта В пароходы, которые уже находятся в пути, и, во-вторых, те, которые выйдут из В за время пути. В момент выхода парохода из А в пути находятся 12 пароходов, считая тот, который вышел в этот момент из В (но не считая тоrо, который в этот момент прибыл в А). Кроме тоrо, за 12 суток пути парохода, вышедшего из А, из В выйдет 11 пароходов (не считая тоrо, который выйдет из В в момент прибытия парохода из А). Таким образом, в открытом море каждый пароход встретит 12 + 11 = 23 парохода.

Решение может быть пояснено графиком движения пароходов (рис. 161). Отложим по оси ординат расстояние между портом А и nортом В, а по оси абсцисс время пароходов в пути и построим графики движения каждого парохода, вышедшего из порта В, и одноrо парохода, вышедшего из А.

Решение задач по физике

Рис. 161

По пересечениям этих линий сразу видно, что каждый пароход встретит 23 парохода в открытом море, и, кроме того, произойдут встречи: в порту А — в момент отплытия и в порту В — в момент прибытия.

Относительность движения

Задача № 1.3. Какую экспозицию нужно делать при фотографировании автомобиля, движущегося со скоростью v = 36 км/ч, чтобы его изображение на негативе не размылось (рис. 1), если для этого смещение изображения должно быть не более 0,1 мм? Длина автомобиля L = 3 м, а размер его изображения на негативе получается равным = 1,5 см.

Решение:

Пусть искомое время равно t секунд. Тогда автомобиль успеет проехать за это время расстояние, равное vt. За это же время смещение изображения должно быть не более 0,1 мм. Отношение этих величин должно, очевидно, равняться отношению длины автомобиля к размеру eгo изображения, т. е. L/l = 300/1,5 = 200. Следовательно,

Решение задач по физике  отсюда   Решение задач по физике

Задача № 1.4. Автомобиль прошел расстояние от пункта А до пункта В со скоростью v1 = 40 км/ч и обратно со скоростью v2 = 30 км/ч. Какова средняя скорость vcp автомобиля?

Решение:

Обычно на этот вопрос отвечают, что средняя скорость автомобиля равна 35 км/ч. Но это неверно. Так было, если бы автомобиль двигался одинаковые промежутки времени с каждой из скоростей. Однако из условия задачи ясно, что одно и то же расстояние s (туда и обратно) автомобиль проходит за разные времена t1 и t2 , так как s = v1t1 = v2t2, Поэтому s = 40t1 = 30t2, откуда t1 = (3/4)t2. Средняя скорость равна

Решение задач по физике

Задача № 1.5. Мальчик бросает мячи один за другим вверх, каждый следующий мяч в тот момент, когда предыдущий находится в наивысшей точке. На какую высоту h поднимаются мячи, если он бросает 2 мяча в секунду? Считать ускорение свободного падения g = 9,8 м/с2.

Решение:

Пусть мальчик бросает n мячей в секунду. Тогда время полета каждого мяча вверх t = (1/n) секунд. Время полета мяча вверх равно времени его падения. Но расстояние и время падения связаны между собой соотношением h = gt2/2 = g/2n2. Следовательно, высота, на которую поднимаются мячи, равнаРешение задач по физике

Задача № 1.6. Камень, падающий свободно с нулевой начальной скоростью, пролетел вторую половину пути за 1 с. С какой высоты h он падал? Считать ускорение свободного падении
g = 9,8 м/с2.

Решение:

Уравнение движения камня: Решение задач по физике откуда Решение задач по физике По условию задачи Решение задач по физике откуда Решение задач по физике = 63 м.

1.7. Два камня падают в шахту. Второй камень начал свое падение на 1 с позже первого. Определить движение первого камня относительно второго. Считать ускорение свободного падения g = 9,8 м/с2.

Решение:

Задача № 1.7. Оба камня движутся относительно Земли равноускоренно с одинаковым постоянным ускорением g. Очевидно, один камень относительно другого движется равномерно, причем постоянная скорость первого камня по отношению .ко второму равна той скорости, которую первый камень приобрел за 1 с, т. е. за время, прошедшее между начальными моментами движения обоих камней.

Нетрудно провести и соответствующий расчет. Путь первого камня определяется уравнением Решение задач по физике путь второго — уравнением Решение задач по физике Расстояние между камнями растет со временем по закону Решение задач по физике т. е. первый камень движется относительно второго равномерно с постоянной скоростью, численно равной g.

Задача № 1.8. Тело движется по прямой равноускоренно под действием постоянной силы F. Как изменится график скорости этого движения, если сила F начнет уменьшаться?

Решение:

Под действием постоянной силы F скорость тела растет по закону v = v0 + at, rде v0 — начальная скорость тела, а — ero ускорение (график скорости v — прямая 1 на рис. 162). Если сила F начнет уменьшаться, движение все же будет оставаться ускоренным, скорость будет продолжать нарастать, но так как модуль ускорения а = F/m будет тоже уменьшаться, то уменьшится нарастание скорости. График скорости изобразится теперь кривой 2 на том же рис. 162. Когда cилa F, постепенно уменьшаясь, станет равной нулю, скорость достигнет постоянного значения. Движение станет равномерным. График скорости приблизится к горизонтальной прямой.

Решение задач по физике

Рис. 162

Задача № 1.9. Два самолета летят на встречных курсах со скоростью v = 200 м/с каждый. Из пулемета, расположенного на борту самолета 1 и стреляющего перпендикулярно курсу, обстреливается самолет 2 (рис. 2). На каком расстоянии s друг от друга должны расположиться пулевые отверстия в борту самолета 2, если пулемет делает 900 выстрелов в минуту? Какую роль играет при этом сопротивление воздуха?
Решение задач по физике                        Решение задач по физике
Рис. 2                                                                                            Рис. 3

Решение:

Самолеты движутся друг относительно друга со скоростью, равной сумме скоростей обоих самолетов, т. е. со скоростью v = 400 м/с. Промежуток времени между двумя выстрелами равен t = 1/900 мин = 1/15 с. Расстояние между пулевыми отверстиями должно быть равно относительному перемещению самолета 2 за это время, т. е.

Решение задач по физике

Если длина фюзеляжа самолета не превышает 27 м, то при указанных условиях стрельбы более чем одна пуля обычно в самолет попасть не может.

Если учитывать сопротивление воздуха, то каждой пуле потребуется большее время, чтобы попасть в самолет 2. Однако все пули будут запаздывать на одно и то же время. Поэтому промежуток времени между попаданием двух вылетающих друг за другом пуль остается по-прежнему равным 1/15 с и расстояние между пулевыми отверстиями по-прежнему должно быть равно 27 м.

Задача № 1.10. Бильярдный шар находится в точке А. Размеры бильярда и расстояние шара от лузы В даны на рис. 3. Под каким углом Решение задач по физике надо направить шар, чтобы попасть в лузу В после отражения шара от двух бортов? Считать, что при ударе о борт направление движения шара меняется по закону зеркального отражения, т. е. угол падения равен углу отражения.

Решение:

Допустим, шару сообщена скорость, равная v. Разложим ее на составляющие по направлениям сторон бильярда и рассмотрим путь шара, например такой, как указано на рис. 163. Запишем два уравнения, очевидные из чертежа:

Решение задач по физике

Из этих уравнений получаем

Решение задач по физике

т. е. находим угол Решение задач по физике под которым надо направить шap, чтобы попасть в лузу В.

Решение задач по физике

Рис. 163

Задача № 1.11. Даны три бильярда разной длины, но одинаковой ширины. От длинных бортов бильярдов (рис. 4) одновременно посылают шары с одинаковой по модулю и направлению
скоростью. Возможно ли, чтобы эти шары вернулись обратно к тому же борту не одновременно?

Решение задач по физике
Рис. 4

Решение:

У всех шаров составляющие скорости, параллельные короткому борту бильярда, в начальный момент времени одинаковы. При отражении от коротких бортов значение этих составляющих не меняется, и, следовательно, так как ширина бильярдов одинакова, все три шара достигнут другого борта бильярда одновременно (независимо от того, ударились ли они прежде о короткие борта или нет). При ударе о другой, длинный, борт рассматриваемая составляющая скорости изменяет направление, но ее модуль остается постоянным, следовательно, к тому борту, от которого они были посланы, все шары вернутся одновременно (рис. 164).

Решение задач по физике

Рис. 164

Задача № 1.12. Ведро выставлено под дождь. Изменится ли скорость наполнения ведра водой, если подует ветер?

Решение:

Скорость наполнения ведра водой (т. е. масса жидкости, попадающая в ведро в единицу времени) не изменится, так как, хотя площадь сечения падающего потока дождя уменьшается: Решение задач по физике (рис. 165, а), скорость капель дождя не только изменит направление, но и соответственно возраст до значения Решение задач по физике (рис. 165, б). Иначе говоря, скорость наполнения ведра водой зависит только от вертикальной составляющей скорости капель, которую ветер не изменяет.

Решение задач по физике

Рис. 165                                                                                Рис. 166

Задача № 1.13. На движущейся горизонтально со скоростью v1 = 20 м/с тележке установлена труба (рис. 5). Под каким углом Решение задач по физике к горизонту нужно наклонить трубу, чтобы капля дождя, падающая отвесно со скоростью v2 = 60 м/с, упала на дно трубы, не задев ее стенок?

Решение задач по физике

Рис. 5

Решение:

Мы можем вместо движения тележки относительно капли вправо рассматривать движение капли относительно тележки влево с той же по модулю, но противоположно направленной скоростью  v1. При этом результирующая скорость капли будет слагаться из двух взаимно перпендикулярных скоростей: v2 и –v1 (рис. 166). По условию задачи требуется, чтобы эта результирующая скорость была параллельна оси трубы, т. е.  составляла с горизонтом угол Решение задач по физике определяемый соотношением Решение задач по физике = 3. Отсюда получаем искомый угол Решение задач по физике

Задача № 1.14. Лодочник для определения скорости течения воды в реке решил провести такой опыт. Он опустил в воду деревянный ковш, а сам начал грести вниз по течению. Через 40 мин. он достиг пункта А, находящегося на 1 км ниже места отправления, и повернул лодку назад. Поймав ковш, он снова повернул лодку по течению и через 24 мин. после этого снова достиг пункта А. Сколько времени возвращался лодочник до встречи с ковшом, если считать, что течение воды в реке и скорость лодки относительно воды постоянны, а также что на повороты лодки времени не тратилось? Чему была равна скорость течения? Какова скорость лодки относительно воды?

Решение:

По отношению к ковшу (который плывет по течению) скорость лодки вверх и вниз по реке должна быть одинакова. Значит, до встречи с ковшом лодочник возвращался также 40 мин. Далее, скорость лодки относительно берега вниз по течению равна 25 м/мин. (так как за 40 мин. она прошла 1 км до точки А). После встречи с ковшом лодка шла вниз по течению 24 мин., значит, место встречи находится на 600 м выше точки А или на 400 м ниже места падения ковша в воду. Таким образом, за 80 мин., прошедших с момента падения ковша в воду до момента встречи, ковш проплыл 400 м, т. е. скорость течения равна 5 м/мин., а скорость лодки относительно воды 20 м/мин.

Задача № 1.15. Почему в кино, когда автомобиль движется вперед, зачастую кажется, что его колеса вертятся назад?

Решение:

Это явление наблюдается, если частота смены кадров киноленты близка к частоте вращения колес автомобиля, но несколько больше ее: тогда за время смены кадров колeco успевает сделать несколько мeньше полного оборота. Например, в случае, изображенном на рис. 167, за время смены кадра колесо делает 3/4 оборота против часовой стрелки. Нам же кажется, что оно делает за то же время 1/4 оборота по часовой стрелке. Если за время смены кадров киноленты колесо успевает сделать полный оборот, нам будет казаться, что оно неподвижно.

Решение задач по физике

Рис. 167

Задача № 1.16. Если перед фонарем, который освещает падающие одну за другой капли воды, поставить диск с одним или несколькими отверстиями и привести его во вращение, то фонарь будет давать прерывистое освещение капель. Число вспышек будет зависеть от частоты вращения диска и от числа отверстий в нем. Такой способ освещения называется стробоскопическим; он позволяет наблюдать периодические явления, происходящие со столь большой частотой, что их нельзя наблюдать глазом при обычном освещении. Если подобрать число оборотов диска стробоскопа так, чтобы за время между двумя вспышками капли успевали пролететь путь, равный расстоянию между соседними каплями, то последние будут казаться неподвижными. Определить нужное для этого число оборотов диска, имеющего два отверстия, если расстояние между каплями s = 2 см, а высота, с которой падают капли, h = 22,5 см.

Решение:

Если диск стробоскопа, имеющего два отверстия, делает n оборотов в секунду, то общее число вспышек в секунду будет 2n, а промежуток времени между двумя вспышками t1 = 1/2n. Скорость капель в том месте, где они освещаются стробоскопом, равна

Решение задач по физике = 210 см/с = 2,1 м/с.

Так как скорость капель велика, а расстояние между каплями мало (s = 2 см), то мы можем считать, что это расстояние капля пролетает с постоянной скоростью. Тогда время, за которое капля пройдет путь s, равно t2 = s/v = 2/210 с. Если капли кажутся неподвижными, то t2 = t1, т. е. 1/2n = 2/210. Отсюда получаем искомую частоту вращения диска: Решение задач по физике = 52,5 об/с. Если за время между двумя вспышками капли успевают пройти путь, равный удвоенному, утроенному и т. д. расстоянию между ними, то они также будут освещаться в одинаковых положениях и казаться неподвижными. Следовательно, число оборотов диска может быть выбрано и в целое число раз меньшим, чем 52,5 об/с. Однако чем меньше число оборотов диска, тем большее время (при данной ширине щели) будет освещаться каждая капля и тем заметнее будет ее перемещение в течение этого времени. Поэтому картина будет становиться менее четкой.

Задача № 1.17. Диск с отверстиями, просверленными по окружностям на расстоянии s = 1 см друг от друга (рис. 6), освещен сзади лампой. Диск вращается с частотой n = 30 об/мин. На каком расстоянии r от центра диска мы увидим сплошной светящийся круг? (Человеческий
глаз не ощущает колебаний яркости, если они происходят чаще, чем 16 раз в секунду.)

Решение задач по физике
Рис. 6

Решение:

Мы увидим светящийся круг там, где освещенные отверстия успевают сменить друг друга за время, меньшее или равное 1/16 с. Так как отверстия отстоят друг от друга на расстояние s = 1 см, то, следовательно, их скорость должна быть равна v = s/t = 16 см/с. Таким образом, мы увидим сплошной светящийся круг там, где линейная скорость вращающегося диска Решение задач по физике 16 см/с. Из соотношения между линейной и угловой скоростями Решение задач по физике находим Решение задач по физике = 51 см, т. е. светящиеся отверстия сольются в сплошной круг на расстоянии r Решение задач по физике 5,1 см от центра диска.

Движение по окружности

Задача № 1.18. По горизонтальной плоскости катится без скольжения с постоянной скоростью v обруч радиусом R. Каковы ускорения различных точек обруча?

Решение:

Равномерное качение обруча без скольжения по горизонтальной плоскости можно рассматривать следующим образом. Пусть заданы два любых его положения. Тогда из одного положения в другое обруч можно перевести так: поступательно переместить обруч из первого положения во второе со скоростью центра масс обруча и равномерно повернуть вокруг него с такой же линейной скоростью, чтобы все точки обруча заняли места, соответствующие второму его положению (рис. 168). Так как это верно для двух любых положений обруча, то это справедливо и для двух как угодно в в близких положений. Следовательно, мы можем рассматривать равномерное качение без скольжения как совокупность одновременно происходящих двух движений: равномерного поступательного движения со скоростью центра масс и равномерного вращения вокруг центра масс обруча с той же линейной скоростью точек на окружности обруча. Но при равномерном поступательном движении нет никакого ускорения, а при равномерном вращении вокруг центра у всех точек обруча одно и то же центростремительное ускорение, равное v2/R.

Решение задач по физике

Рис. 168

Задача № 1.19. Человек держит один конец доски, а другой ее конец лежит на цилиндре (рис. 7). Доска при этом горизонтальна. Затем человек двигает доску вперед, вследствие чего цилиндр катится без скольжения по горизонтальной плоскости; отсутствует также скольжение доски по цилиндру. Kaкoe расстояние должен пройти человек, чтобы
достичь цилиндра, если длина доски равна l?

Решение задач по физике
Рис. 7

Решение:

Когда цилиндр катится без скольжения по горизонтальной плоскости, то его образующая, касающаяся плоскости, неподвижна в данный момент, а сам цилиндр вращается вокруг этой неподвижной прямой. Очевидно, что в этом случае образующая, касающаяся доски, движется вперед со скоростью, вдвое большей той, с которой движется ось цилиндра. Поэтому, когда человек, толкая доску, пройдет путь, равный длине доски, то цилиндр уйдет вперед на расстояние, равное половине длины доски. Таким образом, чтобы дойти до цилиндра, человек должен пройти расстояние, равное 2l.

Это легко проверить с помощью круглого карандаша и линейки.

Задача № 1.20. На шероховатую горизонтальную плоскость бросают обруч радиусом R с линейной скоростью v1. Одновременно ему сообщается вращательное движение в таком направлении, что он должен катиться по плоскости в ту же сторону (рис. 8). При какой угловой скорости Решение задач по физике обруч покатится по плоскости без скольжения?

Решение задач по физике
Рис. 8

Решение:

Чтобы обруч катился по плоскости без скольжения, его центр масс должен двигаться с такой же скоростью, с какой движется tочка касания обруча и плоскости. Линейная скорость этой точки Решение задач по физике Следовательно, чтобы не возникало скольжение, должно выполняться равенство v2 v1, или Решение задач по физике.

Задача № 1.21. Почему когда колесо катится, то часто бывает, что нижние спицы видны отчетливо, а верхние спицы как будто сливаются?

Решение:

Когда колесо катится, то оно в каждый момент времени поворачивается вокруг точки касания с землей. Поэтому линейные скорости верхних спиц больше линейных скоростей нижних спиц, расположенных ближе к неподвижной в данный момент точке.

Задача № 1.22. С какой скоростью и в какое время суток должен лететь горизонтально самолет на широте Ленинграда ( Решение задач по физике = 60°), чтобы летчик видел Солнце все время на юге? Считать радиус Земли R = 6300 км.

Решение:

Самолет должен вылететь в полдень и лететь противоположно направлению вращения Земли, т. е. с востока на запад, со скоростью, равной линейной скорости Земли на широте Ленинграда. Угловая скорость вращения Земли Решение задач по физике рад/ч. Линейная скорость на широте Ленинграда равна (рис. 169) Решение задач по физике = 833 км/ч.

Двигаясь с такой скоростью с востока на запад, летчик будет видеть Солнце все время в том направлении, в каком оно было видно из Ленинграда, т. е. на юге.

Решение задач по физике

Рис. 169

Задача № 1.23. Два человека решили устроить дуэль на револьверах в необычных условиях: они стреляются, стоя на карусели радиусом R, вращающейся с угловой скоростью Решение задач по физике. Первый
дуэлянт стоит в центре О карусели, второй — на ее краю. Как они должны прицеливаться, чтобы поразить один другого? Какой из дуэлянтов находится в более благоприятных условиях? Считать, что пуля первого дуэлянта вылетает из точки О со скоростью, равной v1.

Решение:

Первый дуэлянт должен учитывать, что за время полета его пули второй дуэлянт переместится в другое положение. Время полета пули первого дуэлянта не зависит от вращения карусели: t1 = R/v1. За время t1 карусель повернется и точка А переместится в В (рис. 170, а), т. е. на расстояние, равное длине дуги АВРешение задач по физике поэтому первому дуэлянту надо стрелять не по направлению ОА, а по направлению ОВ. Угол Решение задач по физике находится из условия Решение задач по физике Второй дуэлянт, стоящий на краю карусели, движется со скоростью Решение задач по физике поэтому скорость его пули складывается из двух скоростей: v1 и v2. Чтобы попасть в центр карусели, он должен стрелять тоже не по направлению АО, а под некоторым. углом Решение задач по физике который находится из условия Решение задач по физике (рис. 170, 6). Пока Решение задач по физике т. е. оба дуэлянта должны прицеливаться одинаково, на один и тот же угол влево от противника (при указанном на рисунке направлении вращения карусели).

Решение задач по физике

Рис. 170

Однако с увеличением Решение задач по физике должен расти угол Решение задач по физике и результирующая скорость пули будет уменьшаться, а значит, будет уменьшаться и возможность поражения первого дуэлянта.

При Решение задач по физике т. е. второй дуэлянт должен был бы прицеливаться в направлении, противоположном направлению скорости его движения v2. Но при этом результирующая скорость пули была бы равна нулю.

Время полета пули второго дуэлянта t2 зависит от скорости вращения карусели:

Решение задач по физике

При Решение задач по физике получается, что время Решение задач по физике т. е. пуля дуэлянта, стоящего на краю карусели, "повисает" в воздухе в точке А, и он подъезжает под свою собственную пулю (в этом случае, как мы убедились выше, результирующая скорость пули равна нулю).

При Решение задач по физике результирующая скорость вообще не может быть направлена в точку О, т. е. первый дуэлянт не может быть поражен пулями второго. Первый же может поразить второго, если он правильно выбрал угол Решение задач по физике

Задача № 1.24. Центр квадрата совпадает с центром окружности, расположенной в той же плоскости; радиус окружности значительно меньше стороны квадрата. Из вершин квадрата одновременно начинают двигаться с равными постоянными скоростями собаки, каждая преследуя ближайшую, находящуюся впереди нее (все собаки в начальный момент смотрят в направлении вдоль стороны квадрата по часовой стрелке). Как расположены точки окружности, к которым прибегают собаки? Как направлены скорости собак в этот момент?

Решение:

В любой точке траектории скорость направлена по касательной к траектории в ту точку, где в данный момент находится исследуемая собака, так как собака, преследуя другую, не спускает с нее глаз и старается нагнать ее по кратчайшему пути (рис. 171). Поэтому их пути будут одинаково криволинейны и собаки одновременно достигнут окружности. Точки окружности, к которым одновременно прибегут собаки, будут вершинами квадрата, вписанного в эту окружность, а скорости собак в этот момент, очевидно, направлены по сторонам этого квадрата, т. е. стороны квадрата будут являться касательными к траекториям.

Решение задач по физике

Рис. 171

Механика

Задача № I.1.1 (15 баллов). Частица массой Решение задач по физике и импульсом Решение задач по физике упруго сталкивается с частицей массой Решение задач по физике которая первоначально находилась в покое.

а) Определите максимально возможный импульс частицы Решение задач по физике в лабораторной системе отсчета после столкновения. (Используйте формулы релятивистской теории.)

б) Примените полученный результат к случаю столкновения протона, импульс которого равен отношению энергии покоя протона к скорости света с, с неподвижным электроном. Найдите численное значение максимального импульса электрона в МэВ/с после столкновения.

Решение:

Пусть Решение задач по физике — импульсы частиц соответственно 1 и 2 после столкновения, а Решение задач по физике —максимально возможный импульс частицы массой Решение задач по физике, когда в системе, связанной с центром масс, частица 1 после столкновения движется назад. При этом Решение задач по физике и Решение задач по физике имеют одно и то же направление.

Решение задач по физике

а) Решение задач по физике можно найти из уравнений, выражающих законы сохранения импульса и энергии:

Решение задач по физике

Решение задач по физике

где

Решение задач по физике

Но удобнее использовать 4-векторы. Введем 4-векторы

Решение задач по физике

При этом для удобства мы приняли с = 1, а Решение задач по физике . Тогда уравнения (1) и (2) можно заменить одним уравнением

Решение задач по физике

или

Решение задач по физике

Рассмотрим скалярное произведение

Решение задач по физике

которое является инвариантом, и воспользуемся следующими соотношениями:

Решение задач по физике

где i, j — 1, 2, 3, 4. Отсюда имеем

Решение задач по физике

это соотношение можно упростить, учитывая, что Решение задач по физике параллельно Решение задач по физике. Таким образом, можно написать уравнение

Решение задач по физике

откуда получаем

Решение задач по физике

При этом мы отбросили второе, тривиальное решение уравнения (5), а именно Решение задач по физике = 0.

б) Дано

Решение задач по физике

Следовательно,

Решение задач по физике

Подставляя в (6) эти значения и Решение задач по физике = 0,5 МэВ, находим максимальный импульс электрона после столкновения:

Решение задач по физике

Задача № I.1.2 (15 баллов). На гладкой ровной поверхности обыкновенного стола лежит тонкий однородный стержень массой m и длиной L. На конце стержня перпендикулярно его оси действует импульс силы F, направленный горизонтально.

а) На какое расстояние передвинется центр масс стержня за время полного своего оборота?

б) Чему равны энергии поступательного и вращательного движений стержня и его полная кинетическая энергия после воздействия импульса силы?

Решение:

а) Пусть Решение задач по физике — скорость центра масс стержня, а Решение задач по физике — угловая скорость вращения стержня относительно центра масс.

Решение задач по физике

В случае импульсного воздействия справедливы следующие уравнения движения:

Решение задач по физике

и

Решение задач по физике

или

Решение задач по физике

где Решение задач по физике — момент инерции стержня относительно оси, проходящей через центр масс.

Если за время t стержень совершает полный оборот, то

Решение задач по физике

откуда

Решение задач по физике

Следовательно, за время полного оборота центр стержня переместится на расстояние

Решение задач по физике

Подставляя сюда выражение (2), находим

Решение задач по физике

б) Энергия поступательного движения

Решение задач по физике

Энергия вращательного движения

Решение задач по физике

Полная кинетическая энергия стержня равна их сумме:

Решение задач по физике

Задача № I.1.3 (15 баллов). Однородная балка массой m и длиной L поддерживается на своих концах двумя одинаковыми пружинами с жесткостью k. Балку приводят в движение, нажимая на один из ее концов, смещая его вниз на небольшое расстояние а и затем освобождая. Решите задачу о движении балки, вводя нормальные моды и частоты колебаний. Схематически изобразите нормальные моды.
Решение задач по физике

Решение:

Пусть b — длина каждой пружины в состоянии покоя (заметьте, что в поле силы тяжести она не совпадает с длиной свободной пружины: b = Длина свободной пружины — Решение задач по физике), а Решение задач по физике и x: — соответственно длины пружин 1 и 2 и высота центра масс стержня в момент времени t.

Решение задач по физике

Для абсолютно жесткого стержня Решение задач по физике. В соответствии со вторым законом Ньютона имеем

Решение задач по физике

или

Решение задач по физике

откуда

Решение задач по физике

Положим теперь

Решение задач по физике

Из уравнения

Решение задач по физике = Вращающий момент

получаем

Решение задач по физике

или, поскольку Решение задач по физике,

Решение задач по физике

Подставляя сюда Решение задач по физике, получаем

Решение задач по физике

Решения уравнений (1) и (3) записываются соответственно в виде

Решение задач по физике

Решение задач по физике

где

Решение задач по физике

Запишем начальные условия при t = 0:

Решение задач по физике

Подставляя (6) в (4) и (б), имеем

Решение задач по физике

Решение задач по физике

Решение задач по физике

Решение задач по физике

Из (8) и (10) следует

Решение задач по физике

а из (7) и (9) с учетом (11)

Решение задач по физике

После подстановки (11) и (12) в (4) и (5) получаем

Решение задач по физике

и

Решение задач по физике

Обе нормальные моды колебаний определяются выражениями

Решение задач по физике

Решение задач по физике

В случае Решение задач по физике = 0 имеем Решение задач по физике = —Решение задач по физике, или моду Решение задач по физике

Решение задач по физике

Если же Решение задач по физике = 0, то имеем Решение задач по физике или моду Решение задач по физике.

Решение задач по физике

Задача № I.1.4 (10 баллов). Дана матрица А из N X N действительных элементов.

а) Пусть А — симметрическая матрица, т. е.Решение задач по физике Докажите, что все ее собственные значения действительны.

б) Пусть А не является симметрической матрицей; докажите, что любые ее комплексные собственные значения образуют сопряженные пары.

Решение:

а) Матрица А — симметрическая, т. е. Решение задач по физике. Мы имеем

Решение задач по физике

здесь Решение задач по физике — собственный вектор матрицы А и а — соответствующее ему собственное значение матрицы А. Умножим (1) на Решение задач по физике слева и просуммируем по всем i:

Решение задач по физике

Перейдем от (1) к его комплексно сопряженному выражению и умножим обе его части на Решение задач по физике слева:

Решение задач по физике

Решение задач по физике

i, I    I

В равенстве (4) Решение задач по физике можно поменять местами:

Решение задач по физике

Поскольку А — действительная матрица, то Решение задач по физике = А. Более того, если она еще и симметрическая, то Решение задач по физике. Меняя местами индексы Решение задач по физике в левой части выражения (5) и учитывая свойство Решение задач по физике, получаем

Решение задач по физике

Сравнивая выражения (2) и (6), заключаем, что Решение задач по физике, т. е. а — действительная величина.

б) Пусть а — одно из собственных значений, тогда

Решение задач по физике

здесь Решение задач по физике — элементы действительной матрицы, которая не является симметрической. Перейдем в (7) к комплексно сопряженным величинам:

Решение задач по физике

Отсюда следует, что а* также представляет собой собственное значение матрицы А.

Задача № I.1.5 (15 баллов). В северном полушарии на широте 45° с высоты h (h много меньше радиуса Земли) падает покоившееся вначале тело массой m. В каком месте оно упадет относительно отвеса, опущенного из исходной точки? Не забудьте указать не только величину, но и направление смещения.

Решение:

В системе отсчета, вращающейся с угловой скоростью to, уравнение движения записывается в виде

Решение задач по физике

где

Решение задач по физике

—    действующее ускорение свободного падения, а Решение задач по физике=Решение задач по физике рад/с — угловая скорость вращения Земли. В результате уравнение (1) принимает вид

Решение задач по физике

Решение задач по физике

В системе отсчета, показанной на рисунке, получаем 

Решение задач по физике

Решение задач по физике

Следовательно,

Решение задач по физике

и

Решение задач по физике

Из выражения (3) имеем

Решение задач по физике откуда Решение задач по физике

Подставляя выражение для t в (5), находим

Решение задач по физике

Тело упадет к востоку от отвеса, опущенного из исходной точки.

Задача № I.1.6 (15 баллов). Упругая струна длиной L подвешена между двумя опорами. Натяжение струны Т, а ее линейная плотность р. Общая масса струны Решение задач по физике Колебания струны возбуждаются ударом молоточка, который сообщает небольшому участку длиной а в середине струны поперечную скорость Решение задач по физике Определите амплитуды первых трех низкочастотных гармоник.

Решение:

Движение струны описывается уравнением

Решение задач по физике

где Т — натяжение, а р — линейная плотность струны.

Решение задач по физике

Начальные условия при t = 0 записываются в виде y = 0 при всех значениях х,

Решение задач по физике

Запишем общее решение уравнения (1):

Решение задач по физике

При t = 0 имеем у = 0. Следовательно,

Решение задач по физике при любом значении X.

Это справедливо только в том случае, когда все коэффициенты Решение задач по физике равны нулю:

Решение задач по физике

Таким образом, из решения (2) имеем

Решение задач по физике

Отсюда следует

Решение задач по физике

Искомые амплитуды Решение задач по физике первых трех низкочастотных гармоник можно определить, подставляя в (5) значения п= 1, 2 и З).

Задача № I.1.7 (5 баллов). Твердое тело движется в воздухе с очень большой скоростью V (превышающей среднюю скорость движения молекул воздуха). Докажите, что сила сопротивления пропорциональна Решение задач по физике, где А — площадь лобовой поверхности тела.

Решение:

Число молекул воздуха, которым за промежуток времени Решение задач по физике сообщена скорость V, пропорционально AVРешение задач по физике. Следовательно, полный импульс Решение задач по физике этих молекул пропорционален Решение задач по физике. Сила сопротивления воздуха

Решение задач по физике
как видно из предыдущего, пропорциональна Решение задач по физике.

Задача № I.1.8 (5 баллов). Какой стала бы продолжительность суток на Земле, если Земля вращалась бы с такой скоростью, что тела на экваторе парили в невесомости?

Решение:

Чтобы тела парили в невесомости, центробежная сила на экваторе должна уравновешивать силу земного притяжения, т. е.

Решение задач по физике

Подставляя сюда Решение задач по физике м и g — Решение задач по физике, получаем

Решение задач по физике

откуда

Решение задач по физике

Примерно такой период обращения имеют искусственные спутники Земли на малых высотах.

Задача № I.1.9 (5 баллов). Дайте определение циклической координаты. Какие преимущества она дает при решении задач?

Решение:

Циклическими называются такие координаты, от которых гамильтониан не зависит в явном виде. Поскольку

Решение задач по физике

то импульсы, сопряженные с циклическими координатами, являются интегралами движения.

Задача № I.2.1 (10 баллов).

а) (5 баллов). Покажите, что косоэрмитова матрица Решение задач по физике может иметь только одно вещественное собственное значение, и оно равно нулю.

б) (5 баллов). Используя метод вычетов, вычислите Решение задач по физике

Решение:

а) Пусть а — действительное собственное значение косоэрмитовой матрицы А, т. е.

Решение задач по физике

Умножая обе части равенства (1) на Решение задач по физике слева, получаем

Решение задач по физике

Для эрмитово-сопряженной матрицы имеем

Решение задач по физике

(здесь мы воспользовались условием а* = а). Если мы умножим обе части равенства (3) на Решение задач по физике справа, то получим

Решение задач по физике

Но Решение задач по физике, и (4) принимает вид

Решение задач по физике

Из (2) и (5) следует

Решение задач по физике

б) Нам нужно вычислить
Решение задач по физике

Поскольку функция sin z не определена при Решение задач по физике, мы должны выразить функцию Решение задач по физике. Имеем

Решение задач по физике

Решение задач по физике

В верхней полуплоскости Решение задач по физике убывает экспоненциально, и, следовательно, интеграл

Решение задач по физике

по дуге Решение задач по физике обращается в нуль. Теперь можно вычислить I, заменяя вначале х на z и выполняя затем интегрирование вдоль границы верхнего полукруга:

Решение задач по физике

Решение задач по физике

где Решение задач по физике— вычет подынтегральной функции в точке z = 0. В данном случае подынтегральная функция имеет в этой точке полюс порядка n = 2. Таким образом,

Решение задач по физике

Окончательно получаем

Решение задач по физике

Задача № I.2.2 (15 баллов).

а) (10 баллов). Сколь огромную энергию пришлось бы затратить, чтобы «разнести» всю Землю (т. е. удалить все ее составные части на Решение задач по физике)?

б) (5 баллов). Предположим, что затем произошло бы быстрое (без потерь энергии на излучение) воссоединение разнесенных на бесконечность этих частей Земли. Расплавилась бы она? Свой ответ обоснуйте.

Решение:

а) Будем производить эту разрушительную работу постепенно, удаляя каждый раз с Земли тонкий слой толщиной dr.
Чтобы удалить такой слой на бесконечность, нужно затратить энергию, или совершить работу, численно равную потенциальной

Решение задач по физике

энергии взаимодействия этого слоя с оставшейся массой, но взятую с обратным знаком:

Решение задач по физике

где Решение задач по физике — масса, заключенная внутри сферы радиусом r, Решение задач по физике —масса сферического слоя толщиной —dr и радиусом r, а р — плотность Земли. После подстановки получаем

Решение задач по физике

Полная работа, которую необходимо произвести, равна интегралу от правой части равенства (2) в пределах от r = R до r = 0:

Решение задач по физике

здесь М —масса, а R — радиус Земли.

б) Предположим, что вся потенциальная энергия Решение задач по физикеРешение задач по физике преобразуется в тепловую. Считая теплоемкость Земли постоянной, приходим к следующему соотношению:

Решение задач по физике

где Решение задач по физике — приращение температуры Земли вследствие поглощения ею выделенного тепла. Полагая

Решение задач по физике

находим

Решение задач по физике

Решение задач по физике
Действительно, Земля бы расплавилась.

Задача № I.2.3 (30 баллов).

а) (20 баллов). Жесткий стержень массой m и длиной а подвешен с обоих концов на невесомых нитях, имеющих одинаковую длину L.
Решение задач по физике

Стержень выводят из состояния покоя, прикладывая небольшой импульс силы Р к одному из его концов в направлении, перпендикулярном стержню и нити. Определите частоты и амплитуды нормальных мод колебаний, б) (10 баллов). Стержень находится в покое. Каково натяжение одной из нитей сразу после того, как другую мгновенно перережут?

Решение:

Решение задач по физике

а) Пусть Решение задач по физике —углы отклонения нитей от вертикали. Угловая скорость центра масс стержня

Решение задач по физике

Кинетическая энергия поступательного движения стержня равна

Решение задач по физике

Между углами Решение задач по физике существует соотношение

Решение задач по физике

Кинетическая энергия вращательного движения записывается в виде

Решение задач по физике

где Решение задач по физике — момент инерции стержни.

Потенциальная энергия (при малых углах Решение задач по физике) равна

Решение задач по физике

здесь мы воспользовались разложением функций Решение задач по физике в степенной ряд. Из выражений (1), (2) и (3) получаем выражение для лагранжиана:

Решение задач по физике

где Решение задач по физике

Из уравнения Лагранжа следуют два уравнения движения:

Решение задач по физике

Решение задач по физике

Суммируя оба уравнения и вычитая одно из другого, получаем

Решение задач по физике

или

Решение задач по физике

Решение задач по физике

или

Решение задач по физике

Решения уравнений (7) и (8) можно записать в виде

Решение задач по физике

Решение задач по физике

здесь Решение задач по физике — частоты, соответствующие этим решениям:

Решение задач по физике

Решение задач по физике

Удовлетворяя начальным условиям Решение задач по физике получаем

Решение задач по физике

Следовательно, 

Решение задач по физике

Решение задач по физике

Начальные значения Решение задач по физике при t = 0 можно найти из выражений для импульса
Решение задач по физике
Решение задач по физике
откуда

Решение задач по физике

Решение задач по физике
Из (12), (13), (16) и (17) определяем амплитуды нормальных мод:
Решение задач по физике

Решение задач по физике
б) Пусть Т — натяжение нити. В соответствии со вторым законом Ньютона имеем

Решение задач по физике

Решение задач по физике

Кроме того, имеем уравнение для вращающего момента относительно точки А:
Решение задач по физике

Подставляя Решение задач по физике из уравнения (19) в (18), находим
Решение задач по физике

Задача № I.2.4 (25 баллов). Частица массой т движется в поле центральной силы с потенциалом Решение задач по физике.

а) (10 баллов). Определите кинетическую энергию и момент импульса частицы при движении ее по круговой орбите (радиусом а).

б) (5 баллов). Чему равен период такого кругового движения?

в) (10 баллов). Каков период малых радиальных колебаний частицы относительно r=а, если ее движение под действием возмущений слегка отклонилось от кругового?

Решение:

а) В случае круговой орбиты сила притяжения —dV/dr должна быть равна центростремительной силе:

Решение задач по физике

откуда

Решение задач по физике

При этом кинетическая энергия частицы дается выражением

Решение задач по физике

а момент импульса частицы

Решение задач по физике

где использовано соотношение Решение задач по физике

б) Зная угловую скорость Решение задач по физике находим период кругового движения

Решение задач по физике

в) После воздействия возмущения расстояние частицы от центра будет меняться вблизи r = а по некоторому закону r = r(t). Запишем г в виде r = а + х, где х—малая величина. Движение по х в первом приближении описывается уравнением

Решение задач по физике

где

Решение задач по физике

Решение задач по физике

Подставляя полученные выражения для Решение задач по физике в (4), приходим к уравнению гармонического осциллятора

Решение задач по физике

с частотой

Решение задач по физике

Задача № I.2.5 (20 баллов). Полагают, что можно получить моноэнергетические фотоны высоких энергий путем рассеяния лазерного излучения на пучке быстрых электронов, выходящих из электронного ускорителя. Выведите формулу для максимальной энергии каждого из рассеиваемых фотонов в зависимости от энергии фотонов лазерного излучения и энергии электронов в пучке. Выполните численные расчеты в случае излучения рубинового лазера, которое рассеивается на электронах с энергией 20 ГэВ, получаемых от Стэнфордского линейного ускорителя.

Решение:

Введем следующие 4-векторы, условившись считать с = 1 и Решение задач по физике:

Решение задач по физике

Из законов сохранения импульса и энергии следует

Решение задач по физике

или

Решение задач по физике

После возведения в квадрат получаем

Решение задач по физике

Решение задач по физике

(Замечание. Вернитесь к решению задачи 1.1.1 и обратите внимание на некоторые отличия, возникшие на данном этапе решения.) Отсюда находим Решение задач по физике:

Решение задач по физике

При энергии электронов 20 ГэВ величина Решение задач по физике ничтожно мала по сравнению с Решение задач по физике, следовательно, Решение задач по физике. Чтобы получить максимальную величину Решение задач по физике, следует принять Решение задач по физике = —1, Решение задач по физике = 1 и Решение задач по физике = —1. При этом вектор Решение задач по физике по направлению совпадает с Решение задач по физике и противоположен Решение задач по физике

Решение задач по физике

Из (2) следует

Решение задач по физике

т. е. энергия рассеянного фотона почти совпадает с энергией встречного электрона

Задача № I.3.1 (15 баллов). Шарик массой М и радиусом а скатывается без скольжения по наклонной плоскости, образующей с горизонтом угол Решение задач по физике

Решение задач по физике

а) Определите ускорение центра масс шарика.

б) Если шарик толкнуть вверх по наклонной плоскости с начальной скоростью Решение задач по физике то через какой промежуток времени он вернется в исходную точку?

Решение:

Решение задач по физике

а) Пусть Р — положение мгновенного центра вращения шарика на наклонной плоскости в момент времени t. Вращение шарика относительно этого центра описывается уравнением

Решение задач по физике

где Решение задач по физике — момент инерции шарика относительно оси, проходящей через точку Р. Согласно теореме о параллельных осях), имеем

Решение задач по физике

Следовательно, ускорение центра масс шарика равно

Решение задач по физике

б) Из закона сохранения энергии следует, что в исходной точке шарик должен обладать той же скоростью Решение задач по физике с которой его толкнули, но противоположно направленной. Для движения с постоянным ускорением

Решение задач по физике

где Решение задач по физике — начальная и конечная скорости соответственно. Подставляя сюда Решение задач по физике получаем

Решение задач по физике

откуда

Решение задач по физике

Задача № I.3.2 (15 баллов). Нестабильная частица с энергией покоя 1000 МэВ и средним временем жизни в состоянии покоя Решение задач по физике с распадается на мю-мезон Решение задач по физике и нейтрино.

а) Вычислите среднюю длину распада, если частица имеет импульс 1000 МэВ/с.

б) Какова энергия мю-мезона в последнем случае, если он испущен под углом 15°?

Решение:

а) Частица имеет массу покоя Решение задач по физике = 1000 Решение задач по физике и обладает импульсом Р= 1000 МэВ/с. Скорость частицы равна)

Решение задач по физике

Найдем среднее время жизни частицы

Решение задач по физике

Следовательно, средняя длина распада частицы равна

Решение задач по физике

б) Введем 4-векторы:

Решение задач по физике

В соответствии с законом сохранения энергии и импульса имеем
Решение задач по физике
Возведем обе части уравнения (4) в квадрат:

Решение задач по физике

или

Решение задач по физике

Подставляя известные значения Решение задач по физике и получаем

Решение задач по физике

При столь высокой энергии мю-мезона членом Решение задач по физике можно пренебречь и положить Решение задач по физике Таким образом,

Решение задач по физике

Более подробно об операциях с 4-векторами см. в задаче 1.1.1.

Задача № I.3.3 (15 баллов). Рассмотрим две балки массами Решение задач по физике и Решение задач по физике и длиной Решение задач по физике и Решение задач по физике, жестко прикрепленные к горизонтальному невесомому стержню. Концы стержня лежат на опорах; стержень обладает крутильной жесткостью К и может вращаться только вокруг своей продольной оси. Система совершает колебания под действием силы тяжести. Напишите лагранжиан системы (введя свои переменные) и найдите уравнения движения.

Решение задач по физике

Решение:

Пусть Решение задач по физике — углы отклонения каждого из стержней от вертикали. Моменты инерции стержней равны

Решение задач по физике

Решение задач по физике

а сумма кинетических энергий

Решение задач по физике

Потенциальная энергия всей системы

Решение задач по физике

Лагранжиан системы записывается в виде

Решение задач по физике

Решение задач по физике

Решение задач по физике

(при этом мы произвели разложение Решение задач по физике в степенной ряд). Из выражения (4) находим

Решение задач по физике

После подстановки этих выражений в уравнение Лагранжа получаем следующие уравнения движения:

Решение задач по физике

Решение задач по физике

или

Решение задач по физике

Положим Решение задач по физике Тогда уравнения (6) принимают вид

Решение задач по физике

Преобразуем эти уравнения к виду
Решение задач по физике
Чтобы уравнения (8) не противоречили одно другому, должно выполняться следующее условие:
Решение задач по физике
откуда

Решение задач по физике

или

Решение задач по физике

Уравнение (10) можно решить и определить частоты Решение задач по физике Решение задач по физике) Подставляя найденные частоты Решение задач по физике (j= 1, 2, 3, 4) в уравнения (8), можно получить соответствующие этим частотам соотношения амплитуд Решение задач по физике. Таким образом, общее решение системы (6) записывается в виде

Решение задач по физике

Задача № I.3.4 (25 баллов). На шероховатой плоскости, наклоненной под углом Решение задач по физикегде Решение задач по физике — коэффициент трения как покоя, так и скольжения), расположена частица. К частице прикреплена нить, проходящая через небольшое отверстие в наклонной плоскости.
Решение задач по физике

Нить очень медленно подтягивают, и можно считать, что частица все время находится в почти статическом равновесии. Определите траекторию частицы на наклонной плоскости.

Решение:

Введем три единичных вектора: Решение задач по физике — в направлении кратчайшего спуска по наклонной плоскости, Решение задач по физике — в направлении касательной к траектории частицы и Решение задач по физике — вдоль нити по направлению к отверстию (см. рисунок).

Решение задач по физике

Условие квазистатического равновесия частицы, когда она едва движется, записывается в виде

Решение задач по физике

здесь Решение задач по физике — сила трения, действующая на частицу со стороны плоскости и всегда противоположная направлению движения, а Т — натяжение нити. По определению Решение задач по физике. Подставляя сюда Решение задач по физике, получаем

Решение задач по физике

Теперь уравнение (1) принимает вид

Решение задач по физике

Возведем обе части уравнения (2) в квадрат:

Решение задач по физике

После упрощения получаем

Решение задач по физике

где Решение задач по физике.

Если частица находится выше отверстия, т. е. когда

Решение задач по физике

то из уравнения (3) следует что Решение задач по физике. Таким образом, поскольку натяжение нити Т не может быть отрицательным, принимаем Т = 0.

Если же частица находится ниже отверстия, т. е. когда

Решение задач по физике

то натяжение Т оказывается положительным:

Решение задач по физике

Траекторию частицы определяет результирующая сила Решение задач по физике

Направление Решение задач по физике показано на рисунке, приведенном ниже.

Решение задач по физике

Траекторию частицы можно построить следующим образом. На участке от А до В натяжение нити бесконечно мало (но положительно) и частица просто соскальзывает вниз по наклонной плоскости. Пусть Решение задач по физике; из (4) мы получаем уравнение

Решение задач по физике

где

Решение задач по физике 

Это уравнение можно преобразовать к виду

Решение задач по физике

Оно имеет следующее решение:

Решение задач по физике

где С — постоянная, определяемая из условия С=у=ВР при x = 0.

Таким образом, траектория частицы между точками В и Р, описываемая уравнением (7), представляет собой полуокружность радиусом С/2 с центром в точке x=0, y=Cj2.

Задача № I.3.5 (15 баллов). Стержень массой М и длиной L свободно падает в вертикальной плоскости, как показано на рисунке.
Решение задач по физике

В начальном состоянии покоя стержень составлял с горизонталью угол 30°. Определите давление на ось вращения стержня в тот момент, когда он проходит горизонтальное положение.

Решение:

Пусть Решение задач по физике — вертикальная составляющая реакции опоры. Сила, действующая на опору по вертикали, равна — Решение задач по физике. Когда стержень проходит горизонтальное положение, он обладает кинетической энергией Решение задач по физике, которая соответствует разности потенциальных энергий стержня в начальном и конечном состояниях:

Решение задач по физике

здесь I — момент инерции стержня относительно оси вращения. Для однородного стержня, закрепленного на одном конце, I = Решение задач по физике. Следовательно,

Решение задач по физике

Ордината центра масс стержня Решение задач по физике, и мы имеем 2

Решение задач по физике

Решение задач по физике

Уравнение движения центра масс стержня имеет вид

Решение задач по физике

При горизонтальном положении стержня Решение задач по физике Вращательное движение стержня относительно центра масс описывается уравнением

Решение задач по физике

Подставляя сюда Решение задач по физике получаем

Решение задач по физике

Подставляя Решение задач по физике из выражения (3) в уравнение (2), получаем

Решение задач по физике

откуда

Решение задач по физике

Величина — Решение задач по физике представляет собой составляющую силы, действующую вертикально вниз на ось вращения стержня. Реакцию опоры Решение задач по физике можно найти из условия, что Решение задач по физике должна быть равна центростремительной силе, действующей на стержень. В момент времени, когда стержень проходит горизонтальное положение,

Решение задач по физике

Подставляя сюда выражение для Решение задач по физике из (1), находим

Решение задач по физике

Сила — Решение задач по физике направлена так, что она стремится сдвинуть ось вращения стержня влево.

Задача № I.3.6 (15 баллов). Движущийся протон сталкивается с другим протоном, который первоначально находился в покое. Какой минимальной кинетической энергией должен обладать движущийся протон, чтобы стала возможной реакция р + р = Решение задач по физике Масса антипротона Решение задач по физике равна массе протона. Представьте полученный вами результат в виде отношения кинетической энергии к энергии покоя протона. Каково приближенно численное значение кинетической энергии в МэВ?

Решение:

Пороговое значение энергии движущегося протона определяется выражением

Решение задач по физике

В случае реакции

Решение задач по физике

имеем

Решение задач по физике

Следовательно,

Решение задач по физике

Замечание. Вывод формулы (1) см. в книге: J. D. Jackson, Classical Electrodynamics, стр. 397—400). Другой подход см. в решении задачи IV. 11.7, разд. «Атомная физика и квантовая механика» настоящего сборника.

Задача № I.4.1 (15 баллов). Предположим, что приливные волны на Земле вызываются только Солнцем.

а) Откуда черпается энергия, рассеиваемая во время приливов, и чему равна максимальная величина этой энергии?

б) Какие процессы обеспечивают сохранение суммарного момента импульса системы?

Решение:

а) Приливные волны совершают работу за счет энергии вращения Земли. Максимальная энергия, которую могут рассеять приливные волны, равна кинетической энергии вращения Земли Решение задач по физике где I — момент инерции Земли, а Решение задач по физике — угловая скорость ее вращения.

б) Рассечем мысленно Землю пополам плоскостью, перпендикулярной плоскости земной орбиты и проходящей через центры Солнца и Земли. Поскольку форма Земли несколько отличается от шарообразной, одна из ее половин окажется несколько ближе к Солнцу, чем другая. Поэтому в соответствии с законом обратных квадратов она будет притягиваться к Солнцу сильнее, чем другая половина.

Пусть Решение задач по физике — радиус-векторы, определяющие положения центров масс двух половин Земли относительно Солнца, Решение задач по физике и Решение задач по физике — силы притяжения рассматриваемых половин Земли Солнцем и Решение задач по физике — радиус-вектор, определяющий положение центра масс первой половины относительно центра масс Земли. Таким образом, мы имеем )

Решение задач по физике

Вращающий момент, действующий на Землю со стороны Солнца, приблизительно равен нулю, и можно записать

Решение задач по физике

Здесь этот момент мы представили в виде суммы момента пары сил Решение задач по физике относительно центра масс Земли и момента силы F, приложенной к центру масс Земли, относительно Солнца. Момент пары С уменьшает угловую скорость вращения Земли, в то время как момент Решение задач по физике увеличивает орбитальный момент Земли Решение задач по физике С увеличением последнего возрастает радиус земной орбиты, а также период обращения Земли (в соответствии с законом Кеплера Решение задач по физике). Сохранение суммарного момента импульса наглядно видно из уравнения (1): уменьшение момента, связанного с суточным вращением Земли, приводит к немедленному возрастанию орбитального момента Земли на ту же самую величину.

Задача № I.4.2 (5 баллов). Время жизни мю-мезона составляет Решение задач по физике. Пучок мю-мезонов выходит из циклотрона со скоростью 0,8 с, где с — скорость света. Каково должно быть время жизни мю-мезонов в этом пучке в лабораторной системе отсчета?

Решение:

 Решение задач по физике

Задача № I.4.3 (5 баллов). На рисунке показан блок пренебрежимо малой массы, подвешенный к пружинным весам. К концам нити, переброшенной через блок, прикреплены грузы массами 1 и 5 кг. Грузы движутся с ускорением под действием силы тяжести. Какой вес покажут пружинные весы при движении грузов — 6 кг, меньше или больше?

Решение задач по физике

Решение:

Меньше.

Задача № I.4.4 (5 баллов). Из винтовки вылетает пуля. Если допустить свободную отдачу (т. е. движению винтовки не противодействует плечо стрелка), то будет ли кинетическая энергия винтовки после выстрела равна, меньше или больше кинетической энергии пули?

Решение:

Меньше.

Задача № I.4.5 (5 баллов). На рисунке заштрихованная область представляет однородную пластину в форме буквы L. Определите координаты Решение задач по физике ее центра масс.

Решение задач по физике

Решение:

 Решение задач по физике

Задача № I.4.6 (5 баллов). В стакане с водой при температуре 0°С плавает кубик льда. Лед растаял. Поднимется, упадет или останется прежним уровень воды в стакане?

Решение:

Останется прежним.

Задача № I.4.7 (5 баллов). Волчок вращается вокруг своей оси в направлении, указанном на рисунке. Острие волчка упирается в стол. По часовой стрелке или против нее прецессирует ось волчка, если смотреть на него сверху?

Решение задач по физике

Решение:

Из векторного соотношения между моментом внешней силы М, угловой скоростью прецессии Решение задач по физике и собственным моментом импульса L волчка

Решение задач по физике

следует, что вектор Решение задач по физике направлен вверх, поэтому ось волчка прецессирует против часовой стрелки.

Задача № I.4.8 (5 баллов). К концам пружины жесткостью k прикреплены грузы с массами m и 3m. Чему равен период колебаний системы?

Решение:

Приведенная масса системы равна

Решение задач по физике

Следовательно, период колебаний равен

Решение задач по физике

Задача № I.4.9 (5 баллов). Зависимость потенциальной энергии взаимодействия двух частиц V от расстояния r между ними дается выражением где а и b — положительные коэффициенты. На каком расстоянии r частицы находятся в статическом равновесии?

Решение задач по физике

Решение:

При статическом равновесии

Решение задач по физике

Таким образом,

Решение задач по физике

откуда

Решение задач по физике

Задача № I.4.10 (5 баллов). Две частицы с одинаковыми массами скреплены пружинами, как показано на рисунке. Частицы могут свободно колебаться в продольном направлении. Изобразите стрелками (или опишите) соотношения амплитуд и фаз колебаний частиц, соответствующих двум нормальным модам.(1 и 2).
Решение задач по физике

Решение:

а) Решение задач по физике колебания в противофазе (мода 1);

б) Решение задач по физике колебания в фазе (мода 2).

Задача № I.4.11 (5 баллов). Предположим, что радиус Земли сократился на 1%, а масса ее осталась неизменной. Увеличится или уменьшится ускорение свободного падения g на поверхности Земли? Если да, то на сколько процентов?

Решение:

Поскольку Решение задач по физике, то Решение задач по физике. Следовательно, уменьшение радиуса Земли на 1% привело бы к увеличению ускорения свободного падения g на 2%.

Задача № I.4.12 (5 баллов). Пустая цилиндрическая консервная банка и такая же банка, но с плотно набитым колбасным фаршем начинают одновременно скатываться по наклонной плоскости. Какая из банок скатится вниз первой?

Решение:

Пусть Р — мгновенная ось вращения банки в момент времени t. Запишем уравнение вращательного движения относительно Р:

Решение задач по физике

где I —момент инерции банки относительно оси Р, a Решение задач по физике — квадрат радиуса инерции банки. Для пустой банки

Решение задач по физике

а для заполненной

Решение задач по физике

Отсюда видно, что Решение задач по физике следовательно, Решение задач по физике Заполненная банка скатится первой.

Задача № I.4.13 (5 баллов). Цилиндрический стакан наполнен водой до уровня 40 см. В стенке стакана имеются два одинаковых отверстия: одно — на высоте 10 см, а другое — на высоте 30 см. Чему равно отношение масс воды в начальный момент времени, вытекающей за 1 с из обоих отверстий?
Решение задач по физике

Решение:

Решение задач по физике

здесь Решение задач по физике — давление воды у нижнего отверстия, а Решение задач по физике—у верхнего отверстия.

Задача № I.4.14 (5 баллов). Длинная натянутая струна прикреплена справа к жесткой стене. По струне слева направо распространяется со скоростью 1 м/с поперечное возмущение, имеющее вид равнобедренного треугольного импульса. В начальный момент времени вершина этого импульса отстоит от стены на расстояние 2 м. Нарисуйте профиль струны спустя 2 с и спустя 5 с от начала отсчета.

Решение задач по физике

Решение:

Решение задач по физике

Задача № I.4.15 (10 баллов). Формула для кориолисовой силы имеет вид

Решение задач по физике

а) В каком случае применяют эту формулу и какой смысл имеют входящие в нее символы?

б) Река в северном полушарии течет к югу. Ширина русла реки на широте Решение задач по физике равна W. Уровни воды на восточном и западном берегах реки должны быть различными. Какой из них будет выше?

в) Выведите формулу для разности уровней, подробно поясняя значение каждого используемого в ней символа.

г) Оцените весьма приближенно эту разность уровней для реки Миссисипи). Укажите числовые данные, которые вы приписали (пусть даже по догадке) различным параметрам.

Решение:

Кориолисова сила

Решение задач по физике

а) Она действует на частицу массой m, за которой наблюдают в системе координат, вращающейся с угловой скоростью Решение задач по физике относительно инерциалыюй системы; v —скорость этой частицы во вращающейся системе координат.

б) Кориолисова сила направлена на запад. Поэтому уровень воды на западном берегу выше, чем на восточном.

в)
Решение задач по физике

Поскольку Решение задач по физике то

Решение задач по физике

Разность уровней

Решение задач по физике

здесь v — скорость течения воды, Решение задач по физике — географическая широта, Решение задач по физике —угловая скорость вращения Земли, W — ширина реки, g — ускорение свободного падения на поверхности Земли.

г) Решение задач по физике

Решение задач по физике

Решение задач по физике

Подставляя эти значения в формулу (2), получаем

Решение задач по физике

Задача № I.4.16 (10 баллов). Диск плотно насажен на вал, проходящий через центр диска, причем ось симметрии диска Решение задач по физике образует с валом угол Решение задач по физике

Решение задач по физике

Главный центральный момент инерции диска относительно оси симметрии Решение задач по физике обозначим через С, а относительно любой оси Решение задач по физике', перпендикулярной Решение задач по физике, — через А. Вал вращается в подшипниках с постоянной угловой скоростью Решение задач по физике Определите величину вращающего момента, действующего на подшипники.

Решение:

Решение задач по физике

Как показано на рисунке, единичный вектор Решение задач по физике направлен вдоль оси диска, вектор Решение задач по физике перпендикулярен вектору Решение задач по физике и лежит в плоскости рисунка, а вектор Решение задач по физике перпендикулярен плоскости рисунка. Угловую скорость о можно разложить на две составляющие Решение задач по физике направлениям Решение задач по физике:

Решение задач по физике

Момент импульса диска записывается в виде

Решение задач по физике

Уравнение движения диска в системе координат, вращающейся с угловой скоростью Решение задач по физике имеет вид

Решение задач по физике

где производная Решение задач по физике относится к лабораторной системе, а d/dt — к вращающейся системе координат. Поскольку во вращающейся вместе с диском системе координат диск неподвижен, то второй член в правой части уравнения (3) равен нулю. Следовательно,

Решение задач по физике

где Решение задач по физике

Задача № I.5.1 (10 баллов). Брусок массой М лежит на идеально гладком горизонтальном столе и посредством пружин, характеризуемых жесткостью Решение задач по физике, связан с двумя неподвижными опорами.

а) Какова будет частота продольных колебаний бруска, если его слегка вывести из равновесного положения? Предположим, что амплитуда колебаний бруска равна Лив тот момент, когда он проходит положение равновесия, к нему прилипает упавший сверху груз массой m. Определите:

б) новую частоту колебаний,

в) новую амплитуду колебаний.
Решение задач по физике

Решение:

а) Пусть x—расстояние, на которое брусок смещается относительно положения равновесия. Из уравнения движения

Решение задач по физике

определяем частоту колебаний
Решение задач по физике
б) Новая система, масса которой равна Ai -f- m, имеет частоту колебаний

Решение задач по физике

в) Пусть Решение задач по физике — скорость бруска, с которой он проходит положение равновесия. В соответствии с законом сохранения энергии имеем

Решение задач по физике

где А — амплитуда колебаний. Запишем закон сохранения импульса

Решение задач по физике

где Решение задач по физике — скорость бруска с прилипшим к нему грузом т. Выразим переход кинетической энергии в потенциальную в виде

Решение задач по физике
Исключая из уравнений (4) — (6) и Решение задач по физике находим соотношение между новой амплитудой колебаний В и амплитудой А:

Решение задач по физике

Задача № I.5.2 (10 баллов). На гладком горизонтальном столе расположен клин с углом Решение задач по физике при вершине. Масса клина равна М. На наклонную поверхность клина помещают брусок массой m,который начинает скользить вниз. Трением между любыми поверхностями можно пренебречь. Чему равно ускорение клина относительно стола (до того момента, когда брусок коснется стола)?
Решение задач по физике

Решение:

Решение задач по физике

Пусть N — нормальная сила реакции, действующая на массу m со стороны клина. Уравнение движения бруска массой m в направлении у записывается в виде

Решение задач по физике

а в направлении х

Решение задач по физике

Уравнение движения клина массой М в направлении х имеет вид

Решение задач по физике

Поскольку брусок движется по клину, мы можем написать следующее уравнение связи:

Решение задач по физике

Из уравнений (2) и (3) имеем

Решение задач по физике

Подставляя N из уравнения (2) в (1) и учитывая (5), получаем

Решение задач по физике

Подставляя Решение задач по физике из (5) в (4), находим

Решение задач по физике

Наконец, исключая Решение задач по физике в (7) с помощью (6), получаем ускорение клина относительно стола

Решение задач по физике

Задача № I.5.3 (3 балла). К краю стола, как показано на рисунке, прикреплен блок, массой и коэффициентом трения которого можно пренебречь. Трение на поверхности стола также можно не учитывать. Через блок переброшена невесомая нить, к концам которой прикреплены два бруска массами 10 и 5 кг. Будет ли натяжение нити больше, равно или меньше, чем в том случае, когда брусок массой 5 кг приклеен к столу?
Решение задач по физике

Решение:

Натяжение нити меньше, чем в случае, когда брусок массой 5 кг приклеен к столу.

Задача № I.5.4 (3 балла). Диск вращается в горизонтальной плоскости вокруг своей оси. В узкую радиальную канавку, расположенную на верхней поверхности диска, вставлено лезвие безопасной бритвы. В каком направлении — в сторону или против вращения — начнет падать лезвие, если оно будет двигаться по направлению к центру диска?

Решение:

Кориолисова сила —Решение задач по физике действует в направлении линейной скорости, с которой вращается диск. Следовательно, лезвие падает по направлению вращения диска.

Задача № I.5.5 (3 балла). Представьте себе, что радиус Земли уменьшился на 1%, а масса ее осталась прежней. Увеличится ли, останется прежней или уменьшится энергия вращения Земли? Если изменится, то на сколько процентов?

Решение:

Поскольку должен выполняться закон сохранения момента импульса, мы имеем

Решение задач по физике

Так как момент инерции I пропорционален Решение задач по физике, уравнение (1) принимает вид

Решение задач по физике

или
Решение задач по физике

Следовательно, сокращение радиуса Земли на 1 % приведет к увеличению Решение задач по физике на 2%. Поскольку энергия вращения Земли пропорциональна Решение задач по физике:

Решение задач по физике

то она также увеличится на 2%.

Задача № I.5.6 (3 балла). Однородный тонкий стержень массой М и длиной L висит на шарнире без трения. В нижней своей части, как показано на рисунке, он связан со стеной посредством пружины, обладающей жесткостью к. Чему равен период колебаний стержня?

Решение задач по физике

Решение:

Обозначим через Решение задач по физике угол отклонения стержня от вертикали. Уравнение движения стержня записывается следующим образом:

Решение задач по физике

При малых углах отклонения Решение задач по физике оно принимает вид

Решение задач по физике

После подстановки выражения для момента инерции стержня Решение задач по физике имеем

Решение задач по физике

Отсюда получаем частоту колебаний

Решение задач по физике
и период

Решение задач по физике

Задача № I.5.7 (3 балла). Два тонких стержня каждый массой М и длиной L подвешены на шарнирах и соединены внизу пружиной таким образом, что в состоянии покоя они находятся в вертикальном положении. Жесткость пружины равна k. Покажите стрелками характер колебаний, соответствующих двум нормальным модам. Какие частоты соответствуют этим нормальным модам?
Решение задач по физике

Решение:

Решение задач по физике

В случае малых отклонений уравнения движения стержней записываются в виде

Решение задач по физике

Решение задач по физике

Соответствующие частоты колебаний равны

Решение задач по физике

Задача № I.5.8 (3 балла). Каково (с точностью до одной значащей цифры) отношение масс Солнца и Земли?

Решение:

Отношение масс Солнца и Земли равно Решение задач по физике

Задача № I.5.9 (3 балла). Два человека одинакового веса держатся руками за концы каната, переброшенного через блок. Оба начинают подниматься по канату вверх. Один из них взбирается со скоростью (относительно каната), вдвое превышающей скорость другого. Трением в блоке можно пренебречь. Кто первым достигнет блока: а) более быстрый, б) менее быстрый, в) оба одновременно или г) ответ неоднозначен?

Решение:

в) Оба одновременно.

Задача № I.5.10 (3 балла). Пробка погружена в ведро с водой и удерживается в ней с помощью пружины, прикрепленной ко дну ведра. Ведро держит в руке человек, находящийся в лифте. Что произойдет с пружиной в начале спуска лифта, когда он движется ускоренно: а) пружина растянется, б) сожмется или в) eсли состояние не изменится?

Решение:

б) Пружина сожмется. Деформация пружины описывается выражением

Решение задач по физике

Когда лифт спускается с ускорением, то Решение задач по физике становится меньше и пружина сокращается.

Задача № I.5.11 (3 балла). Один из спутников запущен на круговую орбиту радиусом R. Другой — на орбиту радиусом 1,01 R. Будет ли период обращения второго спутника больше, равен или меньше периода обращения первого спутника? Если периоды обращения различаются, то на сколько процентов?

Решение:

Квадрат периода обращения спутника пропорционален кубу радиуса его орбиты. Следовательно, период обращения второго спутника на 1,5% больше первого.

Задача № I.5.12 (3 балла). Шарик радиусом R погружается в жидкость, коэффициент вязкости которой равен Решение задач по физике. Сила сопротивления, действующая на шарик со стороны жидкости, по закону Стокса равна Решение задач по физике. Одновременно в жидкость погружается другой шарик той же массы, но радиусом 2R. Чему равны отношения

а) их начальных ускорений Решение задач по физике,

б) их установившихся скоростей Решение задач по физике?

Решение:

а) В начальный момент времени t=0 имеем v=0; поэтому) Решение задач по физике

б) В случае двух шариков с одинаковыми массами установившаяся скорость обратно пропорциональна силе сопротивления среды. Следовательно,

Решение задач по физике

Задача № I.5.13 (3 балла). Частица движется в поле одной из сил, определяемых следующими выражениями:

1) Решение задач по физике и

2) Решение задач по физике

Решение задач по физике

а) Для какой из них можно ввести потенциал V(x, у, z)?

б) В поле какой силы сохраняется полная (кинетическая плюс потенциальная) энергия движущейся частицы?

Решение:

а) Обе силы удовлетворяют условиям

Решение задач по физике

Следовательно, для них можно ввести потенциалы

Решение задач по физике

и

Решение задач по физике

б) Полная энергия сохраняется в поле обеих сил.

Задача № I.5.14 (20 баллов). Шар радиусом r вращается вокруг своей оси с угловой скоростью Решение задач по физике. Ось вращения горизонтальна. Шар падает на ровную поверхность, некоторое время вращается на месте, а затем начинает катиться без скольжения. Коэффициент трения равен Решение задач по физике.

а) Чему равна конечная скорость центра масс шара?

б) Какое расстояние пройдет шар, прежде чем установится эта скорость?

в) Вычислите конечную скорость центра масс шара для случая большого Решение задач по физике, когда начальное скольжение полностью отсутствует.

Решение:

Запишем уравнение движения шара в направлении x:

Решение задач по физике

Вращение шара вокруг его оси описывается уравнением

Решение задач по физике

где Решение задач по физике — момент инерции шара.

Интегрируя уравнения (1) и (2), получаем

Решение задач по физике

Решение задач по физике

Пусть, начиная с момента Решение задач по физике шар катится без скольжения:

Решение задач по физике

Подставляя это условие в уравнения (3) и (4), получаем

Решение задач по физике

откуда

Решение задач по физике

а) Конечную скорость движения центра масс шара находим из уравнения (3), подставляя в него выражение (6) для Решение задач по физике

Решение задач по физике

б) Прежде чем установится эта скорость, шар пройдет растояние

Решение задач по физике

в) При движении без скольжения момент внешних сил равен нулю, поэтому должен выполняться закон сохранения момента импульса

Решение задач по физике

где Решение задач по физике а I — момент инерции шара относительно мгновенной оси: Решение задач по физике Таким образом,

Решение задач по физике

и конечная скорость движения центра масс шара равна

Решение задач по физике

что совпадает с полученной ранее скоростью [см. выражение (7)

Задача № I.5.15 (13 баллов). Мяч в форме эллипсоида вращения имеет главные центральные моменты инерции Решение задач по физике). При броске ему случайно сообщили вращение с угловой скоростью Решение задач по физике под углом Решение задач по физике к большой оси мяча.

а) Определите величину и направление момента импульса (относительно большой оси мяча).

б) Чему равна угловая скорость прецессии? (Влиянием сопротивления воздуха и силы тяжести пренебречь.)

Решение:

а) Момент импульса равен

Решение задач по физике

Здесь Решение задач по физике — направляющий вектор вдоль оси симметрии эллипсоида.

б) Частота прецессии определяется по формуле

Решение задач по физике

которая непосредственно следует из динамических уравнении Эйлера.

Задача № I.5.16 (14 баллов). Частица движется в поле центральной силы. Рассмотрите только радиальную часть движения.

а) Что представляет собой «эффективный» потенциал, описывающий радиальную часть движения частицы? (Нарисуйте графики этого потенциала и его составляющих.)

б) Какому условию удовлетворяет эффективный потенциал в случае кругового движения?

в) Вычислите угловую скорость кругового движения в центральном поле с потенциалом Решение задач по физике.

Решение:

а) Эффективный потенциал определяют следующим образом:

Решение задач по физике

На рисунке показана зависимость эффективного потенциала от r для конкретного случая, когда Решение задач по физике

Решение задач по физике

б) Круговое движение имеет место при равенстве полной энергии Е минимальному значению Решение задач по физике Радиус орбиты R можно найти из уравнения

Решение задач по физике

в) Если Решение задач по физике то

Решение задач по физике

Условие минимума Решение задач по физике записывается в виде

Решение задач по физике

откуда находим Решение задач по физикеСледовательно, угловая скорость кругового движения равна Решение задач по физике

Задача № I.6.1 (10 баллов). Тонкая прямоугольная пластина со сторонами а и 2а вращается вокруг оси, совпадающей с одной из ее диагоналей, с постоянной угловой скоростью Решение задач по физике.

а) Вычислите главные центральные моменты инерции.

б) Определите момент импульса и его направление.

в) Вычислите момент, действующий на ось вращения.

Решение:

Решение задач по физике
а) Моменты инерции относительно главных центральных осей Решение задач по физике вычисляются непосредственно:

Решение задач по физике

Решение задач по физике

Момент инерции Решение задач по физике для тонкой пластины равен сумме Решение задач по физике и Решение задач по физике:
Решение задач по физике
б) Пусть Решение задач по физике — единичные векторы вдоль осей х, у и z соответственно. Вектор момента импульса L (а также вектор угловой скорости Решение задач по физике) не совпадает ни с одной из главных осей, но располагается в плоскости х, у. Его величина и направление даются выражением

Решение задач по физике

Здесь Решение задач по физике заменены выражениями (1) и (2) соответственно. Отметим, что в этой формуле Решение задач по физике — угол между векторами Решение задач по физике

в) На ось вращения действует момент

Решение задач по физике

где Решение задач по физике —производная по времени вектора L в инерциальной системе отсчета. В общем случае

Решение задач по физике

Во вращающейся с телом системе отсчета первый член в правой части уравнения (6) обращается в нуль. Следовательно,

Решение задач по физике

Задача № I.6.2 (10 баллов). Космический корабль движется по круговой орбите радиусом Решение задач по физике. Ракетный двигатель мгновенно увеличивает линейную скорость корабля на 8%. Каково расстояние до апогея новой орбиты? Нарисуйте аккуратно новую стационарную орбиту. Постройте график одномерного эффективного потенциала и покажите, как он изменился после сообщения кораблю дополнительного импульса.

Решение:

Космический корабль движется по круговой орбите радиусом Решение задач по физике При этом центробежная сила инерции должна уравновешивать силу притяжения, действующую по закону обратных квадратов:

Решение задач по физике

Следовательно, момент импульса корабля

Решение задач по физике

Возрастание линейной скорости корабля на 8% вызывает увеличение момента импульса также на 8%. Таким образом, J =Решение задач по физике Дифференциальное уравнение движения корабля имеет вид

Решение задач по физике

где Решение задач по физике

Это уравнение имеет следующее решение:

Решение задач по физике

где А — постоянная. Используя начальное условие Решение задач по физике при Решение задач по физике = 0, можно определить А:

Решение задач по физике

Следовательно,

Решение задач по физике

Отсюда находим, что расстояние до апогея равно Решение задач по физике

Решение задач по физике

Пусть Решение задач по физике— скорость корабля и расстояние до него в апогее. В соответствии с законом сохранения момента импульса и энергии имеем

Решение задач по физике

Решение задач по физике

Из уравнений (7) и (8) находим

Решение задач по физике

Подставляя сюда выражение дляРешение задач по физике из равенства (1), т. е.

Решение задач по физике

получаем Решение задач по физике

Задача № I.6.3 (10 баллов). Однородный диск массой m и радиусом а вращается вокруг неподвижной оси. По внешней окружности диска проходит невесомая нить, прикрепленная с одной стороны к диску, а с другой — к пружине, противоположный конец которой закреплен неподвижно. По окружности радиусом а/2 проходит другая нить, которая через пружину связана с грузом, имеющим массу m. Напишите уравнения Лагранжа для диска и груза. (Уравнения не решайте.)
Решение задач по физике

Решение:

Пусть Решение задач по физике — первоначальные длины пружин 1 и 2, когда вся система находится в равновесии, и диск не вращается. При повороте диска на угол Решение задач по физике от положения равновесия пружина 1 растягивается на длину Решение задач по физике Если при этом растягивается и пружина 2 на длину Решение задач по физике то груз опускается на длину Решение задач по физике Кинетическая энергия системы Т равна сумме кинетической энергии диска Решение задач по физике и кинетической энергии груза Решение задач по физике

Решение задач по физике

Решение задач по физике

Потенциальная энергия системы
Решение задач по физике

Решение задач по физике

Составляем функцию Лагранжа

Решение задач по физике

Уравнения Лагранжа записываются в видеРешение задач по физике

Подставляя выражение для лагранжиана (4) в эти уравнения, мы непосредственно получаем два уравнения движения.

Задача № I.6.4 (10 баллов). Груз массой m прикреплен к пружине, а пружина — к точке подвеса. Под действием внешней силы точка подвеса совершает вертикальное движение по закону х = Решение задач по физике. Какова амплитуда стационарных колебаний груза в вязкой среде, если ее сила сопротивления равна —bх? 

Решение задач по физике

Решение:

Обозначим через х смещение груза от положения равновесия в момент времени t. При этом пружина растягивается на длину Решение задач по физике Уравнение движения груза записывается в виде

Решение задач по физике

Оно имеет частное (стационарное) решение

Решение задач по физике

где Решение задач по физике

Амплитуда стационарного движения, установившегося по прошествии достаточно длительного времени, равна

Решение задач по физике

Задача № I.6.5 (10 баллов). Сплошной однородный цилиндр радиусом а катится без скольжения по внутренней поверхности неподвижного цилиндра большего радиуса R. 
а) Выпишите лагранжиан для этой системы. 
б) Найдите уравнение движения. 
в) Определите частоту малых колебаний цилиндра около положения устойчивого равновесия. 

Решение задач по физике

Решение:

Решение задач по физике

а) Кинетическую энергию цилиндра можно представить в виде суммы энергии поступательного движения центра масс цилиндра и энергии вращения его относительно центра масс. Обозначим через Решение задач по физике угловую скорость вращения цилиндра. Кинетическую энергию цилиндра можно записать в виде

Решение задач по физике

где для сплошного цилиндра Решение задач по физике При движении без скольжения Решение задач по физике

Следовательно, выражение (1) принимает вид

Решение задач по физике

Потенциальная энергия цилиндра равна

Решение задач по физике

Составляем функцию Лагранжа

Решение задач по физике

б) Подставляя выражение (4) в уравнение Лагранжа, получаем уравнение движения

Решение задач по физике

в) В случае малых колебаний Решение задач по физике и уравнение (5) записывается в виде

Решение задач по физике

откуда получаем

Решение задач по физике

Задача № I.6.6 (10 баллов). 
а) Определите радиус сходимости степенного ряда 
Решение задач по физике
б) Используя метод вычетов, докажите следующее тождество: 
Решение задач по физике
где а и с — действительные величины, причем а > 0. 

Решение:

а) Чтобы определить значения х, при которых заданный ряд сходится, можно использовать следующий признак сходимости: если для любого бесконечного ряда модуль отношения n-го члена ряда к (n - 1)-му члену имеет предел А < 1, то такой ряд сходится. Мы имеем

Решение задач по физике

Следовательно, при Решение задач по физике ряд сходится абсолютно. В случае Решение задач по физике знаки членов ряда чередуются, члены ряда монотонно убывают по величине и п-й член в пределе стремится к нулю. Поэтому при Решение задач по физике ряд сходится ). В случае х = е применим асимптотическую формулу Стирлинга

Решение задач по физике

Таким образом, из выражения (1) имеем

Решение задач по физике

и ряд при х=е, согласно признаку Раабе, расходится.

б) Преобразуем заданный нам интеграл следующим образом:

Решение задач по физике

Здесь интегрирование выполняется в верхней полуплоскости, если m > 0, и соответственно в нижней полуплоскости, если m < 0. Следовательно, при m > 0

Решение задач по физике

При m < 0 имеем 

Решение задач по физике

Задача № I.6.7 (10 баллов). Спутник движется по сильно вытянутой эллиптической орбите в плоскости земного экватора. Нарисуйте эту орбиту, отметьте положение Земли и опишите влияние небольшого сопротивления атмосферы на траекторию спутника. Рассмотрите особо любые изменения, которые могут возникнуть у эксцентриситета орбиты е, периода обращения спутника Т, большой полуоси орбиты а и расстояния кратчайшего сближения с Землей Решение задач по физике.

Решение:

Решение задач по физике
Как известно из первого закона Кеплера, Земля должна находиться в одном из фокусов эллиптической орбиты.

Тормозящее действие атмосферы проявляется при движении спутника только вблизи Земли. Пусть в результате этого торможения спутник теряет за один оборот импульс Решение задач по физике. При этом момент импульса спутника уменьшается на

Решение задач по физике

где Решение задач по физике — показанное на рисунке расстояние от центра Земли до перигея орбиты. Эксцентриситет орбиты е связан с моментом импульса j соотношением

Решение задач по физике

где Е— полная энергия системы, k — постоянная в выражении для потенциальной энергии V = —mk/r. В случае эллиптической орбиты полная энергия Е отрицательна и 0<е< 1. Дифференцируя соотношение (2), затем находим

Решение задач по физике

здесь Решение задач по физике — энергия, теряемая спутником за один оборот. Используя соотношения

Решение задач по физике

получаем

Решение задач по физике

В случае эллиптической орбиты

Решение задач по физике

Поскольку знак Решение задач по физике отрицателен, из (4) и (5) следует

Решение задач по физике

Из этой формулы мы видим, что эксцентриситет орбиты уменьшается. С каждым оборотом спутник становится ближе к Земле. Его орбита все больше и больше приближается к круговой.

Большая полуось орбиты а пропорциональна 1 /W, где W — энергия связи: W = —Е. Поскольку спутник теряет энергию, то W возрастает. Следовательно, а уменьшается. Уменьшается также и период его обращения Т, так как в соответствии с третьим законом Кеплера квадрат периода пропорционален кубу большой полуоси.

Спутник теряет свою энергию на торможение лишь при сближении с Землей. Поэтому подавляющую часть периода обращения он движется вне атмосферы, а остальную, меньшую, часть — в ней. С каждым оборотом полярный угол, соответствующий наибольшему сближению с Землей, увеличивается. При этом орбита спутника прецессирует в направлении, показанном на рисунке. Кратчайшее расстояние между спутником и Землей Решение задач по физике можно найти, пользуясь законами сохранения момента импульса и энергии:
Решение задач по физике

здесь, поскольку Е < О, знак корня во втором выражении изменен на противоположный. Следовательно, по мере уменьшения J расстояние Решение задач по физике сокращается.

Задача № 1.6.8 (10 баллов). В трубу А (см. рисунок) мощным насосом нагнетают воду. Течение ламинарное. В трубе В скорость течения равна 20 см/с. Сечение трубы В равно Решение задач по физике, сечение трубки С составляет Решение задач по физике. Определите разность уровней h ртутного манометра.

Решение задач по физике

Решение:

Обе трубы горизонтальны; следовательно, каких-либо изменений потенциальной энергии жидкости нет. В сечении С кинетическая энергия единичного объема жидкости равна Решение задач по физике а в сечении Решение задач по физике Дано, что Решение задач по физике = 20 см/с. Скорость vc можно найти из закона сохранения массы

Решение задач по физике

Разность давлений в сечениях В и С находим из закона сохранения энергии для 1 Решение задач по физике воды:

Решение задач по физике

она должна быть равна разности давлений в трубках манометра, подсоединенных к В и С, которая определяется по высоте водяного столба (р = 1) между уровнями):

Решение задач по физике

Следовательно,

Решение задач по физике

Задача № I.6.9 (5 баллов). Три материальные точки расположены в вершинах прямоугольного треугольника, как показано на рисунке. Найдите главные моменты инерции относительно осей, проходящих через точку Р, расположенную в середине гипотенузы. Укажите ориентацию главных осей относительно одной из сторон треугольника.
Решение задач по физике

Решение:

Поскольку рассматриваемые материальные точки расположены в одной плоскости, то ось, перпендикулярная этой плоскости в точке Р, является одной из главных осей инерции.

Решение задач по физике

Назовем ее осью z. Тогда момент инерции относительно этой оси записывается в виде

Решение задач по физике

Очевидно, второй главной осью инерции должна быть гипотенуза треугольника, так как вне ее расположена только одна материальная точка. Следовательно,

Решение задач по физике

Используя соотношение Решение задач по физике находим

Решение задач по физике

Задача № I.6.10 (10 баллов). Ребенок, едущий в поезде, держит за ниточку длиной 1 м шар, наполненный гелием. Поезд мчится со скоростью 100 км/час по искривленному дугой пути радиусом 3 км.

а) Каков угол отклонения нити от вертикали?

б) На приведенном внизу рисунке отметьте направление проекции нити в горизонтальной плоскости.
Решение задач по физике

Решение:

а) Плотность гелия составляет примерно Решение задач по физике плотности воздуха. Поскольку шар находится в воздухе, простой путь решения задачи состоит в следующем: считать плотность шара равной —Решение задач по физике и не обращать в дальнейшем внимания на присутствие воздуха. Действующее ускорение «свободного падения» шара равно в этом случае —Решение задач по физике, а центробежное ускорение —Решение задач по физике, причем

Решение задач по физике

Следовательно,

Решение задач по физике

или

Решение задач по физике

б) К центру кривизны пути.

Задача № I.6.11 (5 баллов). Человек, изображенный на рисунке, весит 70 кг. Он сидит на перекладине и медленно поднимает себя, подтягивая веревку, переброшенную через блок. С какой силой он должен тянуть веревку?
Решение задач по физике

Решение:

Пусть Т—натяжение веревки. В состоянии равновесия 2Т = W = 70 кг. Отсюда Т = 35 кг.

Задача № I.7.1 (20 баллов). Представьте себе, что вы ритмично хлопаете в ладони, причем промежуток времени между двумя последовательными хлопками равен Решение задач по физике. Каждый хлопок дает звук в течение весьма короткого (по сравнению с Решение задач по физике) интервала времени Решение задач по физике. В течение интервала времени At звуковое давление около вашего уха можно считать постоянным. В остальное время оно равно нулю.

а) Покажите качественно, что дает фурье-преобразование звука хлопка. (Для этого не нужно выполнять непосредственно разложение в ряд Фурье.)

б) Представьте себе, что вместо ритмично повторяющихся хлопков имеется только один хлопок длительностью Решение задач по физике. Как будет выглядеть качественно фурье-преобразование в этом случае? Сравните его с предыдущим результатом.

В обоих случаях (а и б) требуется показать с помощью графиков «спектр» Фурье, т. е. показать зависимость интенсивности от частоты с соответствующими масштабными соотношениями, поясняющими качественно форму спектра.

Решение:

Разложение функции f(t) в ряд Фурье записывается в виде

Решение задач по физике

а) Прежде всего отметим, что в нашем случае f(t) —четная функция времени t; следовательно, все Решение задач по физике должны обращаться в нуль. Кроме того, заданный сигнал является периодическим (с периодом Решение задач по физике). Таким образом, мы имеем следующее условие:

Решение задач по физике

Из (1) и (2) находим
Решение задач по физике
и разложение (1) принимает видРешение задач по физике

Нетрудно заметить, что сумма большого числа фурье-компонент с приблизительно одинаковыми амплитудами обращается в нуль, за исключением тех промежутков времени, где многие из этих компонент оказываются в фазе. В соответствии с заданной волновой картиной фурье-компоненты должны находиться в фазе лишь в течение тех промежутков времени, когда длятся хлопки, т. е. в промежутках

Решение задач по физике

должно выполняться условие

Решение задач по физике

Оно справедливо при

Решение задач по физике

Таким образом, приходим к заключению, что

Решение задач по физике

Чтобы остальные компоненты не вызывали существенного искажения исходной волновой картины, они должны быть достаточно малыми:

Решение задач по физике

В результате получаем следующее распределение:
Решение задач по физике

б) Пусть Решение задач по физике — временной интервал, внутри которого можно услышать звук одиночного хлопка. Этот интервал может быть сколь угодно большим, но обязательно конечным. Если допустить, что в случае Решение задач по физике интервал Решение задач по физике между двумя последовательными хлопками превышает Решение задач по физике, то к решению данной задачи можно применить результат, полученный выше. Однако в том случае, когда Решение задач по физике произвольно велико, частота Решение задач по физике определяемая формулой (3), становится весьма малой. Отсюда следует, что спектр одиночного хлопка является непрерывным.

К такому же выводу можно прийти, выполняя преобразование Фурье, т. е. записывая функцию f(t) в виде

Решение задач по физике

причем

Решение задач по физике

Таким образом, мы и здесь получаем непрерывный спектр Решение задач по физике Его поведение отражено выше на рисунке.

Задача № I.7.2 (10 баллов). Даны унитарная матрица S и матрица Т, связанные друг с другом соотношением Решение задач по физике. Покажите, что

а) в предельном случае, когда Решение задач по физике матрица Т является эрмитовой и

б) для любой матрицы Т справедливо следующее равенство:

Решение задач по физике

где Решение задач по физике — произвольный вектор состояния в пространстве, в котором действуют операторы S и Т, а Решение задач по физике представляет полный набор таких векторов состояния.

Решение:

а) S — унитарная матрица, следовательно,

Решение задач по физике

Подставляя сюда заданное в условии задачи соотношение

Решение задач по физике

получаем

Решение задач по физике

или

Решение задач по физике

При Решение задач по физике членом Т+Т можно пренебречь. Тогда выражение (2) принимает вид

Решение задач по физике

т. е. Т — эрмитова матрица.

б) Из выражения (2) имеем

Решение задач по физике

Для упругих переходов Решение задач по физике равенство (3) можно переписать в виде

Решение задач по физике

где

Решение задач по физике

поскольку Решение задач по физике — полный набор промежуточных состояний.

Так как Решение задач по физике, то приведенное выше равенство принимает вид

Решение задач по физике

Используя соотношение

Решение задач по физике

получаем окончательно

Решение задач по физике

Задача № I.7.3 (15 баллов). В кузове грузовика лежит деревянный ящик, частично загруженный кирпичом. Дно ящика — квадрат со стороной 1 м, масса ящика по сравнению с массой кирпича пренебрежимо мала. Известно, что ящик начинает скользить, если грузовик трогается с ускорением, большим чем Решение задач по физике. До какой высоты можно уложить кирпичи в ящике без опасения, что он опрокинется при таком ускорении?

Решение:

Решение задач по физике
Предельная высота кладки h соответствует случаю, когда результирующая сила R направлена в сторону нижнего ребра ящика: при большей высоте кладки ящик опрокидывается. Для моментов сил имеем

Решение задач по физике

откуда Решение задач по физике

Задача № I.7.4 (15 баллов). Ящик с точными приборами поставлен на амортизирующую прокладку для защиты от вертикальных вибраций.

а) Под нагрузкой прокладка сдавливается на 6 см. Чему равен период собственных колебаний ящика?

б) Пол вибрирует с частотой 20 Гц. Каково отношение амплитуд колебаний ящика и пола?

Решение:

а) В состоянии покоя вертикальная составляющая сил должна быть равна нулю:

Решение задач по физике

Нам дано х = —6 см. Следовательно,

Решение задач по физике

При возникновении собственных колебаний их период равен

Решение задач по физике

б) Если пол вибрирует но закону Решение задач по физике то уравнение движения записывается в виде

Решение задач по физике

здесь мы приняли, что состоянию равновесия соответствует значение координаты х = 0. Перепишем уравнение (2), подставляя вместо k его выражение (1):

Решение задач по физике

Это уравнение имеет стационарное решение

Решение задач по физике

Следовательно, искомое отношение амплитуд при Решение задач по физике равно

Решение задач по физике

Задача № I.7.5 (20 баллов). Капитан небольшого судна, попавшего в экваториальную штилевую полосу, решил прибегнуть к хитрости, а именно поднять якорь массой 200 кг на верх двадцатиметровой мачты. Масса остальной части судна равна 1000 кг. (Радиус Земли 6400 км.) Судно придет в движение.

а) Почему?

б) В каком направлении?

в) С какой скоростью?

г) Откуда берется энергия, приводящая судно в движение?

Решение:

а) При подъеме якоря возникает кориолисова сила

Решение задач по физике

действующая на судно и вызывающая его движение.

б) Судно станет относить на запад.

в) Пусть v — скорость, с которой поднимают якорь на мачту. Тогда судно приобретает ускорение

Решение задач по физике

где m — масса якоря, М — масса судна. Высота мачты S и время Т поднятия якоря связаны со скоростью v соотношением

Решение задач по физике

Конечная скорость судна равна

Решение задач по физике

Здесь Решение задач по физике —- угловая скорость вращения Земли Решение задач по физике

Решение задач по физикеРешение задач по физике

Другое решение.

в) Воспользуемся свойством сохранения момента импульса судна и якоря относительно центра масс Земли в инерциальной системе:

Решение задач по физике

Отсюда

Решение задач по физике

и для скорости судна относительно воды получаем

Решение задач по физике

г) Судно фактически теряет кинетическую энергию за счет работы, совершаемой против центростремительной силы.

Задача № I.7.6 (20 баллов). Кольцо массой Решение задач по физике может скользить по стержню массой Решение задач по физике и длиной L, который на одном конце прикреплен к неподвижному шарниру и висит вертикально. Кольцо связано с шарниром через невесомую пружину с жесткостью k так, что при равновесии центры масс кольца и стержня совпадают. Движение происходит в вертикальной плоскости под действием силы тяжести.
Решение задач по физике

а) Составьте лагранжиан системы в координатах Решение задач по физике и R, показанных на рисунке.

б) Получите дифференциальные уравнения движения в координатах Решение задач по физике и R.

в) Найдите одну из нормальных мод и соответствующую ей частоту колебаний (в предположении, что амплитуда колебаний мала).

Решение:

а) Пусть Решение задач по физике — длина пружины в свободном состоянии. Условие равновесия системы записывается в виде

Решение задач по физике

Следовательно,

Решение задач по физике

При растяжении пружины до длины R она запасает потенциальную энергию

Решение задач по физике

Потенциальная энергия стержня и кольца в поле силы тяжести

Решение задач по физике

При Решение задач по физике эта потенциальная энергия принимается равной нулю.

Кинетическая энергия системы равна сумме кинетических энергий вращательного и поступательного движения стержня и кольца:

Решение задач по физике

Лагранжиан системы записывается в виде

Решение задач по физике

б) Подставляя выражение (2) в уравнения Лагранжа

Решение задач по физике

получаем уравнения движения

Решение задач по физике

Решение задач по физике

в) В случае малых колебаний Решение задач по физике Поэтому в уравнении (3) можно положить Решение задач по физике а в (4) Решение задач по физике Таким образом, уравнения (3) и (4) принимают вид

Решение задач по физике

Решение задач по физике

Из последнего уравнения сразу находим частоту одной из нормальных мод:

Решение задач по физике

Задача № I.8.1 (10 баллов). Покажите, что любая действительная и всюду аналитическая функция должна быть постоянной.

Решение:

Действительную и аналитическую функцию можно разложить в степенной ряд:

Решение задач по физике

Отсюда следует, что при Решение задач по физике все члены этого ряда, за исключением первого, неограниченно возрастают. Таким образом, если f(x)—аналитическая функция при Решение задач по физике, то все коэффициенты ап при возрастающих членах должны быть равны нулю. Поэтому

Решение задач по физике

Задача № I.8.2 (10 баллов). Даны вектор V и тензор Т:

Решение задач по физике

на плоскости Решение задач по физике в декартовой системе координат. Вычислите компоненты вектора и тензора в новой системе координат, полученной путем поворота старой системы на +90° вокруг оси Решение задач по физике.

Решение:

Матрица преобразования при повороте системы координат записывается в виде

Решение задач по физике

Обозначим через Решение задач по физике вектор V и тензор Т соответственно в новой системе координат. Тогда

Решение задач по физике

Задача № I.8.3 (20 баллов). Докажите, что функция Решение задач по физике удовлетворяет дифференциальному уравнению

Решение задач по физике

Решение:

Если радиус-вектор г отличен от нуля, то левую часть дифференциального уравнения Решение задач по физике можно записать в сферических координатах Решение задач по физике Мы имеем

Решение задач по физике

где L — оператор, зависящий от Решение задач по физике Поскольку Решение задач по физике то Решение задач по физике. Следовательно, при Решение задач по физике

Решение задач по физике

Пусть теперь Решение задач по физике вычислим интеграл

Решение задач по физике

по объему, ограниченному сферой радиусом а с центром в точке r = 0. В результате прямых вычислений и применения формулы для дивергенции имеем

Решение задач по физике

В предельном случае, когда Решение задач по физике

Решение задач по физике

Учитывая совместно (1) и (2), получаем

Решение задач по физике

Задача № I.8.4 (20 баллов). Составьте лагранжиан и найдите уравнение движения для системы, состоящей из материальной точки массой т, расположенной на верхнем конце невесомого стержня длиной L. Нижний конец стержня прикреплен к шарниру (трение в шарнире не учитывать). Шарнир совершает вертикальные колебания по гармоническому закону

Решение задач по физике

Единственной степени свободы отвечает координата 0 между стержнем и вертикалью. (Такую систему называют обращенным маятником.)
Решение задач по физике

Решение:

Координаты х и у материальной точки зависят от L, h(t) и Решение задач по физике:

Решение задач по физике

Дифференцируя выражения (1) no t, получаем

Решение задач по физике

Кинетическая энергия материальной точки записывается в виде

Решение задач по физике

Потенциальная энергия

Решение задач по физике

Составляем лагранжиан системы:

Решение задач по физике

Подставляя его в уравнение Лагранжа

Решение задач по физике

находим уравнение движения

Решение задач по физике

или

Решение задач по физике

Рассмотрим случай малых колебаний. Перейдем к переменной Решение задач по физике тогда можно написать

Решение задач по физике

и последнее уравнение движения принимает вид

Решение задач по физике

Если шарнир закреплен неподвижно, т. е. Решение задач по физике = 0, то полученное уравнение совпадает с уравнением движения математического маятника.

Задача № I.8.5 (20 баллов). В аэропорту, расположенном на широте экватора, имеются трое одинаковых маятниковых часов. Часы А оставляют в аэропорту, часы В помещают в самолет, отлетающий на восток, а часы 3 — в самолет, отлетающий на запад. Однажды точно в полдень, когда все часы показывают одно и то же время, самолеты взлетают. Самолет .В облетает земной шар по экватору в восточном направлении с постоянной путевой скоростью. Самолет 3 облетает земной шар в западном направлении с той же путевой скоростью. На следующие сутки оба самолета прибывают в аэропорт одновременно, поскольку путевые скорости у них одинаковы. Часы А, оставленные в аэропорту, показывают в момент прибытия самолетов точно полдень, т. е. между взлетом и посадкой обоих самолетов прошло 24 ч, 0 мин и 0,00000000 с.

а) Будут ли эти трое часов по-прежнему показывать одно и то же время? Если нет, то перечислите физические причины, которые, по вашему мнению, могли бы вызвать это расхождение.

б) Примите во внимание наиболее существенную причину и определите показания часов В и 3 в тот момент, когда часы А в аэропорту показывают полдень (после 24-часового полета). Радиус Земли можно считать равным 6000 км.

Решение:

а) Нет, часы будут показывать разное время благодаря действию следующих эффектов:

1) ускорения Кориолиса — Решение задач по физике

2) ослабления гравитационного притяжения с ростом высоты.

3) центростремительного ускорения — Решение задач по физике

4) сил трения между воздухом и маятником,

5) релятивистского эффекта Решение задач по физике

6) ускорения при взлете и посадке,

7) ускорения, связанного с орбитальным движением Земли, и т. д.

б) В качестве системы отсчета выберем инерциальную систему с началом координат в центре Земли. В этой системе кориолисова сила отсутствует. Действующее ускорение свободного падения Решение задач по физике для объекта, движущегося вблизи земной поверхности с угловой скоростью и, дается выражением

Решение задач по физике

т. е.

Решение задач по физике

где Решение задач по физике

Период колебаний математического маятника равен, как известно, Решение задач по физике где L — длина маятника, которую мы считаем неизменной. Угловая скорость самолета 3, летящего на запад, в инерциальной системе равна нулю. Поэтому период колебаний маятника в нем

Решение задач по физике

здесь Решение задач по физике —период колебаний часов, оставленных в аэропорту. Угловая скорость самолета В, летящего на восток, вдвое превышает угловую скорость вращения Земли. Следовательно,

Решение задач по физике

Учитывая, что Решение задач по физике, периоды колебаний обоих маятниковых часов в летящих самолетах можно приближенно представить следующими выражениями:

Решение задач по физике

и

Решение задач по физике

Подставляя сюда Решение задач по физике = 24 ч и учитывая, что Решение задач по физике получаем

Решение задач по физике

Поскольку Решение задач по физике часы В идут медленнее часов А, а поскольку Решение задач по физике часы Решение задач по физике идут быстрее. По прибытии самолетов в аэропорт часы В отстанут на 420 с, а часы 3 уйдут вперед на 140 с относительно часов А, оставленных в аэропорту.

Задача № I.8.6 (20 баллов). Согласно ньютоновской теории тяготения, гравитационный потенциал Солнца определяется (в предположении сферической симметрии) формулой

Решение задач по физике

где Решение задач по физике. СГС (гравитационная постоянная), GM = Решение задач по физикеед. СГС, М — масса Солнца, r — расстояние от его центра.

а) Получите формулу для силы, действующей на частицу массой m, удаленную от Солнца на расстояние r.

б) По Эйнштейну, ньютоновская теория требует уточнения. Если ввести поправку первого порядка, то гравитационный потенциал можно записать в виде

Решение задач по физике

Исходя из соображений размерностей, попытайтесь найти выражение для поправочного коэффициента А. Он должен зависеть от скорости света, а также от G и М. Хорошим считается только решение, полученное с точностью до множителя 2.

в) Оцените относительную величину поправки (к ньютоновскому потенциалу), приняв расстояние от Солнца до Земли равным r = Решение задач по физике см.

Решение:

а) Поскольку Солнце представляет собой однородный шар, то сила равна

Решение задач по физике

б) Согласно общей теории относительности, гравитационный потенциал записывается в виде

Решение задач по физике

где А может зависеть только от с, G и М.

Пусть. D(A) — размерность параметра А. Из (2) приходим к следующему соотношению:

Решение задач по физике

Используя соотношение

Решение задач по физике

получаем
Решение задач по физике
Из (3) и (4) имеем

Решение задач по физике

Следовательно,

Решение задач по физике

в) Отношение второго члена к первому в формуле (2) равно

Решение задач по физике

Для Земли эта величина очень мала. Только для Меркурия эта поправка является ощутимой.

Динамика

Задача № 25. На одно из оснований цилиндра (длина Решение задач по физике и площадь основания Решение задач по физике) действует постоянная сила Решение задач по физике перпендикулярная основанию (рис. 14). Какая сила действует на противоположное основание цилиндра? Какая сила действует на некоторое сечение цилиндра Решение задач по физике параллельное основанию? Движение тела под действием силы Решение задач по физике происходит в среде без сопротивления. 

Решение:
Сила, действующая на сечение Решение задач по физике должна быть такой, чтобы в случае, если мы мысленно разрежем цилиндр по Решение задач по физикеоставшаяся часть цилиндра Решение задач по физике продолжала двигаться с тем же ускорением, с каким двигался бы весь цилиндр Решение задач по физике под действием силы Решение задач по физике
Следовательно, 
Решение задач по физике
где Решение задач по физике — сила, действующая на сечение Решение задач по физике — масса всего тела; Решение задач по физике — масса части тела Решение задач по физике 
Если цилиндр однородный, то 
Решение задач по физике
и если 

Решение задач по физике
то 
Решение задач по физике

Решение задач по физике

Рис.14.


Сила, приложенная к основанию цилиндра Решение задач по физике равна нулю. 

Задача № 26. Лыжник съезжает с вершины горы высотой Решение задач по физике Внизу имеется впадина, представляющая часть дуги окружности радиусом Решение задач по физике Найти (пренебрегая трением) кажущееся относительное увеличение веса своего тела, ощущаемое лыжником в нижней точке спуска с горы. 
Дано: 
Решение задач по физике

Решение:
Когда лыжник спускается с вершины горы Решение задач по физике  и достигает нижней точки Решение задач по физике (рис. 15), то полная сила будет складываться из силы тяжести Решение задач по физике и центробежной силы Решение задач по физике действующей на 
лыжника в результате движения его по окружности, причем обе силы по направлению совпадут, так что 

Решение задач по физике

т. е. в 3 раза больше собственного веса лыжника. 

Решение задач по физике

Рис.15.

Задача № 27. Пуля массой Решение задач по физике летящая с горизонтальной скоростьюРешение задач по физике попадает в предмет, подвешенный на нити, и застревает в нем (рис. 16). Определить угол Решение задач по физике на который отклонится предмет, если его масса Решение задач по физике и длина нити Решение задач по физике
Решение задач по физике

Рис.16.


Дано: 
Решение задач по физике 

Решение:
Для определения угла Решение задач по физике на который отклонится предмет после попадания в него пули, необходимо найти высоту Решение задач по физике на которую он поднимется: 
Решение задач по физике
где Решение задач по физике — скорость предмета и пули после соударения. Из рис. 16 следует, что 
Решение задач по физике
Приравнивая правые части двух последних выражений, получим 
Решение задач по физике
Из закона сохранения количества движенияРешение задач по физике и определим скорость предмета и пули после соударения: 
Решение задач по физике

Подставим в формулу для Решение задач по физике вместо Решение задач по физике его значение: 
Решение задач по физике

Сила, импульс, законы Ньютона

Задача № 28. Мотоциклист движется по горизонтальному участку пути со скоростью Решение задач по физике В определенное время он начинает  тормозить. Через 5 сек после начала торможения мотоцикл останавливается. Каков коэффициент трения колес мотоцикла о полотно дороги в момент торможения? 
Дано; 
Решение задач по физике

Решение:
Согласно второму закону Ньютона, 

Решение задач по физике
где Решение задач по физике — конечная скорость мотоцикла. 
Мотоцикл останавливает сила трения 

Решение задач по физике
где Решение задач по физике — коэффициент трения. Таким образом, 
Решение задач по физике
Перед импульсом ставится знак минус потому, что сила трения направлена в противоположную сторону начальной скорости. 
Коэффициент трения 
Решение задач по физике

Задача № 29. На абсолютно гладкой горизонтальной плоскости лежит Решение задач по физике связанных нитью равных грузов весом Решение задач по физике каждый. Такой же Решение задач по физикеРешение задач по физике-й груз, подвешенный вертикально, прикреплен к ним нитью, перекинутой через неподвижный блок. Определить ускорение, с  которым движется система, а также натяжение нити между Решение задач по физике-м и Решение задач по физике-ы грузом. Сколько грузов надо взять, чтобы наибольшее натяжение было равно Решение задач по физике Трение в блоке не учитывать. 
Дать численный ответ на первые два вопроса задачи, если Решение задач по физике

Решение:
Движущей силой системы связанных нитью грузов является вес вертикально подвешенного груза. Ускорение движения системы будет во столько раз меньше ускорения свободного падения тел, во сколько раз масса одного груза меньше массы всей системы грузов, т. е. 
Решение задач по физике 
откуда 

Решение задач по физике
Силу натяжения нити между Решение задач по физике-м грузом определяем по основному уравнению динамики 
Решение задач по физике
где Решение задач по физике — вес одного груза. 
Наибольшая сила натяжения нити будет между Решение задач по физике-м грузом, и по предыдущему выражению она равна 

Решение задач по физике
Численные ответы при Решение задач по физике следующие: 
Решение задач по физике 

Задача № 30. Какую силу тяги должен развить паровоз, чтобы поезд массой Решение задач по физике через Решение задач по физике после начала движения по горизонтальному пути приобрел скорость Решение задач по физике если коэффициент сопротивления во время движения равен Решение задач по физике При решении задачи 
считать, что поезд в течение указанного времени двигался  равноускоренно. 
Дано: 
Решение задач по физике

Решение:
Общая сила тяги Решение задач по физике паровоза будет, очевидно, равна 
Решение задач по физике
где Решение задач по физике — часть силы тяги паровоза, сообщающая ускорение поезду, а Решение задач по физике—-часть силы тяга, идущая на преодоление силы сопротивления движению поезда. 
Следовательно, для определения общей силы тяги паровоза  необходимо определить Решение задач по физике Решение задач по физике Сила Решение задач по физике согласно второму закону Ньютона, 
Решение задач по физике 
где Решение задач по физике — масса поезда; Решение задач по физике — ускорение, сообщаемое поезду силой Решение задач по физике
Так как поезд двигался равноускоренно и через Решение задач по физике секунд после начала движения приобрел скорость Решение задач по физике то ускорение 
Решение задач по физике
Подставляя значение Решение задач по физике в уравнение Решение задач по физике получим 
Решение задач по физике
Сила тяги поезда, идущая на преодоление сопротивления, равна 
Решение задач по физике
Общая сила тяги паровоза 
Решение задач по физике
Подставив численные значения, получим 
Решение задач по физике

Задача № 31. Тело массой Решение задач по физике движется вверх по вертикальной стене под действием силы Решение задач по физикенаправленной под углом Решение задач по физике к вертикали (рис. 17). Определить, с каким ускорением движется тело, если коэффициент трения тела о стену равен Решение задач по физике 

Решение задач по физике

Рис.17.

Решение:
Разложим силу Решение задач по физике на вертикальную и горизонтальную  составляющие— Решение задач по физике В горизонтальном направлении па тело будут действовать две равные и противоположно направленные силы; Решение задач по физике — составляющая силы Решение задач по физике — сила, с которой стена действует 
па тело. Сила, которая сообщает телу ускорение, есть равнодействующая сил Решение задач по физике и силы трения Решение задач по физике Тогда

 Решение задач по физике

Решение задач по физике

Подставляя значения Решение задач по физике получим 
Решение задач по физике
откуда 
Решение задач по физике
Исследуем результат. 
Решение задач по физикеВ этом случае трение отсутствует, так как тело не давит 
на стену. 

Решение задач по физике В этом случае движение возможно только вниз, так 
как вертикальная составляющая Решение задач по физике при этом сила трения Решение задач по физике будет направлена в  
сторону, противоположную скольжению, т. е. изменит знак, и выражение для ускорения следует записать так: 
Решение задач по физике
Знак минус перед ускорением означает, что оно направлено вниз. Если еще при этом Решение задач по физике т. е. тело будет находиться и покое пли равномерно скользить вниз. 
Решение задач по физике
В этом случае тело будет двигаться вниз под действием двух сил: силы тяжести и данной силы Решение задач по физике а ускорение его по абсолютной величине будет больше Решение задач по физике

Задача № 32. Вагон под действием толчка, сообщенного ему паровозом, поднимался вверх по уклону в течение Решение задач по физике и до остановки прошел путь, равный Решение задач по физике После остановки вагон начал опускаться вниз по уклону и тот же путь прошел за Решение задач по физике Пользуясь этими данными, определить коэффициент трения Решение задач по физике При решении задачи считать постоянными Решение задач по физике и угол уклона Решение задач по физике (рис. 18)* 

Решение задач по физике

Рис.18.

Дано:

Решение задач по физике

Решение:
При спуске вагона по уклону и при его подъеме действовали скатывающая сила  
Решение задач по физике
и сила трения 
Решение задач по физике
где Решение задач по физике — масса вагона; Решение задач по физике — ускорение силы тяжести; Решение задач по физике — угол уклона. 
Эти постоянные силы при спуске вагона направлены в противоположные стороны. Поэтому равнодействующая данных сил равна их разности, направлена параллельно уклону вниз и во время движения ввиду неизменности Решение задач по физике и Решение задач по физике будет постоянной. Под действием этой силы вагон при спуске будет двигаться равноускоренно. Ускорение вагона, сообщаемое ему действием указанной равнодействующей, будет равно Решение задач по физике Тогда, согласно второму закону Ньютона, можно написать 
Решение задач по физике
откуда 
Решение задач по физике
и 
Решение задач по физике
Для определения Решение задач по физике необходимо знать угол уклона Решение задач по физике и ускорение Решение задач по физике Найдем угол уклона Решение задач по физике При движении вагона вверх по уклону, как и при спуске, на него действовали те же силы Решение задач по физике 
и Решение задач по физике Но в данном случае эти силы направлены в одну сторону — в сторону, противоположную движению вагона. Поэтому равнодействующая этих сил равна их сумме, направлена параллельно уклону вниз и будет оставаться во время движения постоянной. Если через Решение задач по физике обозначить ускорение вагона, сообщаемое ему этой  
равнодействующей, то по второму закону Ньютона 
Решение задач по физике
откуда 
Решение задач по физике
Решив данное уравнение и уравнение Решение задач по физике найдем что 
Решение задач по физике
Теперь найдем Решение задач по физике которые необходимы для определения углаРешение задач по физике и коэффициента Решение задач по физике 
Так как при спуске вагон двигался равноускоренно и начал 
свое движение без начальной скорости, то данный в задаче путь Решение задач по физике пройденный за время Решение задач по физикеравен 
Решение задач по физике
откуда 
Решение задач по физике
Подставив численные значения, найдем 
Решение задач по физике
При подъеме вагона ускорение Решение задач по физике направлено в сторону, противоположную движению вагона. Поэтому при постоянстве Решение задач по физике это движение будет равнозамедленным. Следовательно, данный в задаче путь Решение задач по физике проходимый вагоном при его подъеме за Решение задач по физике будет определяться выражением 

Решение задач по физике
где Решение задач по физике — начальная скорость вагона. 
Начальная и конечная скорости при равнозамедленном движении вагона, как известно, связаны уравнением 
Решение задач по физике
Так как конечная скорость Решение задач по физике равна нулю, то 
Решение задач по физике 
Вводя это значение Решение задач по физике в уравнение Решение задач по физике получим 

Решение задач по физике

откуда 
Решение задач по физике
Подставив численные значения, найдем 
Решение задач по физике
Подставляя же найденные численные значения Решение задач по физике в уравнение Решение задач по физике получим 
Решение задач по физике
Вводя численные значения Решение задач по физике в уравнение Решение задач по физике и принимая в нем Решение задач по физике получим численное значение Решение задач по физике 
Решение задач по физике

Задача № 33. Паровоз тянет вверх по уклону поезд массой Решение задач по физике с постоянной скоростью Решение задач по физике Определить развиваемую паровозом мощность, если коэффициент сопротивления Решение задач по физике и тангенс угла уклона равен Решение задач по физике 
Дано: 
Решение задач по физике

Решение:
Мощность паровоза можно определить, пользуясь формулой 

Решение задач по физике

где Решение задач по физике — сила тяги паровоза, направление которой совпадает с направлением движения поезда; Решение задач по физике — скорость поезда. 
Для определения силы тяги необходимо найти силы, препятствующие движению поезда, т. е. силу сопротивления Решение задач по физике и скатывающую силу Решение задач по физике Сила же нормального давления Решение задач по физике как и скатывающая сила Решение задач по физике обусловлена только действием силы тяжести поезда. Из рис. 18 видно, что Решение задач по физике будут равны: 
Решение задач по физике
и 
Решение задач по физике
Умножая Решение задач по физике коэффициент сопротивления Решение задач по физике найдем, что 
Решение задач по физике
Так как силы Решение задач по физике имеют одно и то же направление, противоположное движению поезда, то сила тяги паровоза будет 
Решение задач по физике
Подставляя значение Решение задач по физике в уравнение Решение задач по физике получим 
Решение задач по физике
Подставляя численные значения Решение задач по физикеРешение задач по физике найдем 
Решение задач по физике

Законы сохранения энергии и импульса

Задача № 34. Через конек крыши переброшен шнур, на концах которого имеются грузы с массами Решение задач по физике и Решение задач по физике С каким ускорением движется эта система грузов, если коэффициент трения грузов о крышу равен Решение задач по физике Углы ската крыши соответственно равны Решение задач по физике (рис, 19). 

Решение задач по физике
Рис.19.

Решение:
Допустим, вся система грузов движется в сторону груза Решение задач по физике На первый груз действует скатывающая сила 
Решение задач по физике

а также сила Решение задач по физике со стороны второго груза и сила трения Решение задач по физике 
Решение задач по физике
Ha второй груз действует скатывающая сила 
Решение задач по физике
а также сила Решение задач по физике со стороны первого груза и сила трения Решение задач по физике
Решение задач по физике
Запишем уравнения движения для первого и второго грузов: 
Решение задач по физике 
откуда 
Решение задач по физике 
Подставляя вместо Решение задач по физике ее значение, получим 
Решение задач по физике
откуда следует, что 

Решение задач по физике
Подставляя значения всех величин, получаем 
Решение задач по физике 
Зная ускорение, можно найти силу натяжения шнура. Движение может происходить и в противоположную сторону. Это зависит 
от величин углов и от масс грузов. 
Если Решение задач по физике, то 
Решение задач по физике
В данном случае ускорение не будет зависеть от масс грузов. 
В частном случае ускорение может быть равно нулю. 

Задача № 35. Акробат весом Решение задач по физике имея при себе груз Решение задач по физике прыгает под углом Решение задач по физике к горизонту со скоростью Решение задач по физике В наивысшей точке сшей траектории он бросает груз горизонтально назад с относительной скоростью Решение задач по физике (рис. 20). На сколько увеличится дальность прыжка акробата вследствие этого? 

Дано:

Решение задач по физике

Решение:
Увеличение дальности прыжка на величину Решение задач по физике обусловлено возрастанием горизонтальной составляющей скорости гимнаста вследствие броска груза. 
Решение задач по физике
Рис.20. 


Во время движения па систему гимнаст — груз действует внешняя сила — сила тяжести. Но в верхней точке траектории, т. е. в момент броска, скорости гимнаста и груза строго горизонтальны. 
Следовательно, количество движения системы до и после броска будет постоянным, при этом следует предположить, что время броска ничтожно мало. 
Задачу удобно решать в системе координат, движущейся со скоростью Решение задач по физике где  Решение задач по физике — горизонтальная составляющая скорости гимнаста до броска. 
В системе координат, связанной с землей, 
Решение задач по физике
где Решение задач по физике — горизонтальная составляющая скорости гимнаста после броска; Решение задач по физике — время движения гимнаста от верхней точки траектории до земли. 

На основании закона независимости движения Решение задач по физике Решение задач по физике Отсюда находим Решение задач по физике
Для вычисления Решение задач по физике применяем закон сохранения количества движения в системе Решение задач по физике которая движется со скоростьюРешение задач по физике В этой системе координат количество движения системы гимнаст — груз до броска Решение задач по физике после броска Решение задач по физике —  где 
Решение задач по физике — масса гимнаста. Следовательно, 
Решение задач по физике
Подставив в равенство Решение задач по физике вместо Решение задач по физике и Решение задач по физикеих значения, получим 
Решение задач по физике 

Задача № 36. На сколько следует на закруглении пути приподнять наружный рельс по отношению к внутреннему, если при скорости движения Решение задач по физике и радиусе кривизны закругления Решение задач по физике давление на оба рельса одинаково? Ширина пути Решение задач по физике 

Дано: 
Решение задач по физике

Решение:

Решение задач по физике
Рис.21.

 
Из рис. 21 видно, что 

Решение задач по физике
С другой стороны, 
Решение задач по физике
Так как Решение задач по физикето
Решение задач по физике

После преобразований получим 
Решение задач по физике
Тогда 
Решение задач по физике
или после подстановки численных значений 
Решение задач по физике

Задача № 37. Шарик массой Решение задач по физике вращается на резиновом шнуре, делая Решение задач по физике оборотов в минуту. На сколько растягивается шнур при вращении? 
Растяжение можно считать пропорциональным приложенной силе; под влиянием силы, равной Решение задач по физике шнур растягивается на Решение задач по физике Длина шнура в нерастянутом состоянии Решение задач по физике 
 
Дано: 
Решение задач по физике

Решение:
Силу, растягивающую шнур, можно выразить на основании двух закономерностей: Решение задач по физике как упругая сила шнура, где Решение задач по физике Решение задач по физикеи Решение задач по физике— удлинение шнура;Решение задач по физике Решение задач по физике как центробежная сила при вращении. 
На основании этих соотношений получаем 
Решение задач по физике
откуда 
Решение задач по физике

Задача № 38. Самолет, пролетающий над озером со скоростью, равной Решение задач по физике описывает в горизонтальной плоскости дугу радиусом Решение задач по физикеПри этом плоскость крыльев самолета наклонена под определенным углом к плоскости горизонта. В самолете па столике стоит стакан с водой. Каково будет положение поверхности воды в стакане по отношению к столику и поверхности воды в озере? 
Дано: 
Решение задач по физике

Решение:
Когда самолет движется прямолинейно, сила тяжести стакана с водой Решение задач по физике перпендикулярна поверхности столика. Когда самолет описывает дугу, на стакан с водой действует дополнительно центробежная сила Решение задач по физике (рис. 22). Сила Решение задач по физике прижимающая воду и стакан к столику, будет равнодействующей центробежной силы Решение задач по физике и силы тяжести Решение задач по физике Она будет перпендикулярна к плоскости крыльев самолета и к поверхности столика. Поверхность воды в стакане займет положение, перпендикулярное равнодействующей силе Решение задач по физике Так как плоскость крыльев самолета наклонена по отношению к плоскости воды в озере под углом Решение задач по физике то 
Решение задач по физике
где Решение задач по физике (Решение задач по физике — масса стакана с водой; Решение задач по физике — ускорение силы тяжести). 
Подставляя в формулу Решение задач по физике вместо Решение задач по физике и Решение задач по физике их значения, получим 
Решение задач по физике
откуда 
Решение задач по физике
Подставляя численные значения, находим 
Решение задач по физике 

Абсолютно упругий удар

Задача № 39. Шар массой Решение задач по физике подвешен на нити длиной Решение задач по физике Его отклонили от положения равновесия до высоты точки подвеса и отпустили. При каком значении угла Решение задач по физике (угол между нитью и вертикалью) нить оборвется, если известно, что нить выдерживает удвоенный вес 
шара? 

Решение:
Шар в топке Решение задач по физике (рис. 23) движется по окружности, радиус которой равен Решение задач по физике Роль центростремительной силы играет равнодействующее натяжение нити Решение задач по физике и составляющая Решение задач по физике веса шара, причем  численно 
Решение задач по физике
Так как 

Решение задач по физике
то 
Решение задач по физике

Решение задач по физике
Рис.23. 

При падении шара с высоты Решение задач по физике его скорость равна 

Решение задач по физике
Из чертежа находим 
Решение задач по физике
откуда 
Решение задач по физике
Подставляя в уравнение Решение задач по физике значение Решение задач по физике и учитывая, что Решение задач по физике Решение задач по физике получаем 
Решение задач по физике
или 

Решение задач по физике
откуда 
Решение задач по физике

Задача № 40. На конце горизонтального стержня, вращающегося вокруг вертикальной оси, закреплена Решение задач по физике-образная трубка в следующих двух положениях: 1) плоскость трубки проходит через ось вращения; 2) оба колена трубки находятся на одинаковом расстоянии от оси вращения. 
Каково положение уровней жидкости в обоих коленах трубки, если стержень вращается с числом оборотов Решение задач по физике в секунду? 
Решение:
1. В первом случае можно заметить разность уровней жидкости в обоих коленах вращающейся трубки (рис. 24). Одновременно наступает изменение мениска в трубке (мениск — свободная поверхность жидкости в трубке). Обращает на себя внимание, что мениск в одном колене трубки становится продолжением мениска в другом колене. Разность уровней в трубке растет с увеличением угловой скорости или числа оборотов. Она обусловлена действием центробежной силы на жидкость, находящуюся в горизонтальной части трубки. В результате действия этой силы жидкость расположится так, как показано на рис. 24, а. 

Решение задач по физике
Рис.24. 

При отсутствии вращения мениск уровня жидкости в сосуде всегда перпендикулярен к направлению действия силы тяжести Решение задач по физике. В нашем случае, кроме силы тяжести Решение задач по физике, действует еще  центробежная сила Решение задач по физике Вследствие этого уровень жидкости расположится перпендикулярно к равнодействующей сил Решение задач по физике т. е. под некоторым углом к горизонтальной плоскости. Наклон мениска создает разность уровней в обоих коленах трубки. В положении (2)  
дополнительная сила Решение задач по физике одинакова в обоих коленах, отсюда и одинаковый наклон мениска. В положении (1) дополнительная сила Решение задач по физике растет с удалением от оси вращения. Вследствие этого, если бы цилиндрический сосуд вращался вокруг оси Решение задач по физике, то мениск установился бы по параболе, показанной пунктирной линией (рис. 24, а). 
Подсчитаем разность уровней жидкости в обоих коленах трубки. 
Как известно, центростремительное ускорение можно выразить 
следующим образом: 

Решение задач по физике
где Решение задач по физике — число оборотов в секунду; Решение задач по физике — расстояние от оси вращения. 
Так как ускорение зависит от оси вращения, то для расчета силы, действующей на жидкость, расположенную в вертикальной части трубки, следовало бы столбик жидкости разбить на участки, подсчитать силу, действующую на каждый такой участок, а затем просуммировать все отдельные силы. Мы упростим решение, взяв среднее значение силы. 
Наименьшее 1и наибольшее ускорения соответственно равны: 
Решение задач по физике
Среднее ускорение 
Решение задач по физике
На жидкость действует центробежная сила 
Решение задач по физике
где Решение задач по физике — длина; Решение задач по физике — поперечное сечение горизонтальной части трубки;Решение задач по физике — плотность жидкости. 
Центробежная сила уравновешена силой гидростатического давления, возникающей за счет разности уровней жидкости в обоих коленах трубки; 
Решение задач по физике
Тогда 
Решение задач по физике
откуда 
Решение задач по физике
Оказывается, разность уровней в большой степени зависит от числа оборотов и длины горизонтальной части трубки и значительно слабее — от расстояния до оси вращения. 

Если расстояние внешнего колена от оси вращения обозначим через Решение задач по физике т. е. Решение задач по физике то формулу можно несколько упростить: 
Решение задач по физике
2. Когда трубка вращается равномерно, уровень жидкости в обоих коленах одинаков. Если же трубка вращается с угловым  ускорением Решение задач по физике то легко показать, что разность уровней выражается формулой 
Решение задач по физике
Формула справедлива, если Решение задач по физике

Задача № 41. Воронка с углом растворения Решение задач по физике вращается вокруг вертикальной оси со скоростью Решение задач по физике В каком месте на внутренней стороне воронки должно находится тело, чтобы оно было в равновесии? 
Дано; 
Решение задач по физике

Решение:
Пусть тело в положении равновесия находится на расстоянииРешение задач по физике от оси вращения. В процессе вращения на тело действуют центробежная сила Решение задач по физике и сила тяжести Решение задач по физике Обе силы разлагаются на  составляющие (рис. 25). При отсутствии трения тело находится в равновесии, когда скатывающая сила Решение задач по физике равна составляющей Решение задач по физике центробежной силы Решение задач по физике

Решение задач по физике

Рис.25.


Центробежная сила 
Решение задач по физикетак как Решение задач по физике
Из рис. 25 видно, что 
Решение задач по физике
Тогда Решение задач по физике или 
Решение задач по физике
откуда 
Решение задач по физике
Подставляя численные значения, получим 
Решение задач по физике
Положение будет неустойчивым, если Решение задач по физике тело скатывается вниз, вследствие чего равновесие еще больше нарушается, и не возвращается в свое первоначальное положение. При Решение задач по физике тело будет выброшено из воронки. 
С учетом коэффициента трения условие равновесия запишется в следующем виде: 
Решение задач по физике
Сила трения равна 
Решение задач по физике
Подставляя известные значения в неравенства 
Решение задач по физике
найдем: 
Решение задач по физике
откуда 
Решение задач по физике
Следовательно, 
Решение задач по физике
Тело находится в равновесии внутри воронки на расстоянии от Решение задач по физике

Задача № 42. Ребенок бросает обруч, одновременно сообщая ему вращение в направлении стрелки (рис. 26, а). При каком соотношении между линейной и угловой скоростями обруч покатится обратно к бросавшему? Сопротивлением воздуха пренебречь. 

Решение:
Во время полета обруч сохраняет как линейную, так и угловую скорость. В момент приземления обруч будет некоторое время  двигаться со скольжением, вращаясь против часовой стрелки и скользя вправо (рис. 26, б). Сила трения Решение задач по физике (величина которой роли не играет) 
Решение задач по физике
Рис.26. 
будет постепенно уменьшать линейную скорость Решение задач по физике причем замедление (линейное) будет равно 
Решение задач по физике 
где Решение задач по физике — масса обруча. 
Та же сила, момент которой Решение задач по физике относительно центра обруча (где Решение задач по физике — радиус обруча), вызовет и угловое замедление вращения 
Решение задач по физике
где Решение задач по физике — момент инерции обруча. 
Для того чтобы обруч покатился назад, необходимо, чтобы линейная скорость уменьшилась до нуля раньше, чем угловая, т. е. чтобы Решение задач по физике где Решение задач по физике — время уменьшения до нуля линейной, a Решение задач по физике — угловой скорости. 
Но 
Решение задач по физике
откуда Решение задач по физике т. е. Решение задач по физике При соотношении Решение задач по физике обруч покатится обратно к бросавшему. 

Динамика поступательного движения

Задача № 2.1. С самолета, летящего горизонтально с постоянной скоростью v, сбрасывается бомба. Где будет находиться самолет, когда бомба достигнет земли?

Решение:

Самолет летит по горизонтали с постоянной скоростью v. Бомба летит по параболе, так как ее движение складывается из движения по горизонтали с начальной скоростью v и равноускоренного падения по вертикали. Если бы не было сопротивления воздуха, то скорость бомбы по горизонтали не отличалась бы от скорости самолета, и самолет все время, и в частности в момент падения бомбы, находился бы над бомбой. Однако в действительности, из-за сопротивления воздуха, горизонтальная скорость бомбы все время уменьшается, и бомба отстает от самолета (рис. 172). Поэтому падение на землю и взрыв бомбы происходят не под самолетом, а далеко позади.

Решение задач по физике

Рис. 172

Задача № 2.2. Ствол ружья и центр яблока, подвешенного на нити, находятся на одной прямой (рис. 9). Попадет ли пуля в мишень, если нить обрывается и яблоко начинает свободно падать в момент вылета пули из дула? Сопротивление воздуха не учитывать.

Решение задач по физике

Рис. 9

Решение:

Движение пули представляет собой сложение двух движений: по горизонтали с постоянной скоростью, с которой пуля вылетела из дула, и падения под действием силы тяжести, начинающегося в момент вылета пули из дула. В результате этих двух движений пуля будет двигаться по параболе.  В вертикальном направлении пуля пролетит расстояние s = gt2/2, где t — время ее движения. Точно такое же расстояние пройдет по вертикали и падающее яблоко, так как сила тяжести сообщает всем телам одинаковые ускорения. Поэтому пуля попадет в яблоко или на лету (случай 1 на рис. 173), или в момент падения яблока на землю (случай 2), если только расстояние от стрелка до мишени не больше, чем дальность полета пули.

Решение задач по физике

Рис. 173

Задача № 2.3. В каком из двух случаев ружье стреляет дальше: когда оно закреплено в станке или когда оно просто подвешено, как на рис. 10?

Решение:

Дальность полета пули зависит при прочих равных условиях от начальной скорости пули (скорость при вылете). Если ружье подвешено так, что может отклоняться в горизонтальном направлении, то вследствие отдачи ружья при выстреле начальная скорость пули будет меньше и, следовательно, пуля полетит не так далеко, как в случае ружья, закрепленного в станке.

Абсолютно неупругий удар

Задача № 2.4. Какие капли дождя падают быстрее крупные или меньше? Почему? Считать, что шарообразная форма капли при падении не изменяется.

Решение задач по физике       Решение задач по физике

Рис. 10                                                            Рис. 11

Решение:

На падающую каплю действуют две силы: постоянная сила тяжести, ускоряющая движение капли, и сила сопротивления воздуха, замедляющая ее движение и растущая с ростом скорости капли. Сила сопротивления воздуха растет до тех пор, пока она не станет равной силе тяжести. Дальше прекращается изменение скорости, и падение капель происходит с постоянной скоростью. При увеличении размеров капли сила тяжести увеличивается пропорционально объему, т. е. пропорционально третьей степени радиуса, а сила сопротивления — пропорционально сечению капли, т. е. пропорционально квадрату радиуса. Поэтому при увеличении радиуса капли сила тяжести увеличивается быстрее, чем сила сопротивления воздуха, а значит и та постоянная скорость, с которой капля падает на землю, растет по мере увеличения размеров капли.

Задача № 2.5. Два шара одинакового радиуса и из одного и того же материала, но один сплошной, массой m1, а другой полый, массой m2, падают в воздухе с одинаковой высоты. Какой из шаров упадет быстрее?

Решение:

Так как сечения шаров одинаковы, то при одинаковой скорости они будут испытывать одинаковую силу сопротивления воздуха. Для каждого шара в отдельности, учитывая, что сила тяжести равна mg, а сила сопротивления воздуха равна f, можно записать уравнение движения: mа = mg - f. Мacca m1 сплошного шара больше массы m2 полого шара, поэтому (при m1 > m2) имеем а1 > а2. Следовательно, сплошной шар упадет быстрее, чем полый.

Задача № 2.6. Трубка в форме ромба со стороной а с закругленными углами расположена в вертикальной плоскости, как показано на рис. 11. Один раз шарик скатывается в трубке по сторонам АВ и ВС, а другой раз по сторонам AD и DC. В каком случае он скатится быстрее?

Решение:

Когда стороны АВ и DC ромба (рис. 174) почти горизонтальны, сразу ясно, что во втором случае (движение по сторонам AD и DC) шарик скатится быстрее. Это видно из того, что по стороне DC шарик будет проходить с большой средней скоростью, которую он приобретет, двигаясь по стороне AD. В первом же случае сторону АВ шарик пройдет с очень малой средней скоростью (так как ускорение его мало).

Решение задач по физике

Рис. 174

Результат, полученный для этого частного случая, остается справедливым и в общем случае, в чем можно убедиться с помощью следующего расчета. Пусть стороны АВ и DC образуют с горизонталью угол Решение задач по физике а стороны ВС и DA образуют с вертикалью угол Решение задач по физике Если шарик скатывается по сторонам АВ + ВС, то он тратит на это время t1 + t2 , где t1 и t2 — время, которое он тратит на скатывание по АВ и ВС соответственно. Ускорение при движении по АВ равно Решение задач по физике Поэтому для вычисления времени t1 имеем уравнение Решение задач по физике Ускорение при движении по ВС равно Решение задач по физике и это движение происходит с начальной скоростью Решение задач по физике следовательно, мы можем определить t2, решая уравнение

Решение задач по физике

Для второго случая, при скатывании шарика по сторонам AD + DC, будем иметь такие же выражения с той лишь разницей, что ускоренияРешение задач по физике и  Решение задач по физике нужно поменять местами. Поэтому для первого случая сумма двух времен равна

Решение задач по физике

а для второго случая та же сумма времен равна

Решение задач по физике

Так как ясно, что Решение задач по физике меньше, чем Решение задач по физике, то в первом случае сумма времен больше, чем во втором.

Задача № 2.7. Тело массой m начинает скользить без трения с верхнего конца наклонной грани клина, лежащего на горизонтальной плоскости, причем между клином и плоскостью трение также отсутствует. Масса клина М, угол наклонной грани клина с горизонтом Решение задач по физике. Найти силу давления тела на клин и клина на плоскость, а также ускорения клина и тела относительно плоскости.

Решение:

Рассмотрим силы, действующие на тело массой m и клин массой М (рис. 175). На тело действуют две силы — сила тяжести mg и сила реакции клина N1. На клин действуют: сила тяжести Mg, сила давления тела F и сила реакции плоскости N2. Под действием горизонтальной составляющей силы давления тела клин движется влево относительно плоскости с некоторым горизонтальным ускорением а1 , определяемым уравнением

Решение задач по физике                                                                                                        (1)

В вертикальном направлении клин не имеет ускорения, поэтому

Решение задач по физике                                                                                       (2)

Обозначим составляющие ускорения тела относительно клина: горизонтальную через а2 и вертикальную через а3

Решение задач по физике

Рис. 175

Тогда составляющие ускорения тела относительно плоскости будут: горизонтальная а2 - а1 и вертикальная а3. Эти ускорения определяются уравнениями

Решение задач по физике                                                                                          (3)

Решение задач по физике                                                                                            (4)

причем, очевидно, N1 = F и Решение задач по физике                                                         (5)

Из уравнений (1) и (3) – (5) находим силу давления тела на клин:

Решение задач по физике                                                                      (6)

Теперь из уравнения (2) можно найти силу давления клина на плоскость, равную по модулю силе реакции плоскости:

Решение задач по физике

Далее, из (1) и (6) находим ускорение клина относительно плоскости:

Решение задач по физике                                                                     (7)

Из (3), (6) и (7) получаем горизонтальную составляющую ускорения тела относительно клина:

Решение задач по физике                                                              (8)

и горизонтальную составляющую ускорения тела относительно плоскости:

Решение задач по физике

Из (8) и (5) находим вертикальную составляющую ускорения rела относительно плоскости:

Решение задач по физике

Задача № 2.8. На тонкое кольцо радиусом R в точках А, В, С, D, являющихся вершинами вписанного квадрата, действуют в направлении против часовой стрелки равные по модулю силы: F1 = F2 = F3 = F4 = 10 Н (рис. 12). Кроме того, в точках А и В действуют две равные по модулю силы, направленные по диагоналям квадрата: F5 = F6 = Решение задач по физике. Найти равнодействующую всех сил и точку ее приложения. Как будет двигаться кольцо под действием указанных сил?

Решение:

Разложим силу F5 на две составляющие по направлениям сторон квадрата (рис. 176). Полученные силы Решение задач по физике и F'2 будут равны каждая 10 Н. Поэтому силы F2 и F'взаимно уравновешиваются, а в точке А будет действовать по направлению AD сила F1 и F'1 = 20 Н. Перенесем силу F6 по ее направлению в точку D и разложим на две силы по направлениям сторон квадрата. Получим силы F'3 и F'4, равные каждая 10 Н. Поэтому силы F4 и F'4 взаимно уравновешиваются, а по направлению DK будет действовать результирующая сила F1 + F'1 + F'3 = 30 Н.

Решение задач по физике

Рис. 176

Остается сложить эту силу с антипараллельной ей силой F3. Равнодействующая этих сил R = 20Н, очевидно, будет параллельна складываемым силам и направлена по прямой, отстоящей от стороны AD на расстояние, равное половине стороны вписанного квадрата. Под действием этой силы кольцо будет двигаться поступательно с ускорением и одновременно вращаться (так как сила имеет момент относительно центра кольца) также с ускорением. Этот качественный вывод можно было бы сделать с самого начала из тех соображений, что силы F1 - F4 составляют две пары сил, вращающих кольцо в одну сторону, а силы F5 и F6 имеют равнодействующую, которая будет двигать кольцо поступательно с ускорением.

Задача № 2.9. К спиральной пружине, растяжение которой подчиняется закону Гука, прикреплена чашка весов с гирей (рис. 13). С какой силой надо оттянуть чашку с гирей вниз, чтобы, после того как ее отпустят, гиря в какой-то момент перестала давить на чашку?

Решение задач по физике               Решение задач по физике                    Решение задач по физике

Рис. 12                                                              Рис. 13                                         Рис. 14

Решение:

Когда чашка с гирей находится в покое, то пружина растянута настолько, что ее упругая сила уравновешивает вес чашки и гири. Если пружину вывести из этого положения, оттянув ее немного вниз, и отпустить, то начнутся колебания относительно положения равновесия. В верхнем положении растяжение пружины при этом будет наименьшим. Чтобы гиря перестала давить на чашку, необходимо, чтобы чашка начала двигаться вниз от верхнего положения с таким же ускорением, как гиря, т. е. с ускорением, равным ускорению свободного падения g. Это, очевидно, будет тогда, когда в верхнем положении пружина не будет растянута. Так как крайнее верхнее положение симметрично с крайним нижним относительно положения равновесия чашки с грузом, то для того, чтобы чашка дошла кверху до положения, при котором пружина вовсе не растянута, нужно сначала оттянуть пружину еще на такое же расстояние, на какое она растянута в положении равновесия.

Следовательно, чашку с гирей надо оттянуть с силой, равной весу чашки и гири.

Задача № 2.10. Две пластины массами m1 и m2 соединены пружиной жесткостью k, массой которой можно пренебречь (рис. 14). С какой силой F надо надавить на верхнюю пластину, чтобы, после прекращения действия силы, верхняя пластина, подпрыгнув, приподняла и нижнюю? Считать, что закон Гука выполняется все время.

Решение:

Пружина в начальный момент сжата (по сравнению с нормальной своей длиной) весом верхней пластины на величину х1 = m1g/k. Чтобы пружина смогла приподнять при своем растяжении нижнюю пластину, она должна быть растянута по сравнению с нормальной своей длиной на величину, большую чем х2 = m2g/k.

Следовательно, надо надавить на верхнюю пластину с такой силой, чтобы при прекращении действия силы пружина подпрыгнула на высоту, большую чем х1 + х2 = (m1 + m2)g/k, считая от того положения, которое занимала пластина, пока на нее не давили. Но освобожденная верхняя пластина подпрыгнет кверху на такое же расстояние от положения равновесия, на каком она при нажатии находилась ниже его. Значит, для того чтобы после прыжка верхняя пластина приподняла нижнюю, она должна быть прижата вниз на расстояние, большее чем х1 + х2. Для этого на верхнюю пластину нужно надавить с силой F > k(x1 + х2) = (m1 + m2)g.

Наклонная плоскость

Задача № 2.11. По наклонной плоскости равномерно спускается велосипедист массой m масса велосипеда равна М. Какова по модулю и направлению сила реакции N плоскости?

Решение:

Так как велосипедист движется равномерно, то все силы, действующие на него, взаимно уравновешиваются. Значит, сила реакции наклонной плоскости N должна быть равна по модулю и противоположна по направлению силе тяжести велосипедиста вместе с велосипедом, т. е. должна быть направлена вертикально вверх. Таким образом, сила реакции плоскости в рассматриваемом случае не перпендикулярна к наклонной плоскости (рис. 177) и имеет, кроме перпендикулярной плоскости составляющей N1, еще составляющую N2, направленную вдоль плоскости. Эта составляющая представляет собой силу трения, действующую на велосипед со стороны плоскости. Если бы между плоскостью и велосипедом не было трения, то велосипедист, несмотря на наличие тормозов, спускался бы не равномерно, а с постоянным ускорением.

Решение задач по физике

Рис. 177

Задача № 2.12. Доска массой М, наклоненная под углом Решение задач по физике к горизонту, лежит на двух опорах А и В (рис. 15), по которым она может скользить вниз без трения. С каким ускорением а и в каком направлении должен двигаться по этой доске человек массой m, чтобы доска не скользила?

Решение задач по физике                           Решение задач по физике

Рис. 15                                                                                               Рис. 16

Решение:

Чтобы доска не скользила, на нее вдоль ее направления должна действовать со стороны ног человека сила, уравновешивающая составляющую силы тяжести доски (т. е. равная Решение задач по физике) и направленная кверху. При этом на человека со стороны доски будет действовать равная по модулю и противоположная по направлению сила, направленная вдоль доски вниз. Кроме того, на человека действует составляющая силы тяжести Решение задач по физике Следовательно, человек должен двигаться вдоль доски вниз с таким ускорением (относительно земли), чтобы

Решение задач по физике

отсюда находим ускорение: Решение задач по физике

Задача № 2.13. Клин с углом при вершине в 90° и углами при основании Решение задач по физике и Решение задач по физике находится на гладком столе. По его боковым граням одновременно начинают скользить без трения бруски А и В массой m каждый (рис. 16). Будет ли при этом клин скользить по столу, если трение отсутствует?

Решение:

Брусок А, на который действует сила тяжести mg, давит на клин с силой Решение задач по физике направленной перпендикулярно к соответствующей грани клина, а под действием силы F2 скользит ускоренно вниз по грани клина (рис. 178). Силу F1 можно разложить на две силы: вертикальную Решение задач по физике и горизонтальную Решение задач по физике

Решение задач по физике

Рис. 178

Рассуждая совершенно аналогично в отношении бруска В, мы получим, что брусок В действует на клин по вертикали с силой Решение задач по физике и по горизонтали вправо с силой Решение задач по физике Так как угол при вершине клина равен 90°, Решение задач по физике Решение задач по физике, и, следовательно, силы, с которыми бруски действуют на клин по горизонтали, равны друг другу по модулю, но противоположно направлены и клин no столу скользить не будет. А по вертикали на стол будут действовать, кроме силы тяжести самоrо клина, еще две силы: Решение задач по физике и Решение задач по физике Их сумма равна

Решение задач по физике

Задача № 2.14. На дне закрытой пробирки сидит муха. Пробирка свободно падает, оставаясь в вертикальном положении (рис. 17), Как изменится продолжительность падения, если муха во время падения перелетит из нижней части пробирки в верхнюю?

Решение задач по физике

Рис. 17

Решение:

Если какое-нибудь тело свободно падает и в это время внутри тела происходят какие-нибудь перемещения отдельных ero частей, то центр масс тела продолжает двигаться с ускорением свободноrо падения. Поэтому и центр масс системы пробирка – муха будет двигаться по-прежнему с ускорением свободного падения. Когда муха перелетает из нижней части пробирки в верхнюю, дно пробирки несколько опускается относительно центра масс системы пробирка – муха. Таким образом, дно пробирки скорее ударится о землю, чем в том случае, когда муха будет оставаться неподвижной. Иначе говоря, продолжительность падения уменьшится.

Задача № 2.15. Птица находится в закрытом ящике, стоящем на одной из чашек весов. Пока птица сидит на дне ящика, весы уравновешиваются гирями, положенными на другую чашку. Нарушится ли равновесие весов, если птица взлетит и будет парить в воздухе внутри ящика?

Решение:

Равновесие весов не нарушится, если птица взлетит и будет парить в воздухе внутри ящика. Объясняется это тем, что при парении птица должна отбрасывать воздух вниз, чтобы создать подъемную силу, поддерживающую ее в воздухе. При этом отбрасываемый вниз воздух будет создавать добавочное давление на дно ящика и среднее значение силы этого давления как раз будет равно силе тяжести птицы. Правда, при взлете и резких движениях птицы (когда она будет двигаться с ускорением) значение силы давления может изменяться, так что весы начнут качаться около положения равновесия. Но в среднем равновесие нарушено не будет.

Задача № 2.16. Аэростат массой М опускается с постоянной скоростью v. Сила сопротивления воздуха f пропорциональна скорости. Подъемная сила аэростата равна Q. Какой массы m балласт надо выбросить из гондолы аэростата, чтобы он поднимался вверх с той же по модулю скоростью v?

Решение:

Если аэростат опускается с постоянной скоростью, то сила сопротивления воздуха направлена вверх и, значит, сила тяжести аэростата Mg уравновешена подъемной силой Q и сопротивлением воздуха f:

Mg= Q +f.                                                                                                                      (1)

Если, после того как выброшен балласт массой m, аэростат начинает подниматься с той же по модулю скоростью v, то сила сопротивления воздуха изменит свое направление, т. е. будет действовать вниз. Следовательно, мы получим

Mg – mg=Q – f.                                                                                                                (2)

Из уравнений (1) и (2) находим массу балласта: m = 2(М – Q/g) = 2f/g .

Задача № 2.17. Пуля массой m летит вертикально вверх, достигает максимальной высоты и вертикально же падает вниз. В каких точках этой траектории скорость и ускорение пули имеют максимальное и минимальное значения? Учесть силу сопротивления воздуха f, которая растет с увеличением скорости движения пули.

Решение:

В каждой точке траектории полное ускорение пули направлено вниз; при движении вверх оно равно g + f/m, при движении вниз g – f/m. Сила сопротивления воздуха f всегда направлена противоположно скорости движения и растет при возрастании скорости. Но максимальной скоростью пуля будет обладать в самой нижней точке на пути вверх (в момент вылета), а на обратном пути — в момент падения. (Заметим, кстати, что максимальная скорость на обратном пути меньше, чем на пути вверх, так как часть энергии пули расходуется на преодоление сопротивлении воздуха.) Следовательно, и сила сопротивления воздуха f сначала направлена вниз и будет наибольшей в самой низшей точке пути (в момент вылета), затем она будет уменьшаться по мере уменьшения скорости, пройдет через нуль (в верхней точке) и, изменив направление на обратное, снова начнет возрастать. Ускорение же пули будет все время направлено вниз и будет максимальным в момент вылета, затем оно уменьшается и в момент падения пули будет минимальным.

Упругая сила

Задача № 2.18. В массивную трубку вставлена пружина, которая в свободном состоянии занимает всю длину трубки. На пружину положен шарик, который сжимает ее примерно вдвое (рис. 18). В наклонном положении трубка начинает свободно падать. Что произойдет с шариком?

Решение задач по физике                                                Решение задач по физике

Рис. 18                                                                                          Рис. 19

Решение:

Пока трубка неподвижна, шарик лежит в трубке, и сила давления шарика на пружину уравновешена упругой силой F сжатой пружины. Когда трубка начинает свободно падать, шарик сначала падает с меньшим ускорением, чем трубка, и отстает от нее — пружина распрямляется. Под действием упругой силы пружины шарик не только отстает от трубки в своем движении в вертикальном направлении, но и приобретает скорость в горизонтальном направлении. Поэтому шарик появляется у отверстия трубки, имея некоторую горизонтальную скорость, а дальше, как и всякое тело, брошенное с горизонтальной начальной скоростью, шарик будет падать по параболе (рис. 179).

Решение задач по физике

Рис. 179

Задача № 2.19. На неподвижной тележке укреплено коромысло весов, на одном конце которого висит груз, а другой конец с помощью пружины соединен с полом тележки (рис. 19). Если тележке с помощью постоянной силы сообщить ускорение в горизонтальном направлении, то груз отклонится на некоторый угол в сторону, противоположную ускорению. Изменится ли при этом натяжение пружины?

Решение:

Ускорение тележки скажется в том, что груз отклонится на некоторый угол, и сила натяжения нити, на которой висит груз, увеличится. Но момент сил, действующих на коромысло весов, определяется составляющей силы, перпендикулярной к плечу, т. е. вертикальной составляющей силы натяжения нити, которая не изменится. Это видно из того, что груз, отклонившись, остается. неподвижным относительно тележки, т. е. его ускорение в вертикальном направлении равно нулю, а значит, вертикальная составляющая силы натяжения нити уравновешивает силу тяжести груза. Но если момент силы, действующий на левое плечо коромысла, не изменится, то и натяжение пружины тоже останется неизменным.

Задача № 2.20. В цилиндрической части баллона со сжатым воздухом закреплен поршень. Объем цилиндрической части мал по сравнению с объемом баллона (рис. 20). Если освободить поршень от удерживающих его сил, то поршень выбрасывается из баллона вниз (трение между поршнем и стенками отсутствует). Как изменится время движения поршня в цилиндрической части, если: на поршень положить небольшой шарик? увеличить массу поршня на массу шарика?

Решение задач по физике

Рис. 20

Решение:

Если на поршень положить небольшой шарик, то поршню сообщают ускорение его сила тяжести и сила давления сжатого в баллоне воздуха (так как объем цилиндрической части мал по сравнению с объемом баллона, то давление воздуха во время движения поршня в цилиндрической части можно считать постоянным). Шарик, положенный на поршень, падает только под действием силы тяжести, т. е. с меньшим ускорением, чем поршень, и отстает от него. Поэтому шарик не изменяет ускорения, а значит, и времени движения поршня. Если же массу поршня увеличить на массу шарика, то сила давления газа прежняя, а масса поршня больше. Значит, ускорение поршня будет меньше, а время движения внутри цилиндрической части больше, чем в первом случае.

Задача № 2.21. В кинофильме «Смелые люди» герой фильма на ходу поезда, идущего по ровному пути, вскакивает на крепление буферов и отцепляет два последних вагона. В каких случаях это возможно?

Решение:

Чтобы можно было расцепить сцепку, она не должна быть натянута. Следовательно, это возможно только в том случае, когда поезд идет под уклон или с торможением по ровному месту. Но при этом отцепленные вагоны могут начать отставать от поезда, лишь когда уклон кончился или кончилось торможение (либо нужно начать тормозить отцепленные вагоны).

Неподвижный блок

Задача № 2.22. Два груза массами m1 и m2 соединены нерастяжимой нитью, перекинутой через неподвижный блок. Определить ускорение грузов а, силу натяжения нити Т и силу давления F на ось блока. Массой блока и нити пренебречь.

Решение:

Будем считать для определенности, что m1 > m2 (рис. 180).

Решение задач по физике

Рис. 180

В таком случае m1 опускается вниз, а m2 поднимается вверх. Так как массой блока пренебрегаем, то моменты обеих сил натяжения нити равны между собой, а так как их плечи одинаковы, то, значит, и силы натяжения одинаковы, т. е. Т1 = Т2. Движение груза массы m1 вниз происходит под действием двух сил: силы тяжести m1g и силы натяжения нити Т1 , направленных, как показано на рис. 180. Поэтому, обозначая ускорение груза массы mчерез а, запишем уравнение его движения:

Решение задач по физике                                                                          (1)

Груз массой m2 движется вверх с тем же ускорением а под действием двух сил: силы тяжести m2g и силы натяжения нити Т2, поэтому

Решение задач по физике                                                                            (2)

Из уравнений (1) и (2) получаем

Решение задач по физике

Сила давления F на ось блока равна сумме сил натяжения нитей, т. е.

Решение задач по физике

Задача № 2.23. Через середину стержня проходит горизонтальная ось О, вокруг которой он может вращаться. На концах стержня укреплены грузы массами m1 = 1 кг и m2 = 7 кг (рис. 21). Стержень приведен в горизонтальное положение и освобожден без толчка. Какую силу давления F стержень оказывает на ось в начальный момент после освобождения? Массой стержня и трением в оси пренебречь.

Решение задач по физике                    Решение задач по физике

Рис. 21                                                                                    Рис. 22

Решение:

Оба груза рассматриваем как материальные точки, расположенные в центрах масс грузов. Груз массой m2 начинает опускаться с линейным ускорением а, не равным g, так как из-за наличия груза массой m1 на него действует со стороны стержня направленная вверх сила F1. Поэтому, на основании второго закона Ньютона, можем написать

Решение задач по физике                                                                    (1)

Груз массой m1 начинает подниматься с тем же по модулю ускорением а, так как из-за наличия груза mна него действует со стороны стержня направленная вверх сила F2. Поэтому

Решение задач по физике                                                               (2)

Так как массой стержня можно пренебречь, то моменты сил F1 и F2 , действующих на стержень, относительно оси вращения должны быть равны по модулю и противоположно направлены, а значит, сумма их равняется нулю. (В противном случае стержень с исчезающе малой массой получил бы бесконечно большое угловое ускорение.) Но плечи этих сил равны, а потому F1 = F2 = F.

Тогда из уравнений (1) и (2) получаем

Решение задач по физике

Равнодействующая сил F1 и F2 равна по модулю и противоположна по направлению силе давления стержня на ось -в начальный момент. Следовательно, последняя сила равна

Решение задач по физике

Рассмотренная нами задача принципиально не отличается от задачи 2.22; в ней роль блока выполняет стержень, закрепленный в середине.

Задача № 2.24. На абсолютно гладком столе лежит цепочка массой М и длиной l, свешивающаяся наполовину за край стола (рис. 22, а). Как изменится время ее соскальзывания, если к концам цепочки прикрепить два одинаковых шарика массой m (рис. 22, б)?

Решение:

 Масса единицы длины цепочки равна m0 = M/l. Пусть в некоторый момент со стола свисает часть цепочки длиной х; по условию задачи в начальный момент времени х = l/2. Тогда сила, приводящая цепочку в движение, будет пропорциональна силе тяжести свисающей части, т. е. m0xg. Ускорение цепочки будет равно m0xg/М. В начальный момент, когда m0x = M/2, ускорение равно g/2, а затем оно возрастает, так что цепочка будет двигаться ускоренно.

Если к концам цепочки прикрепить одинаковые массы m, то в момент, когда со стола свисает часть цепочки длины х, движущей силой будет сила m0xg + mg = (m0x + т) g, а ускорение  цепочки в этот момент равно (m0x + m) g / (M + 2m). Для решения вопроса о том, в каком случае цепочка соскользнет быстрее, нужно выяснить, в каком случае ускорение нарастает быстрее; надо сравнить два ускорения:

m0xg/М   и   (m0x + m) g / (M + 2m).

Для этого приведем их к одному знаменателю и сравним числители. Числитель первой дроби равен m0xgМ + 2m0xgm, числитель второй m0xgМ + mgМ. Ясно, что эти числители равны между собой в начальный момент, когда m0x = М/2. В последующие моменты ускорение в первом случае всегда больше, чем во втором. Итак, цепочка соскользнет быстрее, когда шариков на ее концах не будет. Этот же результат можно получить на основании закона сохранения энергии, учитывая, что в момент, когда конец цепочки соскальзывает со стола, ее центр масс находится на расстоянии l/2 ниже края стола. Тогда если v1 — скорость в этот момент, то без шариков на концах

Решение задач по физике или Решение задач по физике а с шариками на концах Решение задач по физике или Решение задач по физике т. е. время соскальзывания цепочки должно быть меньше в том случае, когда шариков на ее кон· цах нет.

Задача № 2.25. Через невесомый блок А перекинута нить, к одному концу которой прикреплен груз m1 , а к другому невесомый блок В, на нити которого висят грузы m2 и m3. Блок А со всеми грузами подвешен к пружинным весам (рис. 23). Найти ускорение а1 груза m1 и показание Т пружинных весов, считая, что Решение задач по физике

Решение задач по физике

Рис. 23

Решение:

Так как блоки невесомы, то Т2 = Т3 и Т1 = 2Т2 (см. задачу 2.22). Далее, показание пружинных весов Т = 2Т1. Груз массой m1 движется вниз с ускорением а1 под действием двух сил: силы тяжести m1g и силы натяжения нити Т1 (рис. 181). Поэтому уравнение движения для этого груза имеет вид

Решение задач по физике                                                                         (1)

Если бы блок В не поднимался вверх с ускорением а1 , то груз m2 (для определенности считаем m2 > m3) двигался бы вниз с ускорением а2, а груз m3 — вверх с тем же ускорением. Учитывая, что блок В движется вверх, мы будем иметь следующие уравнения движения грузов m2 и m3 :

Решение задач по физике                (2)                       Решение задач по физике         (З)

Из (2) и (3) получаем   Решение задач по физике

Подставляя это значение а2 в уравнение (2), получим Решение задач по физике

Теперь из уравнения (1) найдем, что ускорение груза Решение задач по физике

а показание пружинных весов равно Решение задач по физике

Решение задач по физике

Рис. 181

Задача № 2.26. Однородная цепочка массой М и длиной l свешивается со стола и удерживается в равновесии силой трения. Найти коэффициент трения покоя k, если известно, что наибольшая длина свисающего со стола конца, при которой цепочка еще не начинает скользить, равна l1.

Решение:

На цепочку действуют две силы: сила тяжести свисающей части и сила трения со стороны стола на часть цепочки, лежащую на столе. При длине свисающей части l1 уравновешивающая силу тяжести этой части сила трения покоя имеет наибольшее значение. Обозначим через m0 массу, приходящуюся на единицу длины цепочки (m0 = M/l), и через k — коэффициент трения покоя, т. е. отношение максимальной силы трения покоя к силе нормального давления. Тогда условие равенства сил

m0gl1 – km0g (l – l1) = О дает искомую величину: k = 11/(l – l1).

Задача № 2.27. Если локомотив не может сдвинуть тяжелый поезд с места, то машинист применяет следующий прием: он дает задний. ход и, толкнув состав немного назад, затем дает передний ход. Объяснить, почему этот прием позволяет сдвинуть состав с места.

Решение:

Состав обычно останавливается в таком положении, что сцепки между вагонами оказываются натянутыми (так как сжатые при торможении буфера расталкивают вагоны и натягивают сцепки). Поэтому, чтобы состав двинулся вперед, сила тяги локомотива должна преодолеть силу трения покоя сразу всех вагонов. Толкая же состав назад, локомотив толкает сначала лишь первый вагон, для чего он должен преодолеть силу трения покоя только одного вагона. После того как вагон тронулся с места, вместо силы трения покоя начинает действовать сила трения скольжения в подшипниках осей колес и сила трения качения колес по рельсам. Эти силы трения значительно меньше, чем сила трения покоя. Поэтому, толкая вместе с первым вагоном второй, сила тяги локомотива в состоянии преодолеть силу трения покоя второго вагона, и т. д. После того как весь состав подался назад и буфера у всех вагонов сдавлены, машинист дает передний ход. При этом сила тяги локомотива, натягивая сцепку первого вагона, преодолевает сначала трение покоя лишь одного первого вагона, затем трение покоя лишь второго вагона и т. д., пока не сдвинет весь поезд. Сжатые буфера между вагонами также способствуют этому.

Задача № 2.28. По закону Ньютона изменить состояние движения тела может только внешняя сила, действующая со стороны какого-нибудь другого тела. Какая же внешняя сила останавливает автомобиль или другой подобный самодвижущийся экипаж при торможении?

Решение:

Торможение автомобиля, автобуса или вагона трамвая заключается в том, что тормозные колодки создают момент сил, которые должны вызвать замедление вращения колес. Но если бы вращение колес замедлилось без уменьшения скорости самодвижущегося экипажа, то должно было бы начаться скольжение колес по земле, направленное вперед. При этом со стороны земли начинает действовать сила трения, препятствующая возникновению скольжения, т. е. направленная назад. Эта сила уменьшает скорость движения экипажа и, в конце концов, останавливает его. Следовательно, земля является тем внешним телом, со стороны которого действует сила, изменяющая скорость движения.

Задача № 2.29. Щетка с длинной ручкой лежит горизонтально на вытянутых указательных пальцах широко раздвинутых рук (рис. 24). Что произойдет, если левая рука будет неподвижна, а правую мы будем придвигать к левой, держа ее все время на одном и том же уровне? Что произойдет, если неподвижна будет правая рука, а левую мы будем придвигать к ней? Что произойдет, если мы одновременно будем двигать обе руки навстречу друг другу?

Решение задач по физике

Рис. 24

Решение:

Пусть щетка находится на левом конце ручки. Центр масс ручки со щеткой находится между пальцами ближе к левому пальцу. Сила давления ручки со щеткой на оба пальца будет поэтому неодинакова: на левый палец она больше, чем на правый. Поэтому и сила трения между ручкой и пальцем, пропорциональная силе давления, будет больше для левого пальца.

Если правую руку двигать к левой, то щетка будет оставаться неподвижной, так как большая сила трения ручки о левый палец будет удерживать ее на месте до тех пор, пока оба пальца не окажутся на равных расстояниях от центра масс щетки. Дальше при движении правого пальца давление на него станет больше, чем на левый, и щетка будет двигаться вместе с правым пальцем, пока центр масс снова не окажется посредине между пальцами. Так будет продолжаться вплоть до того момента, когда пальцы сойдутся.

Если же левую руку двигать к правой, то щетка двинется вместе с левым пальцем, скользя по правому до тех пор, пока оба пальца не окажутся на равных расстояниях от центра масс. Дальнейшее движение будет происходить так же, как и в предыдущем случае. Если двигать руки навстречу друг другу, то щетка будет двигаться вместе с левой рукой до тех пор, пока центр масс ручки со щеткой не окажется на одинаковом расстоянии от обоих пальцев, а затем оба пальца будут скользить под ручкой, а сама щетка будет оставаться неподвижной. Так как во всех этих случаях центр масс ручки со щеткой останется между пальцами, щетка все время находится в равновесии и не падает.

Центростремительное ускорение

Задача № 2.30. Если быстро движущийся автомобиль резко затормозить, то его передок опускается книзу. Почему это происходит?

Решение:

При торможении автомобиля на его колеса (для упрощения будем считать, что только на задние) со стороны земли начнет действовать сила трения f .

Чтобы рассмотреть, как влияет эта сила на движение центра масс автомобиля, представим себе (рис. 182), что к центру масс О приложены две силы: F1 и F2, равные по модулю силе трения f, направленные параллельно ей и противоположно друг другу. Мы имеем право так поступить потому, что от приложения двух равных и противоположных сил F1 и F2 движение автомобиля не меняется. Но эти три силы можно рассматривать как силу F1 и пару сил: F2 и f. Нетрудно видеть, что сила F1 тормозит автомобиль, а пара сил F2 и f вызывает вращение в направлении, указанном стрелкой, т. е. опускание передней части автомобиля.

Решение задач по физике

Рис. 182

Если тормоза действуют также на передние колеса и, следовательно, сила трения со стороны земли действует на обе пары колес, то результат не изменится, так как при данной силе трения f момент ее относительно точки О не зависит от того, как распределена эта сила между колесами.

Задача № 2.31. Небольшое тело скользит с трением из точки А в точку В по цилиндрическим поверхностям одинакового радиуса, один раз по пути 1, второй раз по пути 2 (рис. 25). Сила трения: не зависит от скорости и коэффициент трения в обоих случаях один и тот же. В каком из двух случаев скорость тела в точке В будет больше?

Решение задач по физике                              Решение задач по физике                       

Рис. 25                                                                     Рис. 26

Решение:

Скорость тела в точке В зависит от того, какую часть своей потенциальной энергии израсходует тело на работу против силы трения. На путях 1 и 2 силы трения не одинаковы, так как не одинаковы силы давления тела на поверхность. Это обусловлено следующими обстоятельствами.

При движении по любому криволинейному пути тело должно иметь центростремительное ускорение. На пути 1 это ускорение направлено вниз, на пути 2 — вверх.

Это ускорение, направленное нормально к поверхности, по которой скользит тело, возникает потому, что нормальная к поверхности составляющая силы тяжести тела и сила реакции поверхности, действующая на тело, не равны по модулю. Результирующая этих двух сил (их разность, поскольку эти силы направлены в противоположные cтopoны и сообщает телу нужное центростремительное ускорение. Поэтому на пути 2, где центростремительное ускорение направлено кверху, сила реакции везде больше нормальной составляющей силы тяжести, на пути 1 — везде меньше нормальной составляющей силы тяжести. Модули нормальной составляющей силы тяжести на путях 1 и 2, вообще говоря, различны, так как различен угол нормали к поверхности с вертикалью. Но так как обе поверхности одинакового радиуса, тo нормальные составляющие силы тяжести изменяются на двух путях в одинаковых пределах, но в обратном порядке. Поэтому можно утверждать, что сила реакции поверхности, действующая на тело (а значит, и сила давления тела на поверхность) на пути 1 в среднем меньше, чем на пути 2. Поскольку коэффициент трения на обоих путях одинаков, то сила трения на пути 1 меньше, чем на пути 2, т. е. работа, совершенная против сил трения, будет меньше на пути 1. Таким образом, в точке В тело обладает большей кинетической энергией, а значит, и большей скоростью, если оно двигалось по пути 1, а не 2

Задача № 2.32. Два одинаковых невесомых блока с параллельными осями установлены на одной и той же высоте. Через оба блока перекинута нерастяжимая и невесомая нить, на концах которой висят два груза одинаковой массы (рис. 26). Имеет место равновесие. Один из грузов отводят в сторону и отпускают. Нарушится ли при этом равновесие?

Решение:

Равновесие нарушится, так как сила натяжения нити, на которой находится колеблющийся груз, не может оставаться постоянной и равной силе тяжести груза. В крайних точках, где скорость груза равна нулю, сила натяжения нити будет равна составляющей силы тяжести груза в направлении нити, т. е. будет меньше силы тяжести груза. В средней точке сила натяжения нити должна не только уравновешивать силу тяжести груза, но и сообщать ему нужное центростремительное ускорение вверх, т. е. сила натяжения должна быть больше силы тяжести груза. Поэтому при колебаниях правого груза левый не может оставаться в покое, и движение приобретает более сложный характер, чем колебания маятника, подвешенного к неподвижной точке. Опыт и расчет показывают, что колеблющийся груз опускается.

Задача № 2.33. Шофер автомобиля, едущего со скоростью v, внезапно увидел перед собой на расстоянии s широкую стену. Что ему выгоднее: затормозить или повернуть?

Решение:

Если шофер затормозит, автомобиль остановится, когда его кинетическая энергия израсходуется на работу против силы трения. При повороте автомобиля та же сила трения будет играть роль центростремительной силы, заставляющей автомобиль двигаться по дуге окружности. В случае торможения mv2/2 = fx, где f — сила трения, х — путь, который пройдет автомобиль после включения тормоза. Отсюда х = mv2/2f. Очевидно, чтобы автомобиль не врезался в стену, должно быть

Решение задач по физике  или    Решение задач по физике

В случае поворотаРешение задач по физике и чтобы автомобиль не разбился, должно быть Решение задач по физике или   Решение задач по физике  Для того чтобы избежать столкновения со стенкой, при торможении нужна сила трения, вдвое меньшая, чем при повороте. Очевидно, выгоднее тормозить, чем поворачивать.

Статика

Задача № 43. Определите, где находится центр тяжести однородной пластинки с вырезом. Все размеры в сантиметрах указаны на рис. 27, а. 

Решение:
Если пластинка не имеет выреза, то ее центр тяжести расположен в точке Решение задач по физике Пусть Решение задач по физике —вес 
пластинки с вырезом, а Решение задач по физике—вес вырезанной части. Вес всей пластинки Решение задач по физике можно рассматривать как сумму весов Решение задач по физике Таким образом, задача сводится к нахождению точки приложения двух антипараллельных сил Решение задач по физике(рис 27, б). 
Пользуясь рисунком, можно  написать, что 
Решение задач по физике
Вес однородной пластинки пропорционален ее площади, поэтому 
Решение задач по физике 

Решение задач по физике
Рис. 27 

откуда Решение задач по физике

Задача № 44. Из однородной круглой пластинки радиусом Решение задач по физике вырезан круг вдвое меньшего радиуса, касающийся края первого круга. Найти центр тяжести полученной пластинки (рис. 28). 

Решение задач по физике

Рис.28.


Дано: 
Решение задач по физике

Решение:
Пусть вес большого круга Решение задач по физике а вес вырезанной части Решение задач по физике Тогда вес оставшейся части большого круга будет равен Решение задач по физике 
Так как Решение задач по физике a Решение задач по физикето отсюда следует, что Решение задач по физике Следовательно, Решение задач по физикеРешение задач по физике
Центр тяжести полученной пластинки найдем из условия равенства моментов: 

Решение задач по физике
откуда 
Решение задач по физике 
 

Задача № 45. Выкладывая карниз из камня, каменщик кладет один на другой четыре кирпича так, что часть вышележащего кирпича выступает над нижележащим (рис. 29, а). Длина каждого кирпича Решение задач по физике
Решение задач по физике 
Рис.29. 


Определить наибольшие длины выступающих частей кирпичей, при которых кирпичи в карнизе будут без цементного раствора еще находиться в равновесии. 

Решение:
Так как кирпичи однородны, то центр тяжести каждого кирпича находится на середине его длины. Вследствие этого самый верхний кирпич будет находиться в равновесии по отношению к лежащему под ним, если его центр тяжести лежит на продолжении линии среза второго кирпича, т. е. наибольшая длина свеса первого кирпича Решение задач по физике 
Центр тяжести первого и второго кирпичей, взятых вместе, будет расположен на расстоянии-Решение задач по физике от внешнего края второго кирпича. На эту длину и можно свесить второй кирпич, чтобы он 
и первый кирпич еще находились в равновесии по отношению к третьему кирпичу. 
Относительно правого верхнего края самого нижнего кирпича два самых верхних кирпича 
будут создавать вращательный момент Решение задач по физике (рис. 29, б). Третий кирпич от верха будет  
создавать вращательный момент противоположного знака, равный Решение задач по физике Условие равновесия будет выполнено, если Решение задач по физике Отсюда Решение задач по физике т. е. третий кирпич может выступать над четвертым не более чем на Решение задач по физике своей длины. 

Задача № 46. Человек весом Решение задач по физике стоит на балке весом Решение задач по физике подвешенной на блоках (рис. 30). Длина балки между точками опоры Решение задач по физике Определить, какую силу должен приложить человек и в каком месте он должен встать, чтобы балка находилась в равновесии и занимала  горизонтальное положение. 
Дано: 
Решение задач по физике

Решение задач по физике
Рис. 30 

Решение:
Пусть Решение задач по физике — сила, с которой человек действует на веревку. Тогда сила, действующая на балку в точке Решение задач по физике должна быть также Решение задач по физике а в точке Решение задач по физике В точке Решение задач по физике на балку действует человек с силой Решение задач по физике Сумма всех сил, действующих на балку в положении равновесия, должна быть равна пулю. 
Отсюда 
Решение задач по физике
или 
Решение задач по физике
Место человека на балке определяем из правила моментов 
Решение задач по физике
Принимая во внимание, что 
Решение задач по физике находим 
Решение задач по физике

Задача № 47. В некоторой точке Земли магнитная стрелка, вращающаяся вокруг горизонтальной оси, установилась под углом Решение задач по физике к горизонту (рис. 31, а).

Решение задач по физике

Рис.31.

Если к верхнему концу стрелки прикрепить гирьку массой в Решение задач по физике то угол наклона уменьшится до Решение задач по физике (рис. 31,6). Какую гирьку надо прикрепить к стрелке, чтобы она заняла горизонтальное положение?  
Дано: 
Решение задач по физике

Решение:
Так как стрелка находится в равновесии, моменты сил, действующие на нее, равны. Напишем равенства, когда стрелка наклонена под углом Решение задач по физике к горизонту (рис. 31, б): 
Решение задач по физике
и когда она расположена горизонтально (рис. 31, в): 

Решение задач по физике
где Решение задач по физике — сила магнитного взаимодействия стрелки с магнитным полем Земли; Решение задач по физике — длина стрелки. Исключая силу Решение задач по физике получим 

Решение задач по физике
откуда 
Решение задач по физике

Задача № 48. На деревянном полу стоит маленькая лестница. Посредине она связана веревкой. Веревка разрывается при действии на нее силы Решение задач по физике Определить, при каком угле при вершине лестницы веревка разорвется, если на верхней ступени лестницы стоит человек весом Решение задач по физике (коэффициент трения дерева о дерево 0,65). Вес лестницы не учитывать. 
Дано: 
Решение задач по физике 

50. Три одинаковых цилиндра уложены, как показано на рис. 34. При каких условиях они будут удерживаться в этом  положении неподвижно? 

Решение:
Все три цилиндра останутся неподвижными в положении, приведенном на рис. 34, если цилиндры 2 и 3 не будут ни катиться, ни скользить по плоскости опоры. 

Решение задач по физике
Рис.34. 


Чтобы любой из нижних цилиндров не катился по плоскости опоры, необходимо следующее: сумма моментов действующих на цилиндр сил относительно оси опоры Решение задач по физике должна быть равна нулю. Применительно к цилиндру 2> например, это условие записывается 
так: 
Решение задач по физике
Отсюда 
Решение задач по физике
Таким образом, цилиндры 2 я 3 не будут катиться по плоскости опоры, если коэффициент трения k между цилиндрами не меньшеРешение задач по физике это является вместе с тем условием отсутствия скольжения между цилиндром 1, с одной стороны, и цилиндрами 2 и 5 с другой. 
Если неравенство Решение задач по физике выполняется, то цилиндры 2 и 3 (чтобы не было их скольжения по плоскости опоры) не должны совершать поворота относительно линий соприкосновения с цилиндром Решение задач по физике Для этого сумма моментов сил, действующих на цилиндр 2 
или 3 относительно осей Решение задач по физике (соответственно), должна равняться нулю. Применительно к цилиндру 3 например, это условие записывается так: 

Решение задач по физике
Отсюда 
Решение задач по физике
Это значит, что цилиндры 2 я 3 не будут скользить по плоскости опоры, если коэффициент трения Решение задач по физике между этими цилиндрами, с одной стороны, и плоскостью опоры, с другой, не меньше Решение задач по физике 
Итак, все три цилиндра будут неподвижно удерживаться в положении, указанном на рис. 34, если коэффициенты трения покоя между цилиндром 1, с одной стороны, и цилиндрами 2 и 3, с другой, а также между цилиндрами 2 и 3 и плоскостью опоры не менее чемРешение задач по физике

Момент силы. Правило моментов

Задача № 3.1. Может ли человек, стоящий у стены так, что его правая нога и правое плечо упираются в стену (рис. 27), поднять левую ногу и не потерять при этом равновесия?

Решение задач по физике

Рис. 27

Решение:

Нет. Человек может поднять левую ногу и не потерять при этом равновесия лишь в том случае, если проходящая через его центр масс вертикальная линия будет проходить также через ступню правой ноги. При описанном положении это невозможно.

Задача № 3.2. В каких случаях герои известной басни Крылова лебедь, щука и рак действительно не сдвинут воза, если считать что силы их равны по модулю? Силу трения между возом и землей не учитывать.

Решение:

1-й случай: все три равные по модулю силы расположены в одной плоскости и составляют друг с другом углы в 120°, равнодействующая их равна нулю (рис. 183, а). 2-й случай: рак и щука тянут воз в противоположные стороны, а лебедь — вертикально вверх (рис. 183, б), причем подъемная сила лебедя меньше силы тяжести воза (последнее условие, однако, в басне не оговорено).

Решение задач по физике

Рис. 183

Задача № 3.3. Для того чтобы отвести штангу троллейбуса от провода, водитель прежде отдергивает как можно дальше назад веревку, привязанную к кольцу, одетому на штангу. Зачем?

Решение:

Это делается для того, чтобы создать возможно большее плечо прилагаемой силы: чем больше плечо, тем меньшую силу надо приложить, чтобы преодолеть действие тугой пружины, поднимающей штангу и прижимающей ее к проводу (рис. 184).

Решение задач по физике

Рис. 184

Задача № 3.4. Лампа висит na кронштейне, состоящем из трех стержней, одни концы которых укреплены в стене (В, С, D), а другие сходятся в одной точке (А). Два верхних  стержня образуют равнобедренный треугольник с углом Решение задач по физике = 60° между стержнями. Плоскость этого треугольника перпендикулярна к третьему стержню АВ, который образует со стеной угол Решение задач по физике = 30° (рис. 28). Масса лампы с абажуром m = 1 кг. Найти силы, растягивающие и сдавливающие стержни. 

Решение задач по физике                   Решение задач по физике    

Рис. 28                                                                       Рис. 29

Решение:

Разложим силу тяжести лампы с абажуром по двум взаимно перпендикулярным направлениям: F1 — по направлению продолжения высоты АЕ равнобедренного треугольника ACD и F2 — по направлению стержня АВ (рис. 185, а). Так как стержень АВ образует угол Решение задач по физике = 30° с направлением подвеса лампы, то легко находим, что равнодействующая сила, растягивающая верхние стержни АС и AD, равна

Решение задач по физике

а силa, сдавливающая стержень АВ, равна

Решение задач по физике

Далее, разлагаем силу F1 = 4,9 Н по направлениям стержней АС и AD (рис. 185, 6).

Решение задач по физике

Рис. 185

В данном случае параллелограмм сил будет ромбом, в котором, как известно, диагонали взaимно перпендикулярны, и так как, кроме того, угол при вершине А ромба равен Решение задач по физике = 60°, то легко найдем силу, растягивающую стержни АС и AD:

Решение задач по физике

Задача № 3.5. Груз массой m закреплен и подвешен на двух веревках АВ  и АС одинаковой длины (рис. 29). В каком случае веревки легче порвутся: когда они свисают или когда они натянуты почти горизонтально? Массу веревок не учитывать.

Решение:

Веревки рвутся тем легче, чем ближе они к горизонтальному направлению. Для объяснения рассмотрим силу натяжения веревки. Вдоль каждой из веревок действуют силы натяжения, равные Т. Эти две силы и сила тяжести груза mg находятся в равновесии, т. е. равнодействующая R сил натяжения равна по модулю и противоположна по направлению силе mg (рис. 186). Если угол Решение задач по физике между силами натяжения составляет 120°, то треугольник сил будет равнобедренным и сила натяжения веревки будет равна силе тяжести груза. Если же этот угол меньше 120°, то каждая из сил натяжения меньше силы тяжести груза и становится тем меньше, чем больше приближаются силы натяжения Т к направлению силы mg. Наоборот, чем больше приближаются силы натяжения Т к горизонтальному направлению, тем больше становятся их значения. Когда веревки принимают почти горизонтальное положение, силы натяжения становятся очень большими и веревки легко рвутся.

Решение задач по физике

Рис. 186

Задача № 3.6. Чтобы сдвинуть с места застрявший автомобиль, иногда пользуются таким приемом: автомобиль привязывают длинной веревкой к дереву, по возможности сильно ее натянув. Затем, натягивая веревку посредине почти перпендикулярно к ее направлению, человек легко сдвигает автомобиль с места. Почему это возможно?

Решение:

Решение этой задачи аналогично решению предыдущей. Если веревка уже натянута, то сила тяжести mg, действующая перпендикулярно к веревке, может создать очень большое натяжение последней. Это видно из того, что сила тяжести mg должна быть равна сумме сил натяжений Т двух концов веревки, сходящихся в точке приложения силы тяжести (рис. 187). И если угол Решение задач по физике близок к 180°, то сила натяжения Т во много раз превосходит силу тяжести mg, с которой человек действует на веревку.

Решение задач по физике

Рис. 187

Задача № 3.7. Гимнаст, идущий по натянутому канату, вызывает восхищение зрителей. Еще более искусным кажется он, когда идет по натянутому канату и несет на коромысле ведра с водой. В каком случае ему легче удержать равновесие?

Решение:

Гимнасту, идущему по канату, легче удержать равновесие, когда он несет на коромысле ведра с водой, так как центр масс гимнаста с ведрами лежит ниже, т. е. ближе к канату. Конечно, гимнаст должен идти по канату так, чтобы не раскачивать ведер на коромысле. Если ведра сильно раскачиваются, то положение центра масс будет отклоняться от вертикали, проходящей через канат.

Задача № 3.8. Показать, пользуясь разложением сил по правилу параллелограмма, что «клин клином вышибают» (рис. 30).

Решение задач по физике

Решение:

Силу F, действующую на клин 1, переносим вдоль ее направления в точку А, где разлагаем на силу F1, перпендикулярную х боковой поверхности клина, и силу F2, параллельную основанию клина (рис. 188). Сила F1 вызывает равную по модулю силу реакции бревна. Сила F2 действует на ранее вбитый клин 2 и вызывает силу реакции с его стороны. Силу F2 переносим в точку В, где клин 2 упирается в бревно. Разлагаем cилy F2 на силу F3, перпендикулярную к боковой поверхности клина, и силу F4, перпендикулярную к основанию клина. Сила Fвызывает равную по модулю силу реакции со стороны бревна, сила F4 выталкивает клин 2.

Решение задач по физике

Рис. 188

Задача № 3.9. Два рычага находятся в равновесии (рис. 31). На первом уравновешены два груза разной массы (m2 = 3m1) из одного материала, на втором — два груза разной массы (m2 = 3m1), но одинаковых по объему. Нарушится ли равновесие рычагов, если погрузить их в воду?

Решение:

На первом рычаге грузы имеют объемы, пропорциональные их массам. При погружении грузов в воду силы, действующие на плечи рычага, изменятся (по закону Архимеда) пропорционально объемам. Поэтому относительное изменение сил, действующих на рычаг, будет одно и то же для обоих концов рычага. Следовательно, под водой равновесие не нарушится.

На втором рычаге оба груза «потеряют в весе» под водой одинаково, а так как их массы разные, то относительная «потеря веса» будет больше для груза, у которого масса меньше. Отношение сил, действующих на плечи рычага, изменится, и равновесие нарушится. Опустится тот конец рычага, для которого относительная «потеря веса» меньше, т. е. перетянет груз m2.

Задача № 3.10. Уравновешенные весы со стальным коромыслом располагаются вдоль магнитного меридиана. Сохранится ли равновесие, если коромысло намагничивается вдоль его длины?

Решение:

Вообще говоря нет, так как в каждой точке земного шара имеется некоторое магнитное наклонение (направление напряженности магнитного поля Земли не горизонтально).

Задача № 3.11. При легкой перегрузке одной из чашек весов коромысло лишь немного наклоняется в сторону большего груза и в этом положении остается в равновесии. Почему наступает равновесие несмотря на то, что массы грузов различны (m1 > m2)?

Решение задач по физике

Рис. 30                                                                 Рис. 31

Решение задач по физике

Рис. 32                                                 Рис. 33                                                  Рис. 34 

Решение задач по физике

Рис. 35                                                           Рис. 36 

Решение:

Момент силы тяжести большего груза всегда остается больше m1g момента силы тяжести меньшего груза, так как плечи сил m1g и m2g остаются равными при любом положении коромысла (рис. 189). Однако при смещении коромысла меняется положение центра масс коромысла и стрелки весов: из точки О он перемещается в точку О', и возникает дополнительный момент М0 силы тяжести коромысла и стрелки весов, который добавляется к моменту силы тяжести m2g. Положение равновесия qпределяется условием равенства моментов сил: момент силы m1g равен сумме момента силы m2g и момента М0.

Решение задач по физике

Рис. 189

Задача № 3.12. В системе, состоящей из неподвижного и подвижного блоков, грузы массами m1 и m2, висящие на блоках, находятся в равновесии, когда нити параллельны. Что произойдет, если точку закрепления нити А передвигать вправо (рис. 32)? Массами блоков пренебречь.

Решение:

Когда участки нити АВ, CD и EF параллельны (рис. 190), то при равновесии сила натяжения нити должна быть равна половине силы тяжести груза массы m1 и, следовательно, масса груза m2 должна быть вдвое меньше массы груза m1. Если затем точка закрепления нити А перемешается вправо в точку А', то соответствующие участки нити уже не параллельны и при силе натяжения, равной половине силы тяжести груза массы m1, нити уже не смогут удержать блок с грузом m1. Следовательно, равновесие нарушается, груз m1 опускается, а груз m2 поднимается.

Решение задач по физике                  Решение задач по физике

Рис. 190                                                                                  Рис. 191

Решение:

Задача № 3.13. С какой силой Т должен человек тянуть веревку, чтобы удержать доску в равновесии, если масса человека = 61,3 кг (рис. 33)? Массами доски, блока и веревки пренебречь. Считать ускорение свободного падения g = 9,8 м/с2.

Решение:

Пусть человек тянет за веревку а с силой Т, выраженной в ньютонах (рис. 191). Тогда сила натяжения веревки b тоже будет Т. Сила натяжения веревки с уравновешивает совокупное действие двух параллельных сил Т и Т, следовательно, она равна . Такова же должна быть и сила натяжения веревки d, являющейся продолжением веревки с, т. е. . Доска висит на двух веревках: b и d (веревка а не прикреплена к доске, а потому не поддерживает ее). Равнодействующая этих параллельных сил равна ЗТ. Кроме этой силы, направленной вверх, на доску действует сила mg - Т, где mg — сила давления человека на доску, Т — сила натяжения веревки а. Так как доска находится в равновесии, сумма всех сил, действующих на доску, равна нулю: Решение задач по физике отсюда Т = 150 Н.

Задача № 3.14. Чему равна горизонтальная сила F, действующая на маленький блок В (рис. 34), если участок нити ВС вертикален, когда к блоку подвешен груз массой m, а участок нити АВ образует с горизонталью АС угол Решение задач по физике = 60°?

Решение:

На блок В действуют четыре силы (рис. 192): сила тяжести груза mg, горизонтальная сила F и две силы натяжения нити Т по обе стороны блока (они одинаковы по модулю). Так как блок находится в равновесии, то сумма проекций всех сил на любое направление равна нулю. Спроецируем силы на горизонтальное направление: F = Tcos 60° = 0; спроецируем силы на вертикальное направление:   mg – Т – Т cos 30° = 0. Из этих уравнений получаем, что горизонтальная сила Решение задач по физике.

Решение задач по физике

Рис. 192

Задача № 3.15. Однородная тонкая пластинка радиусом R имеет форму круга, в котором вырезано круглое отверстие вдвое меньшего радиуса, касающееся края пластинки (рис. 35). Масса сплошной пластинки равна m. Где находится центр масс круга с таким отверстием?

Решение:

Представим себе, что в пластинке сделано не одно, а два одинаковых отверстия, как показано на рис. 193. Тогда центр масс пластинки будет лежать в центре круга О, масса ее будет равна m/2 (так как масса каждой из удаленных частей пластинки равна m/4). Заполним теперь левое отверстие. При этом прибавится масса, равная m/4, и сила тяжести, приложенная в центре заполненного отверстия, т. е. в точке С2, лежащей на расстоянии R/2 от центра пластинки.

Решение задач по физике

Рис. 193

Точка приложения (т. е. центр масс), равнодействующей сил тяжести заполненного левого круга и фигуры, заштрихованной на рис. 193, лежит между точками С2 и О. Пусть ее расстояние от точки О равно х. Тогда (m/2) х = (m/4) (R/2 - х), откуда х = R/6. Этот же результат можно получить иначе. Заполним отверстие кружком и приложим в его центре С1 силу, направленную вертикально вверх и равную силе тяжести этого кружка mg/4. Тогда будем иметь две антипараллельные силы: силу тяжести всей пластинки mg, направленную вниз и приложенную в точке О, и силу тяжести mg/4, направленную вверх и приложенную в точке С1. Точка приложения их равнодействующей, т. е. центр масс, находится на прямой ОС1 влево от О; пусть х — расстояние центра масс от точки О. Тогда mх = (т/4) (х + R/2), х = R/6.

Задача № 3.16. Тонкий невесомый стержень проходит через центры трех шаров разных масс: m1, m2 и m3. Центры масс всех трех шаров отстоят от левого конца стержня на расстояния х1, х2 и х3 соответственно (рис. 36). Ha каком расстоянии х0 от того же конца стержня находится центр масс системы всех трех шаров?

Решение:

Найдем сначала положение центра масс первых двух шаров. Очевидно, что эта точка делит расстояние х2 – х1 на части, обратно пропорциональные массам т 1 и т2 • Обозначим ее расстояние от конца стержня через х (рис. 194).

Решение задач по физике

Рис. 194

Тогда будем иметь (х – х1)/(х2 – х) = т21. Отсюда получим х = (т1 х1 + тх2)/(т1 + т2). В этой точке действует сумма сил тяжести первых двух шаров, которую обозначим через тg. Тогда, применяя полученный нами результат к массам т и т3 , находящимся от конца стержня на расстояниях х и х3 , найдем, что центр масс системы всех трех шаров отстоит от конца стержня на расстояние Решение задач по физике

подставляя в эту формулу значения х и т, получим, что Решение задач по физике

или, вообще, при любом числе n шаров Решение задач по физике

Задача № 3.17. На наклонной плоскости с углом наклона Решение задач по физике = 30° лежит цилиндр массой m. Цилиндр удерживается в состоянии покоя с помощью огибающей его невесомой нити (рис. 37), один конец которой закреплен на наклонной плоскости, а другой натянут вертикально вверх силой Т. Чему равна сила Т?

Решение:

Так как цилиндр находится в состоянии покоя, то сумма проекций всех сил, действующих на цилиндр, на любое направление должна равняться нулю. Спроецируем все cилы на вертикальное направление (рис. 195): Решение задач по физике

Спроецируем все силы на горизонтальное направление: Решение задач по физике Из этих двух уравнений легко находим силу натяжения нити, удерживающую цилиндр: Т= mg/3.

Задача № 3.18. Колесо радиусом R и массой m стоит перед ступенькой высотой h (рис. 38). Какую минимальную горизонтальную силу F нaдo приложить к оси колеса О, чтобы оно могло подняться на ступеньку? Силу трения не учитывать.

Решение задач по физике

Рис. 37                                                                   Рис. 38

Решение:

При подъеме на ступеньку колесо должно вращаться вокруг точки А (рис. 196). Для этого нужно, чтобы момент силы F относительно точки А был больше или равен моменту силы тяжести колеса относительно той же точки.

Решение задач по физике

Рис. 195 Рис. 196

Опуская перпендикуляры из точки А на направления силы тяжести mg и силы F, найдем плечи АВ и АС этих сил. Для того чтобы колесо вкатилось на ступеньку, должно выполняться неравенство Решение задач по физике Так как АС = R – h,Решение задач по физике получаем окончательно Решение задач по физике если Решение задач по физике то Решение задач по физике.

Равновесие тел

Задача № 3.19. В случаях а и б, изображенных на рис. 39, масса m1 груза подобрана так, что масса m2 шарика, опирающегося на гладкую поверхность, находится в равновесии. В каком из этих случаев равновесие устойчивое и в каком — неустойчивое?

Решение задач по физике

Рис. 39

Для упрощения будем предполагать, что блок расположен достаточно далеко, и поэтому направление нити, идущей от массы m2 , совпадает с направлением касательной к поверхности.

Решение:

Равновесие в обоих случаях имеет место тогда, когда составляющая силы тяжести шарика массы m2 в направлении нити равна силе натяжения нити Т, т. е. силе тяжести шарика m1g (рис. 197). Для того чтобы решить вопрос, устойчиво ли это состояние равновесия, нужно рассмотреть, как изменяются силы при небольшом отклонении массы m2 от положения равновесия. Тогда в случае а при смещении массы m2 книзу от положения равновесия наклон касательной увеличится, а значит, увеличится и составляющая силы тяжести в направлении нити. Она станет больше силы натяжения нити Т, и масса m2 будет опускаться еще дальше. Наоборот, если массу m2 сместить немного вверх, наклон касательной уменьшится, составляющая силы тяжести m2g в направлении нити станет меньше силы натяжения нити Т, и груз будет подыматься дальше. Следовательно, масса m2, немного отклоненная от положения равновесия, не будет к нему возвращаться, а будет от него уходить, т. е. равновесие неустойчиво.

Решение задач по физике

Рис. 197

В случае б, если мы сместим массу m2 из положения равновесия вниз, то наклон касательной уменьшится, значит, уменьшится и составляющая силы тяжести m2g в направлении нити; она станет меньше силы натяжения нити Т, и нить потянет массу m2 назад, к прежнему положению равновесия. Такие же рассуждения убедят нас в том, что при смещении массы m2 вверх от положения равновесия она снова возвратится к нему, т. е. равновесие в этом случае устойчивое.  

Задача № 3.20. В серванте имеется выдвижная доска для резки хлеба на ней. К доске спереди приделаны для удобства выдвижения симметрично относительно середины две ручки на расстоянии l друг от друга (рис. 40). Длина доски (в глубь серванта) равна L. При каком наименьшем значении коэффициента трения k между боком доски и стенкой серванта нельзя вытащить доску как бы ни была велика приложенная сила F, действующая на одну из ручек?

Решение задач по физике

Рис. 40

Решение:

Задача № 3.20. Пусть сила F приложена к левой ручке доски. Она вызывает в точках А и В силы реакции стенок серванта (рис. 198), каждую из которых можно разложить на две составляющие: N1 и N2, нормальные к стенкам серванта, и f1 и f2, касательные к тем же стенкам (силы трения). Предполагая, что доску вытащить нельзя, мы должны иметь следующие равенства. Сила F должна быть равна сумме сил трения, чтобы не было поступательного движения доски, т. е. F = f1 + f2.

Решение задач по физике

Рис. 198

Момент силы F относительно центра масс доски должен быть равен сумме моментов нормальных составляющих сил реакции относительно того же центра масс доски (чтобы не было вращения доски), т. е. Fl/2 = (N1 + N2)L/2. Кроме того, по определению имеем f1/N1 =f2/N2 = k. Исключая из обоих уравнений силу F, получаем, что наименьшее значение коэффициента трения должно равняться L/l. При большем его значении вытащить доску из серванта, действуя на одну из ручек, невозможно.

Работа и энергия

Задача № 51. Можно ли совершить Решение задач по физике работы при однократном подъеме килограммовой гири на высоту Решение задач по физике

Решение:
Можно, прилагая к гире силу Решение задач по физике т. е. поднимая ее с ускорением; при этом Решение задач по физике работы пойдет на пополнение запаса потенциальной энергии, а остальные Решение задач по физике перейдут в кинетическую энергию. 

Задача № 52. Для откачки нефти из скважины глубиной Решение задач по физике поставлен насос мощностью Решение задач по физике Коэффициент полезного действия насоса Решение задач по физике Какова добыча нефти за 5 часов работы насоса? 

Дано: 
Решение задач по физике

Решение:
Вес нефти можно найти из формулы работы. Полезная работа 
Решение задач по физике
Затраченная работа 
Решение задач по физике
Тогда 
Решение задач по физике
Подставив численные значения, получим 
Решение задач по физике

Задача № 53. Продолжительность свободного падения тела вдоль наклонной плоскости, образующей с горизонтом угол Решение задач по физике равна Решение задач по физике
Как надо изменить угол наклона плоскости (оставив высоту неизменной), чтобы увеличить продолжительность падения в 2 раза? 
Как при этом изменится величина скорости в конце длины наклонной плоскости? Как изменится величина горизонтальной составляющей скорости? 
Рассмотреть случаи идеально гладкой плоскости, пренебрегая сопротивлением среды. 

Решение:
Решение задачи необходимо начинать со второго вопроса. Для идеально гладкой плоскости, если пренебречь сопротивлением среды, на основании закона сохранения и превращения механической энергии скорости в конце длины наклонной плоскости при одной и той же высоте в обоих случаях будут одинаковы (потенциальная энергия полностью превращается в кинетическую), т. е. 
Решение задач по физике
Выражая далее эти скорости по формуле Решение задач по физике и принимая ускорение при движении по наклонной плоскости без учета трения равным Решение задач по физике получим 

Решение задач по физике
где по условию Решение задач по физике
Тогда 
Решение задач по физике
Решение этого вопроса может быть иное ввиду отсутствия у учащихся соответствующих знаний по математике: 
Решение задач по физике
откуда 
Решение задач по физике
Физический смысл полученного ответа следующий; при одной и той же высоте наклонной плоскости и при большей ее длине угол наклона меньше, а продолжительность свободного падения тела больше. 
Ответ на последний вопрос задачи получаем, разлагая скорость движения на горизонтальную и вертикальную составляющие (рис. 35): 
Решение задач по физике

Решение задач по физике

Рис.35.

Задача № 54. Вагон под действием некоторой силы движется с постоянной скоростью Решение задач по физике по горизонтальному пути с коэффициентом трения Решение задач по физике Затем вагон переходит на участок пути с  коэффициентом трения Решение задач по физике Какой путь пройдет вагон до остановки? 
Дано: 
Решение задач по физике

Решение:
На участке пути с коэффициентом трения Решение задач по физике сила Решение задач по физике  действующая на вагон, уравновешивается силой трения, т. е. 
Решение задач по физике
Запишем закон сохранения энергии для второго участка пути; 
Решение задач по физике 
Конечная скорость вагона Решение задач по физике Знак минус перед Решение задач по физике означает, что угол между силой трения и направлением движения Решение задач по физике a Решение задач по физике
Таким образом, 
Решение задач по физике
55. Из винтовки сделано в горизонтальном направлении два выстрела в щит, находящийся на расстоянии Решение задач по физике. После первого выстрела перед дулом винтовки поставили доску. Вторая пуля, пробив доску, попала в щит на Решение задач по физике ниже первой. Какая работа совершена пулей при пробивании доски, если начальная скорость пули Решение задач по физике Масса пули Решение задач по физике

Дано: 
Решение задач по физике

Решение:
Работа, совершенная пулей при пробивании доски (рис. 36), равна разности кинетических энергий пули до пробивания и после пробивания доски: 
Решение задач по физике
где Решение задач по физике — масса пули; Решение задач по физике — скорость пули после пробивания доски. 
Используя формулу равноускоренного движения 
Решение задач по физике
определим время полета пули между доской и щитом 

Решение задач по физике
Скорость пули 
Решение задач по физике

Решение задач по физике
Рис. 36 


Подставив полученное значение v2 в формулу для определения работы, получим 
Решение задач по физике

Задача № 56. Автомобиль движется с постоянной скоростью Решение задач по физике У подножья горы водитель выключил мотор. Наклон горы Решение задач по физике на Решение задач по физике пути. Коэффициент трения Решение задач по физике На какое расстояние автомобиль поднимется в гору? 
Дано: 
Решение задач по физике

Решение:
У подножья горы автомобиль обладает запасом кинетической энергии Решение задач по физике которая пойдет на сообщение автомобилю потенциальной энергии при подъеме в гору Решение задач по физике и па преодоление сил  трения Решение задач по физике (рис. 37, а), т. е. 

Решение задач по физике

Рис. 37 

Решение задач по физике

гдеРешение задач по физике
Таким образом, 

Решение задач по физике
откуда 
Решение задач по физике

Задача № 57. Санки скользят с высоты Решение задач по физике по наклонной плоскости Решение задач по физике под углом Решение задач по физике к горизонту. Пройдя Решение задач по физике по горизонтальной плоскости, санки поднимаются в гору по плоскости с углом наклона Решение задач по физике горизонту. Определить, на какой высоте остановятся санки, если известно, что коэффициент сопротивления на всем пути одинаков и равен Решение задач по физике (см. рис. 37, б). 
Дано: 
Решение задач по физике

Решение:
Начальная и конечная скорость салок равна пулю. Следовательно, работа всех сил на пути Решение задач по физике равна нулю. На участке пути Решение задач по физике на санки действует скатывающая сила и сила трения, а на участке Решение задач по физике — только сила трения. Таким образом, 
Решение задач по физике 
где 
Решение задач по физике
Подставляя вместо Решение задач по физике их значения и сокращая на Решение задач по физике получим 
Решение задач по физике

Задача № 58. Какой максимальный подъем может преодолеть паровоз мощностью Решение задач по физике двигая состав со скоростью Решение задач по физике если коэффициент трения равен Решение задач по физике Масса состава Решение задач по физике
Дано: 
Решение задач по физике

Решение:
Сила тяги паровоза Решение задач по физике при равномерном движении должна уравновешивать скатывающую силу и силу трения, поэтому 
Решение задач по физике
Откуда 
Решение задач по физике
Для малых углов можно принять Решение задач по физике 
Тогда 
Решение задач по физике

Задача № 59. В стену вбиты два гвоздя, один под другим на расстоянии Решение задач по физике К верхнему гвоздю подвешен математический маятник длиной Решение задач по физике (рис. 38). Маятник отклоняют до горизонтального 
положения и отпускают  (начальная скорость равна 0). Достигнет ли маятник верхней точки? 

Решение задач по физике

Рис.38.

Решение:

Рассмотрим, какие условия должны быть для того, чтобы шарик достиг верхней точки, двигаясь по окружности радиусом Решение задач по физике. При движении шарика по дуге радиусом Решение задач по физике снизу вверх его  
линейная скорость уменьшается вследствие убывания кинетической энергии, при этом   центростремительная сила тоже убывает. В верхней точке центростремительная сила должна быть не меньше веса шарика, иначе шарик не достигнет этой точки. Следовательно, это 
условие можно записать так: 
Решение задач по физике
Пусть Решение задач по физике так как Решение задач по физике а Решение задач по физикеоткуда Решение задач по физике Кинетическая энергия шарика в верхней точке будет Решение задач по физике  или Решение задач по физике Такую энергию шарик может иметь в верхней точке только в том случае, если его потенциальная энергия в  
начальный момент будет 

Решение задач по физике

что могло быть только в том случае, если бы он был поднят на высоту Решение задач по физике
Следовательно, шарик, имеющий в начальный момент потенциальную энергию, равную Решение задач по физике не поднимется на высоту Решение задач по физикеи не попадет в верхнюю точку. 

Задача № 60. При ходьбе на лыжах на дистанцию в Решение задач по физике по горизонтальному пути происходят гармонические колебания центра тяжести спортсмена с вертикальной амплитудой Решение задач по физике и периодом Решение задач по физике Масса лыжника — Решение задач по физике Коэффициент трения лыж о снег равен Решение задач по физике Работа, которую затрачивают мышцы лыжника, чтобы затормозить опускание центра тяжести, составляет Решение задач по физике от работы, которая производится при подъеме центра тяжести на ту лее  высоту. Определить работу лыжника на марше, если всю дистанцию он прошел за Решение задач по физике а также среднюю мощность лыжника. 
Дано: 
Решение задач по физике

Решение:
На дистанции s произошло Решение задач по физике колебаний центра тяжести лыжника. При каждом колебании центр тяжести опускается и поднимается на высоту Решение задач по физике так как амплитуда колебаний равна Решение задач по физикеСледовательно, работа, затрачиваемая лыжником на поднятие- центра 
тяжести и на торможение его при опускании, 
Решение задач по физике
Работа, затрачиваемая лыжником на преодоление силы трения, 
Решение задач по физике
Таким образом, 
Решение задач по физике 

Средняя мощность равна:

Решение задач по физике

Задача № 61. Невесомый стержень, длина которого Решение задач по физике может вращаться вокруг неподвижной точки Решение задач по физике (рис. 39, а). На концах стержня, на расстояниях Решение задач по физике укреплены два груза соответственно с массами Решение задач по физике и Решение задач по физике В начальный момент времени стержень расположен горизонтально. Затем конец стержня, на котором укреплен груз с массой Решение задач по физике начинает опускаться. Какую скорость будет иметь этот груз в самом низком положении? 

Решение задач по физике

Рис.39

Дано: 
Решение задач по физике

Решение:
Потенциальную энергию условимся отсчитывать от уровня Решение задач по физике (рис. 39, б). Тогда закон сохранения энергии запишется так: 
Решение задач по физике
Так как линейные скорости пропорциональны расстояниям до оси вращения, то 

Решение задач по физике
Заменив в первом равенстве Решение задач по физике получим 
Решение задач по физике
Откуда 
Решение задач по физике
Подставив численные значения, найдем, что 
Решение задач по физике

Задача № 62. К концам нити, перекинутой через два блока, подвешены одинаковые грузы (рис. 40). Такой же груз подвешен к середине отрезка Решение задач по физике этой нити (точка Решение задач по физике) и удерживается на таком уровне, 

Решение задач по физике

Рис.40.
что отрезок нити Решение задач по физике является прямолинейным. Расстояние между блоками Решение задач по физике На сколько опустится средний груз, если нить Решение задач по физике пережечь? 

Решение:
До пережигания нити система находится в состоянии Решение задач по физике После пережигания она, пройдя через состояние равновесия Решение задач по физике достигает состояния Решение задач по физике В состоянии равновесия 
Решение задач по физике
Очевидно, потенциальная энергия системы в состоянии Решение задач по физике относительно состояния Решение задач по физике равна 
Решение задач по физике
Такой же потенциальной энергией относительно состояния Решение задач по физике система обладает и в состоянии Решение задач по физике
Решение задач по физике
Приравняв два, последних выражения и учитывая первое  соотношение, получим 

Решение задач по физике

Задача № 63. Поезд массой Решение задач по физике шел равномерно по горизонтальному пути. От поезда оторвался задний вагон массой Решение задач по физике Проехав после этого Решение задач по физике машинист прекратил доступ пара в машину. На каком расстоянии друг от друга остановятся отделившийся вагон и остальной состав поезда? Предполагается, что машина все время работала одинаково и что сопротивление движению поезда и вагона пропорционально движущейся массе. 
Дано: 
Решение задач по физике

Решение:
Пусть Решение задач по физике — путь, пройденный оторвавшимся вагоном до остановки, Решение задач по физике — путь, пройденный остальным составом поезда после прекращения доступа пара в машину до остановки, Решение задач по физике —  расстояние между остановившимся вагоном и поездом: 
Решение задач по физике
Если после отрыва заднего вагона прекратится доступ пара в машину, поезд и оторвавшийся вагой будут двигаться, одинаково и пройдут путь, определяемый из формулы 
Решение задач по физике
или 
Решение задач по физике
При равномерном движении поезда сила тяги паровоза уравновешивается силой сопротивления всего состава: Решение задач по физике Когда задний вагон оторвался, то сила сопротивления уменьшилась, а сила тяги по условию задачи не изменилась. 
На поезд стала действовать сила Решение задач по физике ускоряющая его движение, равная 
Решение задач по физике
Поезд после отрыва вагона стал двигаться под действием этой силы ускоренно и его кинетическая энергия на пути Решение задач по физике возросла па величину Решение задач по физике Изменение кинетической энергии можно определить по работе силы Решение задач по физике на пути Решение задач по физике

Решение задач по физике
Когда машинист прекратил доступ пара в машину, поезд уже 
обладал кинетической энергией 
Решение задач по физике
Подставив в последнее выражение вместо Решение задач по физике его значение Решение задач по физике получим 
Решение задач по физике
Кинетическая энергия ЕК поезда была израсходована на работу по преодолению сопротивления па пути Решение задач по физике
Решение задач по физике
Отсюда 
Решение задач по физике
Подставив в выражение Решение задач по физике вместо Решение задач по физике его значение, получим 
Решение задач по физике

Задача № 64. Физкультурник разбегается в течение времени Решение задач по физике и прыгает в длину. Определить максимальную дальность прыжка, если коэффициент трения Решение задач по физике а максимальная высота прыжка Решение задач по физике 

Решение:
Горизонтальную составляющую скорости в момент начала прыжка Решение задач по физике можно найти из второго закона Ньютона: 
Решение задач по физике
где Решение задач по физике — время разбега; Решение задач по физике —время толчка; Решение задач по физике — масса физкультурника; Решение задач по физике — сила нормального давления при толчке; начальная скорость Решение задач по физике равна нулю. 
Так как Решение задач по физике то 
Решение задач по физике
откуда 
Решение задач по физике 
Зная горизонтальную составляющую скорости, можно определить и дальность прыжка: 

Решение задач по физике

Задача № 65. С какой скоростью растет толщина покрытия стенки золотом при распылении, если атомы золота летят, обладая энергией Решение задач по физике и производят давление на стенку, равное Решение задач по физике Плотность золота Решение задач по физике
Дано; 
Решение задач по физике

Решение:
Пусть Решение задач по физике — толщина слоя, тогда скорость покрытия выразится как Решение задач по физике примем эту величину за искомую. При этом через Решение задач по физикебудем обозначать массу одной молекулы золота, через Решение задач по физике — массу всех молекул. 
Известно, что произведение массы на скорость есть количество движения, а произведение силы на время — импульс силы. Тогда 
Решение задач по физике
Масса отложившегося вещества равна 
Решение задач по физике
где Решение задач по физике — толщина слоя; Решение задач по физике — поверхность стенки; Решение задач по физике — плотность золота. 
Давление, производимое атомами золота, равно частному: 
Решение задач по физике
Подставив вместо Решение задач по физике его значение, получим 
Решение задач по физике 
где Решение задач по физике — скорость атомов золота, которую находим из формулы 
Решение задач по физике
Откуда 
Решение задач по физике
Определим массу атома золота. Атомный вес золота равенРешение задач по физике и по закону Авогадро в каждом килограмм-моле находится Решение задач по физике атомов. 

Следовательно, масса одного атома 
Решение задач по физике

Итак, 
Решение задач по физике
Подставив в формулу 
Решение задач по физике
вместо Решение задач по физике его значение, получим скорость напыления 
Решение задач по физике

Задача № 66. Два абсолютно упругих тела массами Решение задач по физике подвешены на длинных нерастяжимых нитях. Тело массой Решение задач по физике отклонили на угол Решение задач по физике и отпустили. Проанализировать, как будут изменяться в зависимости от отношения масс Решение задач по физике — максимальные углы отклонения тел после удара. 

Решение:
Обозначим длину нити через Решение задач по физике При отклонении на угол Решение задач по физике (рис. 41, а) тело Решение задач по физике будет находиться на высоте 
Решение задач по физике
Потенциальная энергия данного тела в положении Решение задач по физике
Решение задач по физике
При своем движении в точке Решение задач по физике тело имеет определенную  скорость, которую легко подсчитать по закону сохранения энергии 
Решение задач по физике 
или 
Решение задач по физике
После удара о тело Решение задач по физике первое тело отскочит и поднимется на высоту Решение задач по физике второе — на высоту Решение задач по физике (рис. 41, б). Пусть углы отклонения будут соответственно Решение задач по физике До удара количество движения первого тела равнялось то, второго — нулю.

Решение задач по физике

Рис. 41.

Пусть Решение задач по физике — скорости движения тел после удара, тогда по закону сохранения  
количества движения получим 
Решение задач по физике
а по закону сохранения энергии 
Решение задач по физике
Составим систему уравнений 
Решение задач по физике
и найдем значения Решение задач по физике
Решение задач по физике
Разделив второе уравнение на первое, получим 
Решение задач по физике
Подставляя данное значение в уравнение Решение задач по физикеполучим 
Решение задач по физике
откуда 

Решение задач по физике
где 
Решение задач по физике
Аналогичным образом найдем Решение задач по физике
Решение задач по физике 
Таким образом, тело т поднимется на высоту Решение задач по физике
Решение задач по физике
Точно так же 
Решение задач по физике
Из треугольника Решение задач по физике 
Решение задач по физике
Аналогично 
Решение задач по физике
Рассмотрим несколько случаев. 
1. Пусть Решение задач по физике Это возможно, когда Решение задач по физике или Решение задач по физике
Тогда 
Решение задач по физике 
или 
Решение задач по физике
т. е. второе тело остается неподвижным, а первое отскакивает от него на первоначальный угол. 
2. Решение задач по физике В этом случае 
Решение задач по физике
т. е. после удара первое тело полностью останавливается, а второе приходит в движение. Тела обмениваются движениями. 
3. Решение задач по физике Тогда 
Решение задач по физике

Задача № 67. Подвешенный на нити шарик массой Решение задач по физике отклоняясь на угол Решение задач по физике колеблется в вертикальной плоскости. Определить натяжение нити в тот момент, когда она отклонена от вертикали на угол Решение задач по физике

Решение задач по физике

Рис.42.

Дано: 
Решение задач по физике

Решение:
Пусть пить составляет с вертикалью угол Решение задач по физике Разложим действующую на шарик силу тяжести 
по двум направлениям: радиусу окружности и касательной к ней (рис. 42), Тогда натяжение нити равно сумме двух сил: центробежной силы и нормальной составляющей веса шарика: 

Решение задач по физике
где Решение задач по физике — сила натяжения нити; Решение задач по физике — длина ее; Решение задач по физике — линейная скорость. 
По закону сохранения энергии 

Решение задач по физике
Из рисунка видно, что 
Решение задач по физике
Поэтому 
Решение задач по физике
Подставляя это значение в выражение для натяжения нити и учитывая, чтоРешение задач по физике получим

 Решение задач по физике

Работа и мощность. Закон сохранения импульса. Закон сохранения энергии

Задача № 4.1. Цилиндр и куб из одинакового материала, одинаковой высоты и массы стоят на горизонтальной плоскости. Какое из этих двух тел труднее опрокинуть?

Решение:

Для того чтобы опрокинуть куб или цилиндр вокруг ребра, нужно повернуть их так, чтобы их диагональные плоскости ABCD и KLМN (рис. 199, а, б) заняли вертикальное положение; при этом центр масс тела поднимается. Следовательно, необходимо совершить работу, которая будет тем больше, чем выше придется поднять центр масс (масса куба равна массе цилиндра). Если диагональная плоскость (куба или цилиндра) займет вертикальное положение, для чего придется повернуть диагональ AD вокруг ребра АВ на угол Решение задач по физике а диагональ цилиндра МК на угол Решение задач по физике то центр масс поднимается: у куба на Решение задач по физике у цилиндра на Решение задач по физике(рис. 199, в, г).

Решение задач по физике

Рис. 199

Так как массы и высоты, а также и материалы куба и цилиндра одинаковы, то равны и площади их оснований, т. е. Решение задач по физике где R — радиус основания цилиндра. Для куба, очевидно, Решение задач по физике = 45°. Для цилиндра находим Решение задач по физике или Решение задач по физике Подставляя значение h2 , получим Решение задач по физике т. е. Решение задач по физике > 45°. Следовательно, cos Решение задач по физике < cos Решение задач по физике, а поэтому Решение задач по физике следовательно, цилиндр опрокинуть через ребро труднее, чем куб. Если попытаться опрокинуть куб через вершину (а не через ребро), то пришлось бы повернуть его так, чтобы диагональ АС заняла вертикальное положение АС' (рис. 199, д). Для этого нужно куб повернуть на угол Решение задач по физике образуемый диагональю куба с высотой. Тогда Решение задач по физике Так как Решение задач по физике то Решение задач по физике Следовательно, Решение задач по физике и опрокинуть куб через вершину труднее, чем опрокинуть цилиндр.

Задача № 4.2. Подсчитать полную минимальную работу, которую необходимо совершить, чтобы перекантовать ящик массой = 103 кг сначала вокруг ребра А1В1, потом вокруг ребра А2В2. Длина ящика l = 0,8 м, высота h = 0,6 м (рис. 41).

Решение задач по физике

Рис. 41                                                                        Рис. 42

Решение:

Чтобы перекантовать ящик вокруг ребра А1В1 , надо повернуть ящик так, чтобы его диагональная плоскость стала вертикальной (см. задачу 4.1), а дальше ящик упадет под действием силы тяжести. Таким образом, нужно совершить работу по подъему центра масс ящика на соответствующую высоту (рис. 200). Эта работа равна Решение задач по физике При перекантовке через ребро А1В1 имеем = 0,6 м, h' = 1 м; А Решение задач по физике 2 кДж. При перекантовке через ребро А2В2 : = 0,8 м, h' = 1 м; А Решение задач по физике 1 кДж. Полная минимальная совершенная работа равна 3 кДж.

Решение задач по физике

Рис. 200

Задача № 4.3. Изменятся ли работа, совершенная мотором эскалатора, и мощность, если человек, стоящий на движущейся вверх лестнице эскалатора, будет сам также подниматься по эскалатору с постоянной скоростью?

Решение:

Если человек поднимается по лестнице эскалатора с постоянной скоростью, то его среднее давление на лестницу останется неизменным (равным силе тяжести человека). Следовательно, и сила, с которой мотор должен тянуть лестницу, останется неизменной. Однако поднимающийся по лестнице эскалатора человек раньше достигнет верха эскалатора, а значит, и путь, пройденный эскалатором за все время подъема человека до верха, будет меньше, чем в том случае, когда человек на лестнице неподвижен. Поэтому работа, совершенная мотором эскалатора при подъеме движущегося человека, будет меньше работы, совершенной при подъеме неподвижного (остальную часть работы совершит человек). Затрачиваемая же мотором мощность останется неизменной, так как меньшая работа будет совершена за соответственно меньшее время.

Задача № 4.4. Два шкива, находящихся на одном уровне, соединены ремнем; левый шкив ведущий (рис. 42). Когда возможно передать через эту трансмиссию большую мощность: когда шкивы вращаются по часовой стрелке или против?

Решение:

Когда шкивы вращаются по часовой стрелке, ремень охватывает большую часть каждого из шкивов, так как нижняя часть ремня при этом прижата к шкивам вследствие того, что она натянута, а верхняя прогибается благодаря своей массе (рис. 201, а).

Решение задач по физике

Рис. 201

Поэтому сила трения будет больше, и скольжение ремня по шкиву начнется, очевидно, при большей нагрузке на шкив, а значит, и мощность можно будет передать большую, чем при вращении против часовой стрелки (рис. 201, б).

Задача № 4.5. Принцип действия автоматического оружия основан на использовании явления отдачи при выстреле: затвор отходит после выстрела назад и сжимает пружину; сжатая пружина приводит в действие механизм перезарядки. Определить, какую скорость v должна иметь пуля, чтобы затвор отошел назад на расстояние l, если масса пули m, масса затвора М и жесткость пружины k. Массой заряда пренебречь.

Решение:

Для того чтобы затвор отошел назад на расстояние 1, необходимо, чтобы против упругой силы пружины была совершена работа kl2 /2. Эта работа будет совершена за счет кинетической энергии, которую получает затвор при отдаче. Если затвор имеет начальную скорость и, то его кинетическая энергия Ми2/2 = kl2/2, откуда Решение задач по физике С другой стороны, импульс затвора Ми при выстреле должен по модулю равняться импульсу пули mv (так как они противоположны по направлению и в сумме должны дать нуль). Следовательно,

Решение задач по физике

Задача № 4.6. Две пружины одинаковых размеров (железная и медная) упруго растянуты на одну и ту же длину. На растяжение которой из них понадобилось совершить большую работу?

Решение:

Модуль Юнга (модуль продольной упругости) для железа больше, чем для меди. Поэтому для растяжения пружин одинаковых размеров на одну и ту же длину для железной пружины нужна большая сила, чем для медной. Значит, в первом случае совершена большая работа, чем во втором.

Задача № 4.7. Две пружины одинаковых размеров (железная и медная) упруго растянуты одной и той же силой. На растяжение которой из них понадобилось совершить большую работу?

Решение:

Если растяжение производилось одной и той же силой, то железная пружина растянута меньше, чем медная. Поэтому на этот раз для растяжения медной пружины понадобилось совершить большую работу.

Задача № 4.8. На нити длиной l подвешен груз массой m. Минимальный груз массой М, подвешенный на нити, разрывает ее, растягивая нить в момент разрыва на 1 % от ее длины. Определить, на какую минимальную высоту h надо поднять груз массой m, чтобы он, падая, разорвал нить? Считать, что для нити справедлив закон Гука вплоть до ее разрыва.

Решение:

Груз массой m, упав с высоты h, обладает кинетической энергией, равной изменению потенциальной энергии, т. е. mgh. Эта кинетическая энергия должна превратиться в энергию упругой деформации нити, т. е. в случае, если справедлив закон Гука, должна быть равна kx2 /2, где х — наибольшее растяжение нити (в момент разрыва), k — жесткость нити. Согласно условию задачи х = 0,01 l и kx = Mg. Подставляя эти соотношения в уравнение kx2/2 = mgh, получим искомую высоту: h = 0,01 Ml/2m.

Задача № 4.9. С высоты Н по гладкой наклонной плоскости длиной l = Н/3 и с углом наклона Решение задач по физике = 30° соскальзывает без трения шарик и затем падает на горизонтальную плоскость (рис. 43), удар о которую следует считать абсолютно упругим. На какую высоту h поднимается шарик после удара о плоскость?

Решение:

Шарик соскальзывает с наклонной плоскости (рис. 202), имея скорость, равную Решение задач по физике и направленную nод углом Решение задач по физике = 30° к горизонту. Дальше шарик летит по параболе и падает на горизонтальную плоскость со скоростью, направленной под некоторым углом к горизонту. Но высота подъема шарика после абсолютно упругого удара о плоскость зависит только от вертикальной составляющей этой скорости, которая найдется как скорость, с которой упадет шарик с высоты (5/6)Н с начальной скоростью Решение задач по физике

Решение задач по физике

Рис. 202

Из уравнения движения

Решение задач по физике

находим время падения шарика: Решение задач по физике Поэтому его скорость в конце падения будет равна Решение задач по физике Следовательно, высота подъема шарика после упругого удара о плоскость равна Решение задач по физике

Закон сохранения импульса

Задача № 4.10. Пуля массой m попадает в деревянный брусок массой М, подвешенный на нити длиной l ("баллистический маятник"), и застревает в нем. Определить, на какой максимальный угол Решение задач по физике отклонится маятник, если скорость пули равна v (рис. 44).

Решение:

Пуля массой m, летя со скоростью v, обладает импульсом mv. После того как пуля застряла в деревянном бруске, тем же импульсом будет обладать брусок вместе с пулей (абсолютно неупругий удар). Следовательно, скорость v1, которую приобретает брусок сразу после попадания в него пули, определится из закона сохранения импульса: mv = (М + m) v1. При этом кинетическая энергия бруска с пулей будет равнаРешение задач по физике

Затем брусок поднимается и эта кинетическая энергия превращается в потенциальную. Так как вся масса М + m практически находится на расстоянии l от точки подвеса А (рис. 203), то центр масс, вследствие отклонения маятника на угол Решение задач по физике поднимается на высоту Решение задач по физике При наибольшем отклонении маятника потенциальная энергия должна быть равна начальной кинетической энергии, т. е.

Решение задач по физике

Отсюда максимальный угол, на который отклоняется маятник, определяется соотношением Решение задач по физике

Решение задач по физике                             Решение задач по физике

Рис. 203                                                                 Рис. 204

Задача № 4.11. В пробирке массой М, закрытой пробкой массой m, находится капля эфира. При нагревании пробирки пробка вылетает под давлением паров эфира. Пробирка подвешена на невесомом жестком стержне длины l (рис. 45). С какой минимальной скоростью v1 должна вылететь пробка, чтобы пробирка сделала полный оборот вокруг точки подвеса О?

Решение задач по физике                          Решение задач по физике

Рис. 43                                                                                Рис. 44                             Рис. 45

Решение:

Скорость v1 — та минимальная скорость, с которой должна вылететь пробка для того, чтобы пробирка описала полный оборот вокруг точки О (рис. 204). В таком случае пробка получит импульс mv1, а пробирка поэтому должна получить импульс Mv2 , причем по закону сохранения импульса v2 = mv1/M. В начальный момент своего движения пробирка будет обладать кинетической энергией Решение задач по физике Эта энергия должна пойти на то, чтобы поднять массу М на высоту 2l. По закону сохранения энергии будем иметь уравнение Решение задач по физике Отсюда, подставляя значение скорости v2 , получаем Решение задач по физике

Задача № 4.12. Две легкие тележки массами m1 и m2 = 3m1 соединены пружиной (рис. 46). Пружина сжата и связана ниткой. Нитку пережигают, пружина распрямляется и тележки разъезжаются в противоположные стороны. Найти: а) отношение скоростей тележек v1 и v2 ; б) отношение времен t1 и t2 , в течение которых тележки движутся; в) отношение расстояний s1 и s2 , пройденных тележками. Коэффициент трения k для обеих тележек считать одинаковым.

Решение задач по физике

Рис. 46

Решение:

а) По модулю импульс первой тележки должен равняться импульсу второй тележки, так как в начальный момент сумма импульсов равна нулю, т. е. m1v1 = m2v2 ; откуда v1/v2 = m2/m1 = 3.

б) Движение каждой тележки замедляется под действием силы трения. Сила трения определяется коэффициентом трения и массой тележки:

Решение задач по физике

Под действием силы трения скорость каждой тележки убывает до нуля (тележки останавливаются). Импульс силы трения равен изменению импульса каждой тележки:

Решение задач по физике отсюда Решение задач по физике

в) Расстояние, пройденное каждой тележкой, можно найти, зная время движения и среднюю скорость: Решение задач по физике

Задача № 4.13. Снаряд, вылетевший из орудия под некоторым углом к горизонту, в верхней точке своей параболической траектории разрывается на два осколка равной массы. Первый осколок под влиянием взрыва возвращается к исходной точке по прежней траектории. Где упадет второй осколок? Упадут ли оба осколка на землю одновременно? Сопротивление воздуха не учитывать.

Решение:

Так как первый осколок в результате взрыва возвращается по прежней траектории из верхней точки параболы, то, значит; он в результате взрыва получил импульс –mv, равный тому, которым он обладал до взрыва, но противоположный по знаку; иначе говоря, в результате взрыва произошло изменение импульса этого осколка на -2mv. По закону сохранения импульса второй осколок при взрыве должен получить такой же по модулю импульс, но направленный в противоположную сторону, т. е. +2mv в сторону его прежнего движения. Поэтому у второго осколка после взрыва импульс будет равен Зmv, и, следовательно, он начнет движение из верхней точки параболы с тройной скоростью, а поэтому пройдет по горизонтали втрое больший путь, чем первый осколок, т. е. упадет на землю вдвое дальше, чем упал бы снаряд, если бы взрыва в воздухе не было. Так как оба осколка в вертикальном направлении после взрыва не имели никакого импульса, то в этом направлении они будут свободно падать без начальной скорости с одной и той же высоты, а потому упадут на землю одновременно.

Можно предложить и другое решение исходя из того, что во время взрыва внешние силы не проявляются. Поэтому центр масс снаряда до и после взрыва имеет одинаковую скорость. Так как к тому же оба осколка nосле взрыва свободно падают по вертикали, то центр масс снаряда будет продолжать описывать параболу, которую описывал бы неразорвавшийся снаряд. Так как осколки имеют равные массы, то они в своем движении будут всегда расположены симметрично относительно параболы центра масс (рис. 205). Следовательно, второй осколок упадет в точке D, причем BD = АВ.

Решение задач по физике

Рис. 205

Задача № 4.14. Опытный игрок в мяч, ловя его, расслабляет руки и слегка подается назад вместе с мячом. Зачем?

Решение:

Чтобы остановить мяч, игрок должен действовать на мяч со средней силой F = mv/t, где — скорость мяча и t — время, в течение которого мяч останавливается. Расслабляя руки и подаваясь назад, игрок в мяч увеличивает время t и тем самым ослабляет силу, с которой он должен действовать на мяч, а значит и силу удара. Неопытный игрок, наоборот, напрягает руки и тело и тем самым увеличивает силу удара, рискуя при этом упустить мяч.

Задача № 4.15. В неподвижный шар ударяет боком (не по линии центров) другой шар такой же массы, движущийся со скоростью v. Под каким углом разлетятся шары, если они абсолютно упругие и абсолютно гладкие?

Решение:

Как известно, в случае абсолютно упругого центрального удара двух шаров одинаковой массы, из которых один до удара был неподвижен, шары обмениваются скоростями, т. е. неподвижный продолжает двигаться со скоростью первого шара, а этот последний останавливается. Если же направление скорости движущегося шара не проходит через центр покоящегося шара, то в момент удара можно скорость v движущегося шара разложить по двум взаимно перпендикулярным направлениям, из которых одно проходит через центр покоящегося шара. Мы можем считать, что движущийся шар обладает двумя скоростями: v1 и v2 (рис. 206). В направлении скорости v1 происходит абсолютно упругий центральный удар, в результате которого покоившийся шар начинает двигаться со скоростью v1, а ранее двигавшийся шар теряет скорость в этом направлении, но сохраняет скорость v2, в направлении которой продолжает двигаться.

Таким образом, после удара шары разлетаются под прямым углом.

Решение задач по физике

Рис. 206

Задача № 4.16. Несколько одинаковых стальных или костяных шаров подвешены на нитях, прикрепленных к одной доске (рис. 47). Шары в начальном положении касаются друг друга, и нити Параллельны между собой. Если крайний правый шар отвести и отпустить, то, после удара о неподвижные шары, он останется в покое, а крайний левый шар отскочит. Если же отвести справа не один, а два шара и отпустить их одновременно, то слева отскочат тоже два. Как объяснить этот опыт?

Решение задач по физике

Рис. 47

Решение:

Удар костяных шаров можно рассматривать как абсолютно упругий удар. Если при абсолютно упругом центральном ударе двух одинаковых шаров один до удара покоится, то, как известно, после удара покоившийся шар начинает двигаться со скоростью, равной той, которую имел первый шар до удара, а ранее двигавшийся шар останавливается. В нашей модели происходит ряд последовательных абсолютно упругих центральных ударов одного шара о другой с теми же результатами. Но промежуточные шары передают свою скорость следующему и останавливаются. Так же и последний шар отходит со скоростью, которой обладал первый шар .в момент удара; но поскольку он уже не встречает на пути других шаров, то поднимается на такую высоту, как и та, с которой упал правый шар (потерями энергии мы пренебрегаем).

Если же отвести вправо не один, а два шара и отпустить их, то они будут падать независимо один от другого. При достижении ряда шаров (3 – 8) произойдет не один удар, последовательно два удара один за другим, т. е. сначала ударится второй о третий, а затем первый о второй (рис. 207).

Решение задач по физике

Рис. 207

Первый из этих ударов приведет к тому, что последний свободный шар (8-й) отойдет от седьмого и начнет подниматься кверху. После этого 7-й шар окажется свободным и в результате следующего удара (1-го шара) начнет подниматься кверху. Ясно, что оба эти шара (7-й и 8-й) отойдут на столько же, на сколько первоначально мы отвели 2-й и 1-й.

Закон сохранения энергии

Задача № 4.17. Как будет протекать опыт в предыдущей задаче, если подвесить только три шарика и отвести в сторону два соседних шарика на одну и ту же высоту, отпустив их затем одновременно без начальной скорости?

Решение:

Второй из падающих шариков (средний из трех) ударяет упруго в покоящийся 3-й шарик. Они обмениваются скоростями. Ранее покоившийся 3-й шарик приобретает скорость ударившего его 2-го, а 2-й шарик останавливается. Но происходит тотчас второй упругий удар: 1-й из падающих шариков ударяет в остановившийся 2-й шарик. Снова происходит обмен скоростями и остановившийся 2-й шарик приобретает скорость ударившего (т. е. примерно такую же, как и 3-й шарик), а ударивший шарик останавливается. Таким образом, в результате двух упругих ударов отклоняются в сторону 2-й и 3-й шарики на ту же высоту, на какую были отклонены два шарика первоначально.

Задача № 4.18. Пусть шары из задачи 3.16, только не соединенные стержнем, движутся по параллельным траекториям равномерно со скоростями v1v2 и v3. С какой скоростью v0 движется центр масс этих шаров?

Решение:

Разделим обе части формулы, полученной при решении задачи 3.16, на промежуток времени t. Тогда отношение расстояния от края стержня к t будет пропорционально скорости движения. Таким образом, получим Решение задач по физике или, вообще, при любом числе n шаров

Решение задач по физике

Задача № 4.19. Пользуясь классическими формулами скоростей двух шаров после упругого удара

Решение задач по физике

где v1, v2 — скорости шаров до удара, Решение задач по физике — скорости после удара, показать, что скорость центра масс двух шаров после удара (независимо от характера удара) равна скорости центра масс этих шаров до удара.

Решение:

Используем классические формулы скоростей шаров после упругого удара:

Решение задач по физике

Скорость центра масс шаров после удара на основании формулы, полученной при решении задачи 4.18, выразится так: Решение задач по физике Подставим в это выражение значения Решение задач по физике и Решение задач по физике. Тогда легко получим, что скорость центра масс шаров после упругого удара будет равнаРешение задач по физике что совпадает со скоростью центра масс этих шаров до удара.

При неупругом ударе оба шара движутся вместе, т. е. с одной и той же скоростью. Поэтому их центр масс движется с этой скоростью, которая также выражается соотношением Решение задач по физике Следовательно, при неупругом ударе имеет место та же закономерность. Таким образом, импульс центра масс системы шаров до и после удара не изменяется. В частности, если до удара центр масс шаров покоился, то и после удара (как упругого, так и неупругого) центр масс будет оставаться неподвижным. То, что справедливо для шаров, является справедливым для любой системы материальных точек.

Выведенную нами закономерность следует рассматривать как иную формулировку закона сохранения импульса, т. е. при отсутствии внешних сил центр масс системы материальных точек сохраняет свой импульс, а следовательно, и свою скорость. Это положение иногда позволяет проще решать некоторые задачи механики. В дальнейшем мы это покажем.

Задача № 4.20. Через блок, подвешенный достаточно высоко, переброшен канат, по концам которого поднимаются две обезьяны одной и той же массы, причем одна перемещается по канату вдвое быстрее другой. Которая из них раньше доберется доверху? Блок считать невесомым, а канат — невесомым и нерастяжимым.

Решение:

Так как внешние силы, которые могли бы сообщать обезьянам импульс, отсутствуют, то обезьяны могут сообщать друг другу (через канат) только одинаковые импульсы. Поэтому независимо от того, как быстро обезьяны перебирают лапами по канату, подниматься относительно земли они будут с одинаковой скоростью (поскольку массы их равны). Канат же будет передвигаться в сторону той обезьяны, которая быстрее перебирает лапами, именно с такой скоростью, чтобы скорость движения обеих обезьян относительно земли оказалась одинаковой. Поэтому обе обезьяны достигнут блока одновременно.

Задачу можно упростить так, чтобы этот ответ стал совершенно очевидным. Представим себе, что обезьяны находятся на абсолютно гладкой горизонтальной плоскости и держатся за концы каната. Поскольку внешние силы отсутствуют, центр масс двух обезьян должен оставаться неподвижным, а значит, они могут приближаться к нему только на одинаковые расстояния, как бы ни перебирала лапами канат та или другая из них. Поэтому обе обезьяны одновременно достигнут точки, лежащей на середине начального расстояния между ними. Этот же метод позволяет легко ответить на вопрос задачи в случае, когда обезьяны имеют разные массы: более легкая доберется раньше.

Задача № 4.21. Лодка стоит неподвижно в стоячей воде. Человек, находящийся в лодке, переходит с носа на корму. На какое расстояние сдвинется лодка, если масса человека = 60 кг, масса лодки М = 120 кг, длина лодки l = 3 м? Сопротивление воды не учитывать.

Решение:

Пусть человек переходит с носа на корму равномерно в течение времени t (рис. 208). Так как мы предположили, что внешних сил нет, то импульс системы лодка – человек не должен измениться, т. е. во все время движения человека лодка должна двигаться в противоположном направлении с такой скоростью, чтобы общий импульс был равен нулю.

Решение задач по физике

Рис. 208

Пусть лодка за то же время t переместилась в противоположную сторону на расстояние х. Тогда скорость человека относительно земли за это время была (l – x)/t, а скорость лодки x/t. Закон сохранения импульса дает

Решение задач по физике

откуда Решение задач по физике

Этот же результат может быть получен на основании следствия, вытекающего из закона сохранения импульса: при отсутствии внешних сил центр масс системы не может переместиться. Когда человек стоит на носу Н лодки, центр масс системы лодка – человек находится на вертикали, проходящей через точку А, причем СА= 0,5 м. Когда человек перешел на корму К, то центр масс той же системы находится на вертикали, проходящей через точку В, причем ВС = 0,5 м. Так как во время перехода человека с носа на корму никакие внешние силы на систему лодка – человек не действовали, то центр масс системы не может переместиться. Для этого лодка должна переместиться так, чтобы точка В совпала с прежним положением точки А, т. е. лодка должна переместиться вправо на расстояние ВА, равное 1 м.

Задача № 4.22. На железнодорожной платформе, которая может двигаться по рельсам, укреплены две одинаковые пушки, направленные в противоположные стороны (рис. 48). Прицелы установлены так, что при одновременных выстрелах оба снаряда попадают каждый в свою цель. Попадут ли снаряды в цели, если одна из пушек выстрелила немного раньше другой? Что произойдет с платформой после второго выстрела? Силу трения в колесах платформы не учитывать.

Решение задач по физике

Рис. 48

Решение:

Очевидно, когда обе пушки стреляют одновременно, платформа остается неподвижной и именно при этом условии (стрельба с неподвижной платформы) снаряды попадают в цель. Если одна из пушек выстрелит раньше, то по закону сохранения импульса платформа начнет двигаться в противоположную сторону. Поскольку все условия движения снаряда в стволе орудия остались прежними, а сам ствол вместе с платформой начинает двигаться в противоположном направлении (это движение начинается вместе с движением снаряда, т. е. nока он находится еще в стволе), снаряд вылетит из ствола с меньшей начальной скоростью относительно земли, чем в случае неподвижной платформы. Следовательно, снаряд не долетит до цели. Движение платформы будет продолжаться вплоть до выстрела из второй nушки. Поэтому при выстреле ствол второй пушки вместе с платформой будет двигаться в сторону выстрела, и, значит, снаряд при вылете из пушки будет иметь несколько большую скорость, чем при стрельбе из неподвижной nушки. Следовательно, будет иметь место перелет снаряда.

Так как скорость снаряда при втором выстреле больше, чем при первом, то, как следует из закона сохранения импульса, после второго выстрела платформа должна начать двигаться в направлении, противоположном второму выстрелу. Для упрощения мы не учитываем того обстоятельства, что ствол пушки после выстрела движется относительно лафета. Но ясно, что это обстоятельство не изменит принципиально той роли, которую играет движение платформы.

Задача № 4.23. Три шара равных масс подвешены на пружинах с одной и той же жесткостью так, что расстояния между 1-м и 2-м и между 2-м и 3-м одинаковы (рис. 49). Таким образом, центр масс всей системы совпадает с центром 2-го шара. Если обрезать нить, удерживающую 1-й шар, то система будет падать, причем ускорение центpa масс системы должно быть равно

(mg + mg + mg) / Зm = g

(по второму закону Ньютона ускорение центра масс системы равно сумме сил, действующих на систему извне, деленной на массу всей системы). Но пружина I тянет 2-й шар вверх сильнее, чем пружина II тянет этот шар вниз (упругая сила пружины I в начальный момент F10 = 2mg, а упругая сила пружины II в начальный момент F20 = mg ), и, следовательно, в начальный момент центр масс второго шара будет иметь ускорение меньше g. Между тем центр масс системы должен все время двигаться с ускорением g. В чем дело?

Решение задач по физике                  Решение задач по физике

Рис. 49                                 Рис. 50

Решение:

Пружины I и II, имеющие одну и ту же жесткость, в начальный момент действуют с разными силами: 2mg и mg, но при этом имеют одинаковую длину. Значит, их длина в недеформированном состоянии должна быть различна. При свободном падении обе пружины должны быть не растянутыми, т. е. они сожмутся до нормальной длины (их деформации исчезнут), и так как эта длина у двух пружин разная, то расстояния между центрами 1-го и 2-го и 2-го и 3-го шаров уже не будут одинаковыми. Таким образом, центр второго шара после начала падения перестает быть центром масс системы трех шаров.

Задача № 4.24. Карманные часы положены на горизонтальную подставку, которая может свободно вращаться вокруг вертикальной оси (рис. 50). Как изменится от этого ход часов? Трением диска об ось пренебречь.

Решение:

Пружина часов одним концом прикреплена к корпусу часов, а другой ее конец прикреплен к маятнику (рис. 209). Период колебаний маятника зависит от жесткости и длины пружины. Чем короче пружина, тем меньше период колебаний. Когда часы лежат на подставке, нужно считать, что корпус часов жестко соединен с нею. Если подставка может свободно вращаться вокруг вертикальной оси, то поворот маятника в одну сторону вызовет поворот подставки. и :a:opnyca часов в другую сторону.

Решение задач по физике

Рис. 209

Вследствие этого пружина будет закручиваться с обоих концов в противоположные стороны. Поэтому неподвижная точка пружины окажется где-то внутри пружины — между концом, прикрепленным к корпусу часов, и маятником. Иначе говоря, длина пружины как бы уменьшается, что ведет жк уменьшению периода колебаний. Итак, часы начнут спешить.

Задача № 4.25. Маленький шарик бросают вертикально вверх с некоторой начальной скоростью. В момент, когда он достигает максимальной высоты, бросают другой такой же шарик с той же начальной скоростью по той же вертикали. На некоторой высоте шарики сталкиваются, и в этот момент из той же точки бросают вверх по той же вертикали с той же начальной скоростью третий такой же шарик. Через сколько времени с момента бросания третьего шарика упадут один за другим все три шарика? Удар шариков при встрече считать абсолютно упругим.

Решение:

Наиболее· наглядным является графическое решение задачи (рис, 210). Tк как все три шарика имеют одинаковую начальную скорость, то, если бы столкновений не было, график зависимости пути (высоты) от времени для всех трех шариков имел бы одинаковый вид.

Решение задач по физике

Рис. 210

Чтобы учесть результат столкновений, нужно принять во внимание следующее: 1) шарики сталкиваются, когда графики их движения пересекаются, 2) при столкновении шарики обмениваются скоростями (удар упругий), т. е. после столкновения каждый шарик продолжает движение другого (по его графику). (На рис. 210 цифры указывают, движению какого шарика соответствуют разные участки графиков.) Будем отсчитывать время от момента бросания первого шарика, и обозначим через t1 время, необходимое для его подъема до максимальной высоты. Тогда третий шарик упадет через 2t1, второй через 3t1 и первый через 3,5t1.

Задача № 4.26. По принципу относительности Галилея две системы координат, движущиеся равномерно и прямолинейно друг относительно друга, равноправны, т. е. физические законы, справедливые в одной системе, справедливы и в другой. Пусть система II движется относительно системы I равномерно и прямолинейно со скоростью v. В этом же направлении движется тело А со скоростью v1 по отношению к системе I (следовательно, со скоростью Решение задач по физике  относительно системы II). В течение некоторого времени t на тело А действует постоянная сила F, направленная по той же прямой, как и скорости v и v1; она изменяет скорость тела относительно системы I от значения v1 до значения v2. Изменение кинетической энергии тела будет в системе I равно

Решение задач по физике,

в системе II

Решение задач по физике

т. е. будет меньше. В разных системах координат изменение кинетической энергии разное. Как это согласовать с принципом относительности Галилея?

Решение:

Принцип относительности требует, чтобы в двух рассматриваемых системах соблюдались одни и те же физические законы и, в частности, закон сохранения энергии, по которому изменение энергии тела должно быть равно работе внешних сил. Поэтому в системе I должно быть справедливо соотношение

Решение задач по физике                                               (1)

где s — длина пути, пройденного телом в системе I за то время, в течение которого скорость возросла от v1 до v2.

В системе II, соответственно,

Решение задач по физике                                (2)

где s1 — длина пути, пройденного телом в системе II за то же время. Но так как скорость тела в системах I и II различна, то s и s1 различны. В самом деле, так как тело движется под действием силы F с ускорением F/m, то в системе имеем

Решение задач по физике

а в системе II , соответственно,

Решение задач по физике 

Следовательно, s – s1 = vt. Но так как F/m =а= (v2 – v1)/t, то t = m(v2 – v1)/F, s – s1 = mv(v2 – v1)/F, а следовательно, F (s – s1) = mv (v2 – v1).

Таким образом, работа внешней силы в системе I настолько больше, чем в системе II, насколько изменение кинетической энергии в системе I больше изменения энергии в системе II. Так как в системе I изменение энергии равно работе внешних сил, то это же справедливо и для системы II. Следовательно, принцип относительности Галилея не нарушен.

Динамика вращательного движения

Задача № 5.1. Для уменьшения потерь на трение при вращении вала в опорном подшипнике было предложено сточить конец вала на конус (рис. 51). При этом модуль силы трения, очевидно, не изменится, поскольку при уменьшении площади соприкасающихся поверхностей увеличивается давление между ними и соответственно возрастает сила трения на единицу площади соприкосновения. Однако стачивание вала все же может уменьшить потери на трение, если трение на боковых стенках подшипника отсутствует. Почему уменьшаются потери на трение после стачивания?

Решение:

 Сила трения в обоих случаях одинакова, но момент силы трения относительно оси вращения в случае сточенного конца вала меньше, а потери на трение при вращении определяются именно моментом сил трения.

Задача № 5.2. К валу приложен вращающий момент М = 1 кН · м. На вал насажено колесо радиусом R = 0,5 м (рис. 52). С какой силой F надо прижимать тормозные колодки к колесу, чтобы оно не вращалось? Коэффициент трения k = 0,25.

Решение:

Если сила, с которой надо прижимать тормозные колодки к колесу, равна F, то сила трения будет равна по модулю kF. Момент сил трения, действующих на две колодки, будет равен kF · 2R. Этот момент пары сил трения должен уравновешивать вращающий момент М2kFR = М. Отсюда F = M/2kR =4 кН.

Задача № 5.3. Через неподвижный блок с моментом инерции I переброшена нить, на которой висят грузы разных масс m1 и m2 (рис. 53). Каковы будут силы натяжения Т1 и Т2 нити no обе стороны блока? 

Решение задач по физике

Рис. 51                                                                                  Рис. 52

Решение задач по физике

Рис. 53                                                     Рис. 54

Решение:

Если бы блок был невесомым, то силу натяжения нитей по обе стороны блока следовало бы считать одинаковой (см. задачу 2.22). Но так как блок обладает моментом инерции, нужно учесть, что нить будет натянута по обе стороны блока по-разному, и разность сил натяжения будет создавать момент, вращающий блок. Пусть сила натяжения нити слева Т1 , справа — Т2. Тогда, применяя второй закон Ньютона к движению масс m1 и m2, получим уравнения

m1a = m1g – Т1,                                                     (1)

-m2a = m2g – Т2,                                                   (2)

При этом мы считаем положительным направление ускорения груза массы m1, а поэтому ускорение а во втором уравнении взято со знаком минус.

Если угловое ускорение блока обозначить через Решение задач по физике причем Решение задач по физике где R — радиус блока, то, применяя второй закон Ньютона к вращению, будем иметь

Решение задач по физике                                                        (3)

Решая совместно уравнения (1), (2) и (3), найдем силы натяжения:

Решение задач по физике

При I = 0 отсюда получаем Т1 = Т2, что совпадает с результатом задачи 2.22.

Момент инерции твердого тела

Задача № 5.4. Поворот автомобиля должен быть обусловлен внешней силой, действующей под некоторым углом к направлению движения. Этой внешней силой может быть только сила трения между шинами и землей. Почему же при повороте передних колес автомобиля направление силы трения изменяется таким образом, что она вызывает поворот всего автомобиля?

Решение:

Автомобиль в момент поворота передних колес обладает скоростью поступательного движения v, которую можно разложить по двум взаимно перпендикулярным направлениям х и у (рис. 211). Колеса должны были бы одновременно катиться по направлению х и скользить по направлению у. Но если должно возникнуть скольжение, то всегда появляется сила трения, направленная в сторону, противоположную той, в которую должно было бы происходить скольжение. Эта внешняя сила f и вызывает поворот автомобиля в нужную сторону,

Решение задач по физике

Рис. 211

Задача № 5.5. Тело соскальзывает по желобу в форме мертвой петли (рис. 54) с наименьшей допустимой высоты, описывает петлю и не падает. При этом в верхней точке петли тело на желоб не давит. На какое же тело действует в этот момент центробежная сила?

Решение:

Так как во верхней точке петли центростремительная сила — это сила, с которой тело притягивается к земле, то центробежная сила — это сила, с которой земля притягивается к телу, т. е. она действует на землю.

Задача № 5.6. Желоб предыдущей задачи имеет в верхней части петли разрыв, симметричный относительно вертикали, проходящей через центр петли. Радиусы R желоба, идущие к краям А и В разрыва, образуют угол, равный Решение задач по физике (рис. 55). С какой минимальной высоты Н должен начать скользить без трения шарик, чтобы пролететь разрыв и снова попасть на желоб? Какова траектория шарика в разрыве желоба?

Решение:

Пусть уровень разрыва желоба на h ниже края желоба, с которого начинает скользить без трения шарик. Тогда скорость шарика на уровне разрыва дается уравнением

mgh = mv2/2.                                                                     (1)

Длина хорды АВ (рис. 212), равная Решение задач по физике  — это дальность свободного полета, который должен совершить шарик, начавший движение от точки А со скоростью v под углом Решение задач по физике к горизонту, т. е. Решение задач по физике Таким образом, для определения скорости v имеем уравнение

Решение задач по физике

откуда

Решение задач по физике                                                       (2)

Подставляя значение v2 из (2) в уравнение (1), будем иметь после сокращения на Решение задач по физикеРешение задач по физике а высота края желоба над нижней точкой петли будет равна

Решение задач по физике

Эта высота имеет минимальное значение при Решение задач по физике = 45°. Траектория шарика в разрыве петли есть парабола.

Решение задач по физике

Рис. 212                                                                            Рис. 213

Задача № 5.7. На вращающемся диске укреплен отвес, который устанавливается под углом а к вертикали (рис. 56). Известны расстояние r от точки подвеса до оси вращения и длина нити отвеса l. Определить угловую скорость вращения Решение задач по физике.

Решение задач по физике            Решение задач по физике

Рис. 55                                                                         Рис. 56

Решение:

Отвес устанавливается так, что равнодействующая силы тяжести mg и натяжения нити Т дает центростремительную силу Решение задач по физике (рис. 213). Очевидно, что Решение задач по физикеСледовательно,

Решение задач по физике

Задача № 5.8. Почему монета, катящаяся по плоскости в вертикальном положении (без наклона), движется по прямой; а наклоненная движется криволинейно?

Решение:

У монеты, катящейся в вертикальном положении, сила тяжести mg и сила реакции плоскости N уравновешивают друг друга, поэтому они не могут искривить путь монеты (рис. 214, а). У наклоненной монеты (рис. 214, 6) сила реакции N со стороны плоскости по-прежнему равна силе тяжести монеты, но, кроме того, возникает сила трения f , действующая вдоль плоскости.

Решение задач по физике

Рис. 214

Ее появление объясняется тем, что при наклонном положении монеты должно было бы возникнуть скольжение точки А монеты влево. При этом всегда появляется сила трения, препятствующая возникновению скольжения. Таким образом, равнодействующая R силы реакции плоскости N и силы трения f оказывается наклоненной к плоскости (рис. 214, в).  Равнодействующая же силы тяжести mg и силы R, направленная горизонтально в сторону наклона, и вызывает искривление траектории движения центра масс монеты.

Задача № 5.9. Конькобежец может описать круг на льду, наклоняясь в сторону центра круга. Откуда берется центростремительная сила, необходимая для движения по окружности?

Решение:

На конькобежца, наклонившегося в сторону центра круга, действуют две силы: сила тяжести и сила реакции ледяной дорожки, которая имеет горизонтальную составляющую и поэтому направлена наклонно. Горизонтальная составляющая реакции ледяной дорожки (рис. 215) возникает вследствие того, что конек легко скользящий вдоль своего направления, не скользит в перпендикулярном направлении. Эта горизонтальная составляющая аналогична силе трения, действующей на монету в задаче 5.8.

Сила тяжести конькобежца mg и сила реакции N ледяной дорожки дают равнодействующую силу R, направленную горизонтально и сообщающую конькобежцу центростремительное ускорение, необходимое для движения по окружности.

Решение задач по физике

Рис. 215

Кинетическая энергия вращательного движения твердого тела

Задача № 5.10. На шкив двигателя плотно надета цепочка. Двигатель приводят в быстрое вращение. Затем постепенно сдвигают цепочку на край шкива и, наконец, сбрасывают ее. Тогда цепочка катится как жесткий обруч по столу или по полу. Объяснить, как возникает центростремительная сила, необходимая для того, чтобы каждое звено цепочки описывало кривую? Как возникают силы, действующие на цепочку и создающие в ней напряжении?

Решение:

При вращении цепочки на шкиве в ней возникает деформация — цепочка растягивается, как всякое быстро вращающееся тело. Вследствие этого возникают упругие силы между отдельными звеньями цепочки. На каждое звено цепочки действуют два соседних звена. Эти упругие силы F1 и F2 направлены под углом друг к другу, а их равнодействующая R направлена к центру цепочки и является центростремительной силой (рис. 216, а), заставляющей это звено описывать кривую.

Решение задач по физике

Рис. 216

В свою очередь каждое звено цепочки действует на остальную часть цепочки с двумя силами, равными по модулю и противоположными по направлению тем, которые действуют на данное звено (рис. 216, б). Эти силы придают цепочке жесткость И делают ее подобной жесткому кольцу. Работа сил трения замедляет вращение цепочки, и цепочка в конце концов теряет жесткость и падает.

Задача № 5.11. Летчик массой m =70 кг описывает на самолете, летящем со скоростью v = 180 км/ч, «мертвую петлю» радиусом R = 100 м. Откуда берется центростремительная сила, действующая на летчика в нижней и верхней точках петли, на что действует центробежная сила в этих же точках и с какой силой прижимается летчик к сидению в верхней и нижней точках петли? Считать ускорение свободного падения g = 9,8 м/с2.

Решение:

Для того чтобы летчик описывал указанную в условии задачи мертвую петлю, на него во всех точках петли должна действовать центростремительная сила

Fц.с. = mv2/R = 1,750 кН.

В нижней точке петли эта сила берется из составляющей силы реакции сидения, а в верхней точке — из силы тяжести летчика и из силы реакции сидения. Поэтому центробежная сила в нижней точке петли действует целиком на сидение, а в верхней точке — частично на сидение, а частично на землю. В нижней точке петли на сидение действует и сила тяжести летчика, и центробежная сила, поэтому летчик прижимается к сидению с силой 2,436 кН, а в верхней точке петли на сидение действует только часть центробежной силы, равная центробежной силе минус сила тяжести летчика, поэтому в верхней точке петли летчик прижимается к сидению с силой 1,064 кН.

Задача № 5.12. Если велосипедист на повороте не наклонится в сторону поворота, то он упадет в противоположную сторону. Почему?

Решение:

Велосипед и велосипедист имеют общую скорость по касательной к закруглению. На повороте велосипедист поворачивает рулевое колесо в сторону поворота. Тогда прежняя скорость может быть разложена на две составляющие: по направлению движения рулевого колеса и перпендикулярно к нему. В направлении первой составляющей скорости рулевое колесо может продолжать двигаться в сторону поворота, в направлении второй составляющей скорости движению велосипеда будет препятствовать трение колеса о землю и оно не будет двигаться в эту сторону. Велосипедист же должен продолжать двигаться в эту сторону, поскольку на него не действуют силы, уменьшающие эту составляющую скорости. Продолжая двигаться в этом направлении (в котором велосипед не движется), велосипедист неизбежно упадет.

Задача № 5.13. Почему на большой скорости автобус на повороте наклоняется в сторону, противоположную повороту, но не падает, как велосипедист в предыдущей задаче?

Решение:

При повороте автобуса центростремительное ycкopeниe ему сообщают силы трения земли о колеса, приложенные к нижней точке колес. Центру масс эти силы непосредственно не сообщают ускорения, поэтому корпус автобуса движется по кривой большего радиуса, чем колеса. Корпус слегка сдвигается наружу, и это приводит к растяжению рессор, опирающихся на внутренние (по отношению к повороту) колеса, и сжатию рессор, поддерживающих внешние (по отношению к повороту) колеса. При этом возникает вращающий момент упругих сил, который препятствует дальнейшему увеличению наклона кузова.

Отметим, что со стороны рессор действует момент обратного направления. Однако на очень больших скоростях этот момент очень велик и может привести к тому, что колеса, описывающие внутреннюю кривую, оторвутся от земли и автобус все же опрокинется.

Задача № 5.14. Шарик массой m, подвешенный на нити длиной l, отклонили от положения равновесия так, что он поднялся на высоту h (рис. 57). Затем шарик отпустили. На какую высоту он поднимется, если на пути нити поставить стержень А, перпендикулярный к плоскости чертежа (опыт Галилея)?

Решение задач по физике

Рис. 57

Решение:

Если шарик поднялся на высоту h, то его потенциальная энергия возросла на величину mgh. Эта энергия будет при колебании маятника переходить в кинетическую и обратно, причем, когда шарик снова поднимется на наибольшую высоту и остановится, он будет обладать той же потенциальной энергией. Значит, шарик должен подняться на ту же высоту, так как при встрече нити со стержнем А кинетическая энергия маятника не изменяется. Поэтому, где бы ни был расположен стержень А, перпендикулярный к плоскости чертежа (лишь бы расстояние АВ было меньше l – h/2), шарик поднимется на ту же высоту h. Если же АВ больше l – h/2, то это будет невозможно. Но в этом случае шарик в наивысшей точке будет обладать еще некоторой скоростью (так как еще не вся кинетическая энергия превратилась в потенциальную). Он будет продолжать двигаться в том же направлении, и нить обовьется вокруг стержня.

Задача № 5.15. Маленький шарик подвешен в точке А на нити, длина которой l. В точке О на расстоянии l/2 ниже точки А в стену вбит гвоздь. Шарик отводят так, что нить занимает горизонтальное положение, и отпускают  (рис. 58). В какой точке траектории исчезнет натяжение нити? Как дальше будет двигаться шарик? До какой наивысшей точки поднимется шарик? В какой точке шарик пересечет вертикаль, проходящую через точку подвеса?  

Решение задач по физике

Рис. 58                                                                                Рис. 59

Решение:

Шарик сначала описывает четверть окружности радиуса, равного длине нити l. Затем нить задевает гвоздь, вбитый в стенку в точке О, и шарик описывает дугу окружности вдвое меньшего радиуса. Наконец, когда сила тяжести шарика будет сообщать ему центростремительное ускорение, необходимое для движения по окружности, натяжение нити обратится в нуль. Пусть это произойдет в точке С (рис. 217).

Решение задач по физике

Рис. 217

Ее положение определим следующим образом. Составляющая силы тяжести по направлению радиуса равна Решение задач по физике где Решение задач по физике — угол, образованный нитью в этот момент. Далее, значение v2 в точке С равно 2gH, где Н= АВ = АО – ВО = = l/2 – (1/2) cos Решение задач по физике. Поэтому центростремительная сила в точке С равна

Решение задач по физике

Итак, в точке С имеем равенство Решение задач по физике откуда Решение задач по физике = 2/3. Далее шарик летит как тело, брошенное под углом Решение задач по физике к горизонту с начальной скоростью Решение задач по физике В этом случае верхняя точка параболы находится выше уровня точки взлета на Решение задач по физике Вертикаль, проходящая через точку подвеса, находится от точки С на расстоянии

СВ = Решение задач по физике 

Чтобы пройти по горизонтали такой путь, шарику потребуется время Решение задач по физике За это время шарик по высоте пройдет путь Решение задач по физике т. е. пересечет вертикаль АО в точке, лежащей на (5/96)l ниже точки В.

Момент силы

Задача № 5.16. Почему при ударе кием с силой F по нижней части бильярдного шара он движется замедленно, а при ударе по верхней части — первое время ускоренно?

Решение:

Шар будет двигаться замедленно, если сила трения будет направлена против движения, и ускоренно, если эта сила направлена по движению. Направление силы трения противоположно направлению скорости, с которой происходит скольжение в точке К — точке касания шара и поверхности (рис. 218, а). Скорость же скольжения в этой точке зависит от соотношения двух скоростей: скорости поступательного движения шара v0 и скорости вращения вокруг центра масс.

Решение задач по физике

Рис. 218

При ударе кием по нижней части бильярдного шара (шар вращается по часовой стрелке, рис. 218, б) скорость в точке К, обусловленная вращением, направлена вперед, и возникает скольжение, также направленное вперед. Поэтому появляется сила трения, направленная назад, которая замедляет движение шара. При ударе кием по верхней части бильярдного шара (рис. 218, в) шар вращается против часовой стрелки. При достаточно «высоком» ударе, когда скорость его вращения велика (так что обусловленная этим вращением линейная скорость в точке К больше скорости поступательного движения), возникает скольжение, направленное назад. Появляется сила трения, направленная по движению, и шар движется ускоренно.

Задача № 5.17. Линейная скорость точек земного экватора — около 460 м/с. Что было бы с пулей, вылетевшей из ружья на экваторе параллельно земной поверхности в западном направлении с такой же скоростью, если бы отсутствовала атмосфера?

Решение:

Если смотреть на пулю извне, не с Земли, а, например, с Луны, то путь ее представится как вертикальное падение на Землю, так как скорость движения пули относительно Земли и скорость движения Земли относительно внеземного наблюдателя равны по модулю и противоположны по направлению. Если смотреть на пулю с Земли, путь ее представится, как обычно, параболой, заканчивающейся падением пули на Землю.

Уравнение динамики вращательного движения твердого тела

Задача № 5.18. Даны два цилиндра одинаковых размеров и из одного и того же материала. Один цилиндр сплошной, другой сборный, состоящий из двух цилиндров, почти без зазора вложенных один в другой, причем трение между внешним и внутренним цилиндрами отсутствует (рис. 59). Какой из цилиндров будет быстрее скатываться без скольжения по одной и той же наклонной плоскости? Какое положение будет занимать при скатывании внутренний цилиндр, если он не совсем плотно входит во внешний?

Решение:

Качение без скольжения можно рассматривать как вращение вокруг мгновенной оси О (образующей цилиндра, касающейся в данный момент наклонной плоскости) и поступательное движение этой оси. Вращающим моментом при этом является момент силы тяжести mg относительно оси О (рис. 219). Массы цилиндров, а значит, и их силы тяжести равны, поэтому и вращающие моменты для обоих цилиндров равны. Однако их моменты инерции различны.

Вo втором (сборном) цилиндре играет роль момент инерции лишь наружного цилиндра, так как внутренний цилиндр, вследствие отсутствия сил трения, не вращается.

Решение задач по физике

Рис. 219

Очевидно, что момент инерции первого (сплошного) цилиндра больше, чем момент инерции внешнего полого цилиндра во втором случае. При одном и том же вращающем моменте угловое ускорение обратно пропорционально моменту инерции цилиндра. Поэтому угловое ускорение сборного цилиндра больше, т. е. он скатывается быстрее.

Положение внутреннего цилиндра в зазоре можно определить из следующих соображений. Если бы внутренний цилиндр скользил без трения по наклонной плоскости, то ero ускорение под действием силы тяжести было бы большим, чем ускорение центра скатывающегося цилиндра. Это видно хотя бы из того, что потенциальная энергия, которой обладает цилиндр в поле тяжести, при скольжении превращается только в кинетическую энергию поступательного движения, а при скатывании в энергию и поступательного и вращательного движений. Следовательно, скорость, а значит, и ускорение поступательного движения при скольжении должны быть больше, чем при скатывании. Так как в случае сборного цилиндра оба они — и внешний и внутренний — опускаются с одинаковым ускорением, то, значит, внешний цилиндр замедляет движение внутреннего. А для этого сила F, с которой внешний цилиндр давит на внутренний, должна иметь составляющую F1, направленную вдоль наклонной плоскости кверху, т. е. сила F должна быть отклонена несколько назад от направления перпендикуляра к наклонной плоскости (рис. 220).

Решение задач по физике

Рис. 220

Так как трения нет, то сила F должна быть нормальна к поверхности цилиндров, и, чтобы при этом она была отклонена назад, цилиндры должны касаться друг друга по образующей, лежащей, во всяком случае, впереди радиуса, проведенного к точке касания внешнего цилиндра с наклонной плоскостью.

Закон всемирного тяготения

Задача № 68. Какую необходимо сообщить телу минимальную скорость в горизонтальном направлении к поверхности Земли, чтобы оно стало искусственным спутником Земли? Определить период  
обращения спутника Земли на высоте Решение задач по физике 
Дано: 
Решение задач по физике

Решение:
Пусть тело находится на некоторой очень малой высоте над поверхностью Земли. При сообщении телу некоторой определенной скорости в горизонтальном направлении оно будет двигаться вокруг Земли по круговой орбите, превратившись в искусственный спутник 
Земли (это утверждение справедливо, если пренебречь сопротивлением воздуха). Соответствующее значение скорости этого тела носит название первой космической скорости. 
На тело, двигающееся по круговой орбите, действует центростремительная сила, величина которой выражается формулой 

Решение задач по физике
где Решение задач по физике — масса тела; Решение задач по физике — скорость движения; Решение задач по физике — радиус кривизны траектории. 
В рассмотренном случае центростремительной силой является сила притяжения тела Землей. Эта сила притяжения выражается формулой 
Решение задач по физике 
где Решение задач по физике — масса Земли; Решение задач по физике — гравитационная постоянная; Решение задач по физике —  расстояние тела от центра Земли (равное радиусу кривизны траектории). 
Приравнивая выражения для центростремительной силы Решение задач по физике и силы притяжения Решение задач по физике получаем 
Решение задач по физике
Отсюда можно найти следующее выражение для скорости Решение задач по физике

Решение задач по физике
Подставляем в это выражение численные значения входящих в него величин: 
Решение задач по физике
Значение скорости Решение задач по физике можно найти, если даже значение массы Земли Решение задач по физике является неизвестным. Для этого надо разделить обе части равенства Решение задач по физике
Решение задач по физике
Согласно второму закону Ньютона, отношение Решение задач по физикеравно ускорению Решение задач по физике свободно падающего тела у поверхности Земли: 

Решение задач по физике
Следовательно, 

Решение задач по физике
и 
Решение задач по физике
Подставим в последнее уравнение значения Решение задач по физике выраженные в единицах системы Решение задач по физике 
Выполнив арифметические действия, получим 
Решение задач по физике
Период обращения спутника вокруг Земли 
Решение задач по физике
где Решение задач по физике — угловая скорость обращения спутника вокруг Земли. Она равна 
Решение задач по физике
где Решение задач по физике — скорость спутника; Решение задач по физике — высота нахождения спутника над поверхностью Земли. Таким образом, 
Решение задач по физике

Задача № 69. На какую высоту должен быть запущен искусственный спутник Земли, чтобы его период обращения был равен периоду вращения Земли вокруг своей оси? Масса Земли Решение задач по физике 
Дано: 
Решение задач по физике 

Решение:
Центростремительное ускорение может быть определено из  формулы кинематики 

Решение задач по физике
и по закону всемирного тяготения 
Решение задач по физике
где Решение задач по физике —радиус Земли; Решение задач по физике—линейная скорость обращения спутника вокруг Земли; Решение задач по физике — гравитационная постоянная; Решение задач по физике —высота, на которой находится спутник. 
Очевидно, что 
Решение задач по физике
откуда 

Решение задач по физике
Так как скорость 

Решение задач по физике
то 

Решение задач по физике
Следовательно, 
Решение задач по физике
Расстояние от центра Земли до спутника равно Решение задач по физике Высота Решение задач по физике
Таким образом, спутник, пролетающий на высоте примерно Решение задач по физике будет парить над одной и той же точкой земной поверхности (если, конечно, он вращается в ту же сторону, что и Земля). При этом спутник должен также находиться над одним из пунктов земного экватора. 

Задача № 70. На экваторе некоторой планеты тела весят вдвое меньше, чем на полюсе. Плотность вещества планеты Решение задач по физике  Определить период обращения планеты около собственной оси. 
Дано: 
Решение задач по физике

Решение:
По условию задачи вес тела на экваторе планеты вдвое меньше, чем на полюсе. Это означает, что центростремительная сила,  необходимая для удержания тела на экваторе, при вращении планеты вокруг своей оси составляет половину силы тяготения. Рассмотрим это подробнее. 
По третьему закону Ньютона со стороны опоры на тело действует сила Решение задач по физике равная и противоположная весу тела Решение задач по физике 
Таким образом, к покоящимся на планете телам приложены две силы: сила тяготения Решение задач по физике и реакция опоры Решение задач по физике численно равная Решение задач по физике
Эти силы сообщают телу необходимое центростремительное ускорение при вращении планеты вокруг своей оси. На полюсе и на экваторе они направлены по одной линии (по вертикали). 
По второму закону Ньютона 
Решение задач по физике
где Решение задач по физике — скорость движения тела вокруг оси. 
На полюсе Решение задач по физике
Реакция опоры и, следовательно, вес тела на полюсе равен силе 
тяготения. 
На экваторе 
Решение задач по физике
где Решение задач по физике — период обращения, 

Решение задач по физике
По условию 

Решение задач по физике
откуда 

Решение задач по физике

По закону всемирного тяготения 

Решение задач по физике
где Решение задач по физике—масса тела; Решение задач по физике — масса планеты; Решение задач по физике — радиус планеты, но Решение задач по физике где Решение задач по физике — плотность планеты. 
Подставляя в формулуРешение задач по физике выражения для Решение задач по физике получаем 
Решение задач по физике 
После сокращения получим 
Решение задач по физике
Подставив численные значения, найдем 
Решение задач по физике

Гравитационное взаимодействия

Задача № 71. Какую скорость приобретает метеорит, падая с высоты Решение задач по физике до ощутимо плотных слоев атмосферы, расположенных на высоте Решение задач по физике если допустить, что сопротивление движению до этой высоты так мало, что им можно пренебречь? Какое количество теплоты при этом выделится, если заметное торможение метеорита началось на высоте Решение задач по физике Считать, что вся кинетическая энергия метеорита превратилась в тепло. 
Дано: 
Решение задач по физике 

Решение:
При отсутствии потерь кинетическая энергия метеорита возрастает настолько, насколько уменьшится его потенциальная энергия: 
Решение задач по физике
где Решение задач по физике — масса метеорита; Решение задач по физике —вертикальные составляющие скорости метеорита. 
Предположим, что начальная скорость метеорита Решение задач по физике В первом приближении можно воспользоваться формулой потенциальной энергии 
Решение задач по физике
где Решение задач по физике — ускорение для какой-то средней высоты Решение задач по физике над Землей между начальной и конечной точками падения. Для определения Решение задач по физике для высоты Решение задач по физике воспользуемся законом всемирного тяготения: 
Решение задач по физике
где Решение задач по физике — масса Земли; Решение задач по физике — радиус Земли. 
С другой стороны, 
Решение задач по физике
Из двух последних равенств находим, что 
Решение задач по физике
Так как 
Решение задач по физике
то 
Решение задач по физике
При такой скорости в процессе торможения должно выделиться огромное количество тепла. 
При полной остановке метеорита вся его кинетическая энергия Решение задач по физике обратится в теплоту Решение задач по физике т. е. 

Решение задач по физике
т. е. на каждый килограмм приходится Решение задач по физике теплоты.

Задача № 72. Считая, что орбита первого советского искусственного спутника Земли имела окружность радиусом Решение задач по физике определить число оборотов спутника за сутки. 
Дано: 
Решение задач по физике

Решение:
Число оборотов определим по формуле 
Решение задач по физике
где Решение задач по физике — угловая скорость спутника. В каждый момент времени ее можно рассчитать по формуле 
Решение задач по физике
где Решение задач по физике — линейная скорость; Решение задач по физике — радиус обращения. 
Определим скорость Решение задач по физике
Если тело обращается по окружности под действием силы тяготения, то последняя играет роль центростремительной силы. Она должна быть равна произведению массы спутника Решение задач по физике на  
ускорение Решение задач по физике Поэтому 
Решение задач по физике
где Решение задач по физике — масса Земли. Откуда 

Решение задач по физике
Число оборотов 
Решение задач по физике
Из выражения Решение задач по физике найдем значение Решение задач по физике и подставим в предыдущее уравнение 

Решение задач по физике
После подстановки данных получим Решение задач по физике оборотов. 

Задача № 73. Подсчитать ускорение свободно падающих тел на  поверхности Солнца, если известны радиус земной орбиты Решение задач по физике радиус Солнца Решение задач по физике и время обращения Земли вокруг Солнца (один год).
Дано: 
Решение задач по физике

Решение:
При движении Земли по своей орбите вокруг Солнца возникает центростремительная сила Решение задач по физике которая удерживает Землю на ее круговой орбите; 
Решение задач по физике
Так как 
Решение задач по физике
то 
Решение задач по физике
По третьему закону Ньютона одновременно с центростремительной возникает центробежная сила, которая уравновешивается силой всемирного тяготения 
Решение задач по физике
Приравняв правые части, получим 

Решение задач по физике
В этом уравнении неизвестно Решение задач по физике Сила тяжести любого тела Решение задач по физике уравновешивается силой всемирного тяготения Решение задач по физике
Приравняв правые части, получим 
Решение задач по физике
Подставив это выражение в предыдущее уравнение, получим 
Решение задач по физике
откуда 
Решение задач по физике

Задача № 74. В однородном шаре радиусом Решение задач по физике и массой Решение задач по физике имеется сферическая полость радиусом Решение задач по физике поверхность которой касается шара и проходит через его центр (рис. 43). На расстоянии Решение задач по физике от центра шара находится точечное тело, масса которого Решение задач по физике С какой силой шар с полостью будет притягивать тело Решение задач по физике 

Решение:
Рассмотрим случай, когда точечное тело Решение задач по физике находится со стороны вырезанной полости. Сила взаимодействия между сплошным шаром и телом Решение задач по физике определяется по формуле 
Решение задач по физике

Решение задач по физике
Рис. 43 


Так как шар полый, то его масса уменьшится на 

Решение задач по физике
Расстояние центра массы Решение задач по физике от тела Решение задач по физике 
Решение задач по физике
Вследствие того, что шар полый, сила взаимодействия между двумя телами Решение задач по физике уменьшится на 
Решение задач по физике
Тогда искомая сила Решение задач по физике или 
Решение задач по физике
Предлагаем читателям рассмотреть самостоятельно случай, когда тело Решение задач по физике расположено по другую сторону шара. 

Классическая теория тяготения Ньютона

Задача № 6.1. В свинцовом шаре радиусом R сделана сферическая полость, которая касается поверхности шара и проходит через его центр. Масса шара до того, как была сделана полость, равнялась М. С какой силой F свинцовый шap будет притягивать маленький шарик массой m, находящийся на расстоянии r от центра свинцового шара на прямой, соединяющей центры шаров и полости со стороны полости (рис. 60)?

Решение задач по физике

Рис. 60

Решение:

Если бы свинцовый шар был сплошной, то он притягивал бы маленький шарик с силой

Решение задач по физике

где G — гравитационная постоянная. Можно считать, что сила притяжения Fспл сплошного шара складывается из двух сил: из силы притяжения нашего шара со сферической полостью внутри (на рис. 60 заштрихован) и силы притяжения меньшего шара радиуса R/2, заполняющего сферическую полость в нашем шаре. Цель задачи — найти первую силу. Масса шара, который заполнил бы сферическую полость, равна

Решение задач по физике

а центр его лежит на расстоянии r - R/2 от шарика массы m. Искомая сила, равная разности сил притяжения сплошного шара и меньшего шара, заполняющего сферическую полость, выразится так:

Решение задач по физике

Очень часто эту задачу решают неверно. Так как ошибка поучительна, мы приведем решение и разъясним, в чем ошибка.

Вычислялось положение нового центра масс свинцового шара после того, как в нем сделана полость; расстояние нового центра масс от центра шара можно определить из уравнения

Решение задач по физике

откуда х = R/14. Затем определялась сила притяжения свинцовым шаром с полостью (масса его равна (7 /8)М) шарика массы m так, как будто это две точечные массы, находящиеся на расстоянии l + R/14 друг oт друга, т. е. по формуле

Решение задач по физике

Нетрудно видеть, что этот результат отличается от полученного нами решения. Они совпадают, только если R << l. Ошибка заключается в неправильном предположении, что шар с полостью притягивает массу m так же, как его притягивала бы точечная масса той же величины, помещенная там, где находится центр масс шара с полостью. Центр масс есть точка приложения равнодействующей всех параллельных сил, действующих на определенные элементы тела, причем каждая из этих сил пропорциональна массе данного элемента тела. Но силы, с которыми на массу m действуют отдельные элементы шара, во-первых, не параллельны друг другу, так как все они направлены к точке m; и во-вторых, хотя они и пропорциональны массам элементов тела, но для элементов равной массы, вообще говоря, различны, так как зависят от расстояния данного элемента до точки m. Поэтому заменять силу тяготения данного тела силой тяготения точечной массы такой же величины, помещенной в центре масс данного тела, вообще говоря, нельзя. Только в специальных случаях, когда размеры тел малы по сравнению с расстоянием между ними (т. е. когда тела можно считать материальными точками) или когда притягивающее тело особо симметричной формы, например однородный шар, можно вычислять силу тяготения этого тела, считая, что вся его масса сосредоточена в центре масс. Этим последним обстоятельством мы и пользовались, когда вычисляли силы тяготения сплошного шара и заполняющего полость меньшего шара.

Силы всемирного тяготения

Задача № 6.2. Солнце притягивает любую точку на земной поверхности сильнее, чем Луна, а между тем явления приливов и отливов вызываются главным образом действием Луны, а не Солнца. Почему?

Решение:

Явление приливов и отливов возникает вследствие того, что данное небесное тело (Луна, Солнце) сообщает различные ускорения всему земному шару в целом и воде, находящейся на его поверхности. Всему земному шару в целом небесное тело сообщает такое же ускорение, какое оно сообщало бы телу, помещенному в центре земного шара. Но по закону всемирного тяготения ускорение, сообщенное телом массы М другому, находящемуся на расстоянии r, есть Решение задач по физике  где G — гравитационная постоянная. Следовательно, разность ускорений воды, находящейся на поверхности Земли, и всей Земли в целом выразится так:

Решение задач по физике

где r — расстояние от небесного тела до центра Земли, а R — радиус Земли (рис. 221). Так как величина R очень мала по сравнению с r, то приближенно для этой разности будем иметь

Решение задач по физике

Значение этой разности для Солнца и Луны и определяет явления приливов и отливов, вызываемые ими. Так как r для Луны порядка 60 Rr для Солнца порядка 25 000 R, то r3 для Солнца больше, чем для Луны приблизительно в 75 · 106 раз, а масса Солнца больше массы Луны всего лишь в 27 · 106 раз. Поэтому приливное действие Луны почти в три раза больше приливного действия Солнца.

Решение задач по физике

Рис. 221

Задача № 6.3. На все тела на Земле действует сила притяжения Солнца. Ночью (Солнце «под ногами») эта сила складывается с силой притяжения Земли, днем (Солнце «над головой») — из нее вычитается. Следовательно, ночью все тела должны быть тяжелее, чем днем. Верно ли это?

Решение:

Сила притяжения Солнца действует не только на груз, находящийся на весах, но и на Землю, и поэтому сообщает грузу и весам одинаковое ускорение (если пренебречь приливными эффектами; см. задачу 6.2). Поэтому сила притяжения Солнца не изменяет растяжения пружин, т. е. не влияет на показания весов. Так же как сила притяжения Земли не растягивает пружину, если груз вместе с пружиной и подставкой, к которой она прикреплена, свободно падает на Землю, так и сила притяжения Солнца не растягивает пружины, так как подставка вместе с Землей «падает» на Солнце. Это "падение" выражается в том, что Земля в своем движении по орбите обладает центростремительным ускорением, как раз равным тому ускорению свободного падения, которым обладала бы она или любое другое тело, помещенное на орбиту Земли и не имеющее начальной скорости. Следовательно, масса тeл на Земле днем и ночью одна и та же.

Задача № 6.4. С помощью ракеты тело поднято на высоту 500 км. 1. Каково ускорение свободного падения на этой высоте? 2. С какой скоростью v нужно бросить это тело no направлению, перпендикулярному к земному радиусу, чтобы оно описывало окружность вокруг Земли? 3. Каков будет при этом период обращения тела вокруг Земли? Радиус Земли считать равным 6 500 км, ускорение свободного падения на поверхности Земли — 9,80 м/с2. Сопротивления атмосферы не учитывать. (В данной задаче тело будет находиться в условиях почти таких, в каких находился первый советский искусственный спутник Земли.)

Решение:

По закону всемирного тяготения ускорение свободного падения, приобретаемое телом, обратно пропорционально квадрату расстояния от Земли, и поэтому тело, поднятое на высоту 500 км и, следовательно, находившееся на расстоянии 7000 км от центра Земли, будет обладать ускорением свободного падения, равным Решение задач по физике

Чтобы тело могло описывать окружность вокруг Земли, ускорение, сообщаемое ему Землей, должно быть равно требуемому центростремительному ускорению. Поэтому ац.с = 8,45 м/с2 = v2/(7 · 106 ). Отсюда получаем, что v Решение задач по физике 7,7 км/с. 3. При найденной скорости движения один оборот тела вокруг Земли будет совершен за время, равное

Решение задач по физике

Задача № 6.5. В романе Жюля Верна «Путешествие на Луну» говорится, что человек теряет свой вес, когда достигает точки, в которой притяжение к Земле становится равным притяжению к Луне. А как же наши космонавты находятся в состоянии невесомости, хотя орбиты космических кораблей с людьми nока еще далеко не достигают точки, где силы притяжения Земли и Луны одинаковы?

Решение:

Жюль Верн полагал, что состояние невесомости наступает тогда, когда сила тяготения перестает действовать, в частности, например, потому, что силы тяготения Земли и Луны уравновешивают друг друга. Однако это очень частный и потому наименее интересный случай. Состояние невесомости наступает и в том случае, когда на тело действуют только силы тяготения. При этом, так как и тела и подставки или подвесы, их поддерживающие, под действием сил тяготения испытывают одинаковое ускорение (g — вблизи поверхности Земли), то тела перестают давить на подставки и подвесы и сами не испытывают деформаций (которые возникают, когда тело, находящееся под действием силы тяготения, не движется с ускорением g, а покоится на подставке или подвесе). На космический корабль, движущийся по орбите, и на все находящиеся в нем тела действует только сила тяготения, поэтому все тела и космонавты пребывают в состоянии невесомости.

Колебательные движения

Задача № 75. Радиус Решение задач по физике равномерно вращается против часовой стрелки с угловой скоростью Решение задач по физике (рис. 44, а). Точка Решение задач по физике — проекция точки Решение задач по физике на ось Решение задач по физике За начало отсчета времени приняли тот момент, когда угол Решение задач по физике Напишите формулу, по которой можно найти скорость точки Решение задач по физике для любого момента времени Решение задач по физике 
Решение задач по физике

Рис. 44.

Решение:
Угол поворота при равномерном вращении 
Решение задач по физике
Угол между подвижным радиусом Решение задач по физике и осью Решение задач по физике (рис. 44, б) 
Решение задач по физике
Скорость точки Решение задач по физике равна горизонтальной составляющей скорости точки Решение задач по физике поэтому 
Решение задач по физике
Знак минус стоит потому» что скорость направлена в сторону убывания координаты Решение задач по физике 

Задача № 76. Тележка, на которой укреплен маятник с периодом колебаний Решение задач по физике находится на наклонной плоскости, расположенной под углом Решение задач по физике к плоскости горизонта. Чему будет равен период колебания Решение задач по физике маятника, когда тележка начнет скатываться по наклонной плоскости? 
Дано: 
Решение задач по физике

Решение:
Движение маятника будет равноускоренным по отношению к поверхности Земли и колебательным по отношению к поверхности тележки. Скатывающая сила Решение задач по физике сообщит маятнику ускорение  поступательного движения, под действием силы Решение задач по физикепроисходят колебания (рис. 45). Сила Решение задач по физике является нормальной составляющей веса маятника Решение задач по физике а сила Решение задач по физике направлена параллельно наклонной плоскости вниз. Из рисунка видно, что 
Решение задач по физике

Решение задач по физике
Рис.45. 


где Решение задач по физике — масса маятника; Решение задач по физике - ускорение силы тяжести. 
Зная Решение задач по физике можно легко определить период колебания маятника 

Решение задач по физике

Задача № 77. На пружине подвешен грузик массой Решение задач по физике Период колебаний системы составляет Решение задач по физике Затем подвешивают еще один грузик, в результате чего период колебаний пружины возрастает до Решение задач по физике Определить, на сколько удлинилась пружина под действием перегрузки. 
Дано: 
Решение задач по физике

Решение:
Период колебаний пружины под действием грузика массой Решение задач по физике определяется формулой 
Решение задач по физике
Если к пружине подвесить еще один грузик Решение задач по физике то период колебаний станет равен 

Решение задач по физике
откуда 

Решение задач по физике
При небольших растяжениях пружины 
Решение задач по физике
где Решение задач по физике — действующая сила; Решение задач по физике — удлинение под действием этой силы. 
В нашем случае Решение задач по физике 
Тогда 

Решение задач по физике
Подставляя значение Решение задач по физике в уравнение Решение задач по физике найдем
Решение задач по физике
 

Задача № 78. За счет отклонения из положения равновесия ареометр в сосуде с водой совершает гармонические колебания с периодом Решение задач по физике Каков будет период колебаний ареометра в керосине? 
Дано: 
Решение задач по физике 

Решение:
Пусть отклонение ареометра из положения равновесия в воде Решение задач по физике (рис. 46). Тогда сила, стремящаяся вернуть его в положение равновесия, равна 
Решение задач по физике
где Решение задач по физике — плотность воды; Решение задач по физике — площадь поперечного сечения трубки ареометра. 
С другой стороны, 
Решение задач по физике
где Решение задач по физике — масса ареометра. 
Тогда 
Решение задач по физике
откуда 
Решение задач по физике

Решение задач по физике

Рис.46.


Аналогичным образом можем написать формулу для периода колебания ареометра в керосине 
Решение задач по физике
где Решение задач по физике — плотность керосина. 
Отношение периодов колебаний 
Решение задач по физике
откуда 
Решение задач по физике
Подставляя численные значения, получим 
Решение задач по физике

 

Колебания и волны

Задача № 7.1. На идеально гладкой поверхности лежит груз массой m, растянутый пружинами жесткостью k1 и k(рис. 61). Если груз вывести из положения равновесия (отклонить в сторону), он начнет колебаться с периодом Т. Изменится ли период колебаний, если те же пружины закрепить не в точках А1 и А2, а в точках В1 и В2? Считать, что при всех растяжениях колебания пружины подчиняются закону Гука.

Решение:

До тех пор пока справедлив закон Гука, смещение груза массы m на величину х от положения равновесия вызывает появление силы упругости

F = - (k1+k2) x,

не зависящей от того, как были растянуты пружины, когда груз массой m находился в положении равновесия. Зависимость силы, действующей на m, от отклонения от положения равновесия определяет период колебаний; эта зависимость не изменяется при изменении растяжения пружин, соответствующего равновесию, поэтому и период колебаний не будет зависеть от этого растяжения. Следовательно, перемещение точек закрепления пружин из А1 и А2 в В1 и В2 (рис. 61) приведет лишь к смещению средней точки (около которой происходят колебания), но не повлияет на период.

Задача № 7.2. На чашку, подвешенную на пружине жесткостью k, падает с высоты h тело массой m и остается на чашке, т. е. его удар о дно чашки можно считать абсолютно неупругим (рис. 62). Чашка с телом начинают колебаться. Определить амплитуду их колебаний. (Для упрощения сначала рассмотреть случай, когда массой М чашки можно пренебречь.)

Решение:

Когда тело массой m достигнет чашки, она будет обладать кинетической энергией

mv2/2 = mgh.                                  (1)

После удара тело и чашка будут обладать тем же импульсом, что и тело до удара, и (поскольку массой чашки мы пренебрегаем) той же скоростью и той же кинетической энергией, т. е. mgh. Чашка с телом после удара будет опускаться и растягивать пружину. Растяжение пружины будет происходить за счет начальной кинетической энергии и работы силы тяжести. Если отклонение чашки вниз считать положительным, то работа силы тяжести равна mgx, где х — отклонение чашки от начального положения. Поэтому наибольшее растяжение пружины х0 определяется из условия, что вся кинетическая энергия и работа силы тяжести пошли на упругую деформацию пружины, т. е.

Решение задач по физике Решение задач по физике                       (2)

откуда Решение задач по физике

Наибольшему отклонению вниз соответствует положительный корень этого уравнения (так как мы условились считать отклонение вниз положительным). Так как это наибольшее отклонение больше, чем mg/k (что соответствует положению равновесия чашки, когда в ней лежит тело), то, достигнув наинизшего положения, чашка начнет подниматься кверху, пройдет через начальное положение и поднимется вверх, сжав пружину. Когда она остановится в верхнем положении, опять потенциальная энергия сжатой пружины будет равна сумме начальной кинетической энергии и работы силы тяжести, т. е. наибольшее отклонение вверх будет определяться также уравнением (2), но этому отклонению будет соответствовать второй, отрицательный корень уравнения.

Итак, чашка будет совершать колебания между двумя крайними положениями:

Решение задач по физике

При этом положению равновесия чашки будет соответствовать смещение х0 = mg/k. Следовательно, наибольшие отклонения чашки в обе стороны от положения равновесия будут одинаковы и равны

Решение задач по физике                                                                        (3)

Это и есть амплитуда колебаний чашки. Если массой М чашки пренебречь нельзя, то скорость, с которой чашка начнет опускаться, не будет равна скорости, с которой тело достигает чашки и которая определяется уравнением (1). Для определения скорости V, с которой начнет опускаться чашка под давлением упавшего тела, надо применить закон сохранения импульса. В таком слyчae будем иметь:

mv = (М + m) V,          V = mv/(M + m).

Подставляя значение v из уравнения (1), получим

Решение задач по физике

Далее, необходимо иметь в виду, что движущейся массой теперь будет сумма масс чашки и тела. Кроме того, в начальный момент пружина растянута на длину а весом чашки, причем

ka=Mg.                                                         (4)

Поэтому закон сохранения энергии дает

Решение задач по физике

Подставив в это выражение значение а из (4) и перенеся все члены в левую часть, получим после преобразований

Решение задач по физике

Отсюда

Решение задач по физике

Новым положением равновесия будет

Решение задач по физике

Рассуждения, аналогичные предшествующему случаю, приведут к значению амплитуды Решение задач по физике                                                              (5)

Как и следовало ожидать, выражение (5) переходит в (3), если считать М = 0.

Задача № 7.3. Как будет меняться период колебания маятника, состоящего из сосуда, подвешенного на длинной нити, если сосуд наполнен водой, которая постепенно вытекает через отверстие в дне сосуда (рис. 63)?

Решение задач по физике     Решение задач по физике

Рис. 61                                                         Рис. 62                     Рис. 63

Решение:

По мере вытекания жидкости из сосуда сначала центр масс жидкости, а значит и центр масс маятника, будет опускаться и расстояние от центра масс до точки подвеса увеличиваться. Поэтому вначале пo мере вытекания воды период колебаний маятника будет постепенно расти.

Однако снижение центра масс сосуда с водой не происходит монотонно. Когда воды в сосуде останется мало, то при ее вытекании центр масс сосуда с водой может начать повышаться, а период колебаний маятника уменьшаться. Это можно усмотреть хотя бы из того, что, когда вся вода вытечет из сосуда, центр масс маятника будет выше, чем в том случае, когда уровень воды в сосуде лежит несколько ниже центра масс самого сосуда. Такого немонотонного хода изменения периода колебаний происходить не будет, если центр масс самого сосуда находится в дне сосуда.

Задача № 7.4. Два одинаковых маятника связаны невесомой пружиной (рис. 64). В одном случае оба маятника колеблются так, что они в каждый момент отклонены на одинаковый угол в одну сторону. В другом случае они колеблются так, что в каждый момент они отклонены на одинаковый угол в противоположные стороны. В каком случае период колебаний будет меньше?

Решение:

Во втором случае период колебаний будет меньше. В самом деле, в первом случае (рис. 222, а) каждый маятник колеблется только под действием горизонтальной составляющей силы тяжести, так как пружина не растягивается и не сжимается. Во втором случае (рис. 222, б) к составляющей силы тяжести прибавляется сила упругости пружины, направленная всегда к положению равновесия. Поэтому ускорение маятника в каждый момент во втором случае будет больше, чем в первом. Так как период колебаний тем меньше, чем больше ускорение, то во втором случае период колебаний будет меньше, чем в первом.

Решение задач по физике

Рис. 222

Задача № 7.5. Два одинаковых маятника, связанных невесомой пружиной (см. рис. 64), были отклонены в плоскости чертежа в одну и ту же сторону на один и тот же угол и вследствие этого  совершают колебания в плоскости чертежа. Увеличится или уменьшится период колебаний, если один из маятников удалить, а среднюю точку пружины закрепить неподвижно?

Решение:

Если сначала маятники были отклонены в одну и ту же сторону на один и тот же угол, то и колебания они будут совершать так, что их отклонение в каждый момент времени будет одно и то же (т. е. колебания будут происходить с одинаковыми фазами). Тогда пружина не будет растягиваться и, если она невесома, никакого влияния на период колебания маятников не окажет: маятники колеблются с периодом, свойственным каждому из маятников в отдельности. Средняя точка пружины в этом случае тоже совершает колебания.

Если же среднюю точку пружины закрепить неподвижно, то пружина начнет влиять на период колебания маятника, так как при колебаниях она будет растягиваться и сжиматься. Колебания маятника будут происходить не только под действием горизонтальной составляющей силы тяжести, но и под действием силы упругости пружины, которая всегда действует в направлении к положению равновесии И поэтому увеличивает ускорение маятника по сравнению с тем, которое он nмел бы под действием только силы тяжести. Это приводит к уменьшению периода колебаний.

Задача № 7.6. Подставка совершает в вертикальном направлении гармонические колебания (рис. 65), причем амплитуда этих колебаний А = 0,5 м. Каков должен быть наименьший период этих колебаний; чтобы лежащее на подставке тело не отделилось от нее?

Решение задач по физике                            Решение задач по физике

Рис. 64                                                           Рис. 65

Решение:

Тело, лежащее на подставке, не будет отделяться от нее в крайнем верхнем положении, если ускорение силы тяжести будет не меньше направленного вниз ускорения подставки при ее гармоническом колебании. Наибольшее ускорение вниз подставка будет иметь в верхнем положении (см. рис. 65). Если амплитуда колебаний есть А, а период Т, то амплитуда ускорения будет равна Решение задач по физике Мы получим наименьшее значение периода колебаний Т, приравняв амплитуду ускорения подставки ускорению свободного падения, т. е. Решение задач по физике отсюда наименьший период колебаний равен

Решение задач по физике

Задача № 7.7. Подставка совершает в горизонтальном направлении гармонические колебания с периодом Т = 5 с; Находящееся на подставке тело начинает по ней скользить, когда амплитуда колебаний достигает значения А = 0,6 м. Каков коэффициент трения k между телом и подставкой?

Решение:

Тело, находящееся на подставке, не будет скользить по ней до тех пор, пока сила трения, действующая со стороны подставки на тело, будет достаточно велика для того, чтобы сообщать телу такое же ускорение, какое имеет подставка, т. е. будет больше произведения массы этого тела на наибольшее значение ускорения подставки. Если колебания происходят с амплитудой А и периодом Т, то амплитуда ускорения будет равна Решение задач по физике. Следовательно, тело остается в покое, пока силы трения Решение задач по физике

С другой стороны, если mg — сила тяжести, k — коэффициент трения, то та же сила трения f = kmg. Поэтому тело начнет скользить, когда Решение задач по физике

Задача № 7.8. Как изменится период колебаний математического маятника, если его точку подвеса двигать: а) вертикально вверх с ускорением а; б) вертикально вниз с ускорением а < g и в) горизонтально с ускорением а?

Решение:

Чтобы оценить влияние ускорения точки подвеса на характер движения маятника, можно воспользоваться аналогией с поведением пружинных весов (динамометра). Как известно, при ускорении точки подвеса динамометра, равном а и направленном вверх, динамометр показывает силу F = m(g + а), где m — масса подвешенного груза; при ускорении, направленном вниз, F = m(g - а). Следовательно, ускорение точки подвеса эквивалентно появлению добавочной силы , направленной противоположно ускорению; таким образом, при ускорении вверх она направлена вниз и складывается с mg, а при ускорении вниз — вычитается из mg. Так же будет влиять ускорение точки подвеса и на колебания маятника (рис. 223).

Решение задач по физике

Рис. 223

а) Ускорение маятника в каждой точке будет больше, чем при неподвижной точке подвеса, и период его колебаний уменьшится.

б) Ускорение маятника в каждой точке будет меньше, чем при неподвижной точке подвеса, и период его колебаний увеличится.

в) Сила тяжести и добавочная силa, которой эквивалентно движение точки подвеса, направлены под прямым углом друг к другу, и их результирующая будет отклонена от вертикали в сторону, обратную ускорению а и образующую с вертикалью угол Решение задач по физике который определяется из соотношения Решение задач по физике (рис. 223, в). Поэтому, если маятник не колеблется, он будет висеть отклоненным на угол Решение задач по физике в сторону, противоположную ускорению а. Если вывести маятник из этого положения равновесия, то он будет колебаться около него с периодом, соответствующим ускорению Решение задач по физике т. е. большим g. Следовательно, период колебаний маятника уменьшится.

Задача № 7.9. Качающийся маятник прикреплен к массивной доске. В некоторый момент доска начинает свободно падать. Как при этом будет продолжать двигаться маятник относительно доски? Разобрать два случал. Доска начинает падать в момент, когда маятник находится: а) в одном из крайних положений, б) в каком-нибудь промежуточном положении. Силу трения не учитывать.

Решение:

Если доска с маятником начинает свободно падать, то сила тяжести, действующая на маятник, сообщает маятнику ускорение свободного падения и уже не является силой, возвращающей маятник к положению равновесия, как при неподвижной доске. Так как доска массивна, то движение маятника не влияет на движение доски и она имеет ускорение свободного падения. Поскольку других внешних сил нет, маятник будет сохранять относительно доски то движение, которым он обладает в момент начала падения. Следовательно: а) маятник остается неподвижным относительно доски в своем крайнем положении (так как его скорость в этом положении равна нулю), б) маятник продолжает вращаться вокруг точки подвеса равномерно со скоростью, которой он обладал в момент начала падения. К этому же результату можно прийти с помощью рассуждений, приведенных в решении задачи 7.8, полагая а= g.

Задача № 7.10. Определить период колебаний шарика, скользящего с высоты h вниз и вверх по двум наклонным плоскостям с углами, равными Решение задач по физике и Решение задач по физике (рис. 66). Силу трения и потери скорости при ударе не учитывать.

Решение задач по физике

Рис. 66

Решение:

 Если шарик начинает скользить с высоты h (см. рис. 66), то он приходит вниз со скоростью Решение задач по физике Дальнейшее движение его вверх по наклонной плоскости будет равнозамедленным со скоростью v = v0 – at, где а — ускорение, сообщаемое шарику силой тяжести.

Для движения шарика по правой из наклонных плоскостей ускорение Решение задач по физике поэтому его скоростьРешение задач по физике Очевидно, шарик будет двигаться вверх по наклонной плоскости до тех пор, пока его скорость v не станет равной нулю, т. е. в течение промежутка времени Решение задач по физике 

Столько же времени шарик будет двигаться вниз, поэтому полное время его движения по правой наклонной плоскости

Решение задач по физике

Аналогично, для левой наклонной плоскости имеем

Решение задач по физике

Полный период колебания шарика равен

Решение задач по физике

Подставляя значение Решение задач по физике получим Решение задач по физике

Задача № 7.11. Горизонтальный маятник, регистрирующий землетрясения, происходящие на большом расстоянии от места установки маятника, имеет ось качания, образующую небольшой угол Решение задач по физике с вертикалью. Схематически маятник можно представить в виде равностороннего треугольника, одна из сторон которого является осью, а в противоположной вершине укреплен груз массой m (рис. 67). Стороны треугольника, каждая длиной l, можно считать невесомыми. Найти период малых колебаний маятника. 

Решение:

 Разложим силу тяжести груза mg на две силы (рис. 224): F1 — по направлению, параллельному наклонной оси, и F2 — по направлению, к нему перпендикулярному. Последняя сила и будет действовать подобно силе тяжести в обычном маятнике, а сила F1, параллельная наклонной оси, никакого влияния на колебания маятника оказывать не будет. Длиной данного математического маятника служит высота h треугольника. В данном случае она равна Решение задач по физике Поэтому период малых колебаний маятника (с учетом, что ускорение равно Решение задач по физике) равен Решение задач по физике

Решение задач по физике

Рис. 224

Задача № 7.12. Имеется маятник, состоящий из металлического шарика, подвешенного на длинной шелковой нити. Как изменится период колебаний маятника, если шарик зарядить отрицательно, а другой, положительный, заряд поместить: а) внизу, на одной вертикали с точкой подвеса (рис. 68, а); б) в точке подвеса (рис. 68, б); в) сбоку, на одном уровне с шариком, так, что последний, качаясь, не может прикоснуться к этому заряду (рис. 68, в).

Решение задач по физике

Рис. 67                                                               Рис. 68

Решение:

а) Когда положительный заряд находится на одной вертикали с точкой подвеса, т. е. ниже шарика маятника, на  шарик, кроме силы тяжести, будет действовать сила притяжения к положительному заряду по закону Кулона. Эта сила кулоновского взаимодействия будет давать составляющую, направленную к положению равновесия, тем самым увеличивая возвращающую силу. Таким образом, колебания будут происходить так, как если бы ускорение свободного падения g увеличилось. Поэтому период колебаний маятника уменьшится.

б) Когда положительный заряд помещен в точке подвеса, то сила кулоновского взаимодействия противоположных по знаку зарядов будет всегда направлена вдоль нити, поэтому возвращающая сила не изменится и период колебаний шарика останется прежним.

в) Когда заряд находится сбоку, на одном уровне с наинизшим положением шарика, то сила взаимодействия между зарядами в течение одного полупериода (справа от положения равновесия) будет противоположна возвращающей силе и, следовательно, будет уменьшать ускорение маятника, а в течение другого полупериода (слева от положения равновесия) будет совпадать по направлению с возвращающей силой и, следовательно, будет увеличивать ускорение маятника. Но сила взаимодействия уменьшается с расстоянием и будет в среднем больше при отклонении шарика в сторону заряда (т. е. когда она уменьшает ускорение) и меньше при отклонении шарика в противоположную сторону (т. е. когда она увеличивает ускорение маятника). Поэтому, в конечном счете, заряд. помещенный сбоку, уменьшает ускорение шарика и, следовательно, увеличивает период колебаний маятника. Кроме того, смещается в сторону заряда и среднее положение,около которого будут происходить колебания.

Звук и звуковые волны

Задача № 7.13. Радиостанция, находящаяся в точке А, посылает сигнал проверки времени. Этот сигнал принимают в точках В и С два приемника (рис. 69). Слушатель, находящийся в точке В, принимает сигнал от своего приемника и через 1 с слышит звук того же сигнала, принятый в точке С приемником с мощным громкоговорителем. Чему равно расстояние между точками В и С?

Решение задач по физике

Рис. 69

Решение:

Так как радиосигналы распространяются со скоростью 300000 км/с, то радиоприемники принимают сигналы почти мгновенно и можно считать, что сигналы радиостанции А одновременно достигли приемников в точках В и С, хотя они и находятся вообще на разных расстояниях от станции А. Звуковой же сигнал из С в В распространяется со скоростью 330 м/с. Поэтому расстояние ВС определится как путь, пройденный звуком за 1 с, т. е. ВС = 330 · 1 = 330 м.

Задача № 7.14. Желая настроить струну в резонанс с камертоном, их заставили звучать одновременно. Появились биения. После того как к ножке камертона прикрепили грузик, частота биений уменьшилась. Что надо сделать со струной (подтянуть или ослабить), чтобы настроить ее в резонанс с камертоном?

Решение:

Частота биений равна разности частот колебаний струны и камертона. Прикрепление грузика к камертону уменьшает частоту его колебаний, а натяжение струны увеличивает частоту колебаний струны.

Поэтому если прикрепление грузика понижает частоту биений, то это значит, что частота колебаний камертона приблизилась к частоте: струны, которая колеблется с меньшей частотой, чем камертон. Чтобы настроить струну в резонанс с камертоном, надо ее подтянуть.

Задача № 7.15. Всегда ли справедливо выражение: «как аукнется, так и откликнется», т. е. всегда ли отраженный звук имеет ту же высоту тона, что и падающий?

Решение:

Высота тона (частота колебаний) отраженного звука не равна высоте тона падающего звука в случаях. хоrда источник звука или препятствие, от которого звук отражается, движутся (эффект Доплера).

Задача № 7.16. Зачем камертон (рис. 70) делается с двумя ножками? Годился ли бы камертон для своего обычного применения, если бы одну из ero ножек отпилили?

Решение задач по физике

Рис. 70

Решение:

У нормально работающего камертона ножки колеблются в противоположных фазах, т. е. движутся всегда в противоположные стороны (рис. 225). Поэтому центр масс камертона остается неподвижным и, следовательно, не требуется никаких внешних сил, чтобы такие колебания происходили. Камертон может совершать такие колебания, не будучи жестко закреплен.

Решение задач по физике

Рис. 225

Если одна из ножек камертона отрезана, а оставшаяся ножка совершает колебания такого же типа, как и прежде, то его центр масс уже не остается неподвижным. Следовательно, чтобы такие колебания происходили, должна действовать внешняя сила, т. е. камертон должен быть жестко закреплен (например, рукоятка зажата в тиски); со стороны закрепления в этом случае действует внешняя сила, которая приводит в движение центр масс. Если же рукоятку держать в руке, то закрепление не будет достаточно жестким, и колебания прежнего типа не смогут про исходить. Таким образом, наличие двух ножек у камертона делает ненужным жесткое закрепление камертона, т. е. позволяет пользоваться камертоном, держа его рукоятку в руке.

Задача № 7.17. Герой одного из рассказов О'Генри дал пинок поросенку с такой силой, что тот полетел, «опережая звук собственного визга». С какой силой F должен был ударить поросенка герой рассказа, чтобы описанный случай произошел в действительности? Массу поросенка примем равной m = 5 кг, а продолжительность удара t = 0,01 с.

Решение:

По второму закону Ньютона импульс силы, приложенной к какому-нибудь телу, равен изменению количества движения этого тела. Обозначив силу, действующую на поросенка, через F, а скорость поросенка через v, получим Ft = mv. Чтобы поросенок обгонял свой собственный визг, он должен двигаться со скоростью, большей скорости звука, т. е. v должно быть больше 330 м/с. Следовательно, Решение задач по физике

Задача № 7.18. Где человек должен слышать более громкий звук: в пучности или в узле смещений стоячей волны?

Решение:

Человек воспринимает звук, когда на барабанную перепонку уха действует переменное давление. В пучности смещений стоячей волны имеет максимальное значение амплитуда колебаний смещения, а амплитуда давлений практически равна нулю. В узле же смещений стоячей волны амплитуда смещения равна нулю, а давление колеблется с максимальной амплитудой. Поэтому в узле смещений стоячей волны человек должен слышать более громкий звук.

Задача № 7.19. Прибор Квинке позволяет убедиться в том, что усиление звука будет происходить тогда, когда разность хода двух волн одной и той же длины, приходящих в одно место, будет равна четному числу полуволн, а ослабление — когда разность хода равна нечетному числу полуволн. Прибор состоит из металлической трубки аb (рис. 71), которая разветвляется на две ветви с и с', сходящиеся снова в трубку de. Если к концу трубки а поднести звучащий камертон, то звуковая волна, разделяясь в на две части bcd и Ьс'd, вновь соединяется в трубке de. Если bcd - Ьс'd Решение задач по физике то в трубке d слышен звук, а если эта разность равна Решение задач по физике то звука в d не слышно. В последнем случае куда девается энергия звуковых колебаний, возбуждаемых камертоном?

Решение задач по физике

Рис. 71

Решение:

Встреча волн в d не мешает их дальнейшему распространению. Волны идут друг другу навстречу, и в трубах bcd и bс'd образуются стоячие волны, которые сходятся в b. Если звука в d не слышно, значит, здесь — пучность смещения и узел давления стоячей волны. В этом случае в b образуется узел смещения и пучность давления стоячей волны, и поэтому здесь звук будет отчетливо слышен, т. е. звуковая энергия вернулась к источнику. Как всегда, явление интерференции волн приводит к перераспределению энергии в пространстве.

Задача № 7.20. Почему звук в зале, заполненном публикой, звучит более глухо, чем в пустом?

Решение:

В пустом зале звук отражается от стен почти без поглощения, а одежда людей представляет материал, который поглощает звук.

Механика жидкостей и газов

Задача № 79. Деревянный цилиндр плавает на поверхности воды так, что он погружен в воду на Решение задач по физике Какая часть цилиндра будет погружена в воду, если поверх воды налить слой масла, полностью закрывающий цилиндр? Плотность масла Решение задач по физике 
Дано: 
Решение задач по физике

Решение:
Когда цилиндр погружается и в воду, и в масло, то на него будет действовать выталкивающая сила и со стороны воды, и со стороны масла. Поэтому выталкивающая сила (общая) должна 
уменьшиться па величину силы Решение задач по физике с которой вода  выталкивает масло, против первоначальной Решение задач по физике т. е.  выталкивающая сила 

Решение задач по физике
Так как какая-то часть цилиндра погружена в воду, то вода также стремится вытолкнуть цилиндр с силой, равной произведению погруженного в псе объема на удельный вес воды, т. е. Решение задач по физикеТак; как цилиндр находится внутри жидкости, то очевидно, что общая выталкивающая сила Решение задач по физике не превышает выталкивающей силы воды, т. е. 
Решение задач по физике
или 
Решение задач по физике
Заменив Решение задач по физике и решив уравнение относительно Решение задач по физике (объема той части цилиндра, которая погружена в воду), получим 
Решение задач по физике
Теперь, заменив Решение задач по физике будем иметь 
Решение задач по физике
После подстановки численных значений получим 
Решение задач по физике
Итак, цилиндр погружен в воду на Решение задач по физике

Задача № 80. Кусок пробки весит в воздухе Решение задач по физике кусок свинца— Решение задач по физике Если эти куски связать, подвесить к чашке весов и опустить в керосин, то показание весов будет Решение задач по физике Определить удельный вес пробки, учитывая, что удельный вес керосина равен Решение задач по физике а свинца — Решение задач по физике
Дано: 
Решение задач по физике

Решение:
Пусть Решение задач по физике — вес пробки, Решение задач по физике — ее удельный вес, Решение задач по физике — вес свинца, Решение задач по физике—его удельный вес. Тогда объем пробки Решение задач по физике объем свинца Решение задач по физике общий объем пробки и свинца 

Решение задач по физике
Согласно закону Архимеда, на погруженное в жидкость тело 
действует выталкивающая сила Решение задач по физике (в нашем случае Решение задач по физике) равная весу вытесненной жидкости в объеме погруженного тела 
Решение задач по физике
где Решение задач по физике—удельный вес керосина. 
Отсюда 
Решение задач по физике
Решая полученное уравнение относительно искомой величины Решение задач по физике получаем 
Решение задач по физике
подставив численные значения, найдем 
Решение задач по физике

Задача № 81. На поверхности воды плавает цилиндрический сосуд, площадь дна которого Решение задач по физике В сосуд налита вода до высоты Решение задач по физике Погружение сосуда при этом равно Решение задач по физике (рис. 47, а). Как изменятся Решение задач по физике и Решение задач по физике если внутрь сосуда поместить плавающее тело весом Решение задач по физикеимеющее форму куба? 

Решение:
Изменение веса плавающего в воде цилиндра по закону Архимеда должно быть равно изменению выталкивающей силы при большем погружении его в воду. Цилиндр опускается при  погружении в него тела, вес которого Решение задач по физике (рис. 47, б). Можно написать 
Решение задач по физике
или 

Решение задач по физике
откуда 
Решение задач по физике  
В свою очередь Решение задач по физикеРешение задач по физикеЭто справедливо, если не учитывать толщину стенок сосуда. 

Задача № 82. Баллон весом Решение задач по физике содержит Решение задач по физике водорода при Решение задач по физике К нему привязана гондола весом Решение задач по физике Какая сила действует на лины в момент старта? 
Дано: 
Решение задач по физике

Решение:
Определим сначала выталкивающую силу, действующую на баллон, Удельный вес воздуха и водорода при Решение задач по физике и давлении Решение задач по физике составляет: 
Решение задач по физике
Исходя из известных законов 
Решение задач по физике
получим 
Решение задач по физике
где Решение задач по физике — плотность водорода при Решение задач по физике и давлении Решение задач по физикеРешение задач по физикеАналогично для воздуха 
Решение задач по физике
После подстановки численных значений получим 
Решение задач по физике
Выталкивающая сила по закону Архимеда 
Решение задач по физике
Тогда ускорение системы в момент старта 

Решение задач по физике
где Решение задач по физике — масса баллона; Решение задач по физике — масса гондолы; Решение задач по физике
Натяжение лин Решение задач по физике На лины действует еще сила веса гондолы. Тогда полное натяжение лин составляет 
Решение задач по физике

Рабочие жидкости и их свойства

Задача № 83. Два тела плотностью Решение задач по физике в пустоте имеют один и тот же вес. Их подвешивают к концам рычага и помещают в жидкость плотностью Решение задач по физике Каково должно быть отношение плеч рычага, чтобы не нарушилось равновесие? 

Решение:
Обозначим через Решение задач по физике объемы обоих тел, через Решение задач по физике — их вес в пустоте и через Решение задач по физике — плечи рычага. В жидкости вес тел соответственно равен: 

Решение задач по физике
При равновесии 

Решение задач по физике
откуда 

Решение задач по физике
Но 

Решение задач по физике
поэтому 
Решение задач по физике

Задача № 84. Вывести формулу поправки, которую надо вносить при взвешивании в воздухе на рычажных весах тела плотностью Решение задач по физике с помощью гирь плотностью Решение задач по физике если плотность воздуха Решение задач по физике 

Решение:
Обозначим все величины, относящиеся к гирям, индексом 1. 
Тогда вес гирь в воздухе будет равен 
Решение задач по физике
где Решение задач по физике— вес гирь, отнесенный к вакууму; Решение задач по физике — потеря в весе в воздухе: 
Решение задач по физике
где Решение задач по физике — объем, занимаемый гирями. 
Вес тела в воздухе равен 
Решение задач по физике
где Решение задач по физике — объем, занимаемый взвешиваемым телом. 
При равновесии весов Решение задач по физике
Решение задач по физике
Приравнивая правые части для Решение задач по физике получаем 
Решение задач по физике

откуда

Решение задач по физике
Замечая, что обычно Решение задач по физикеи пользуясь формулами  приближенного умножения и деления, перепишем предыдущую формулу в виде 
Решение задач по физике

или

Решение задач по физике

Решение задач по физике

Рис.48.

При взвешивании тел с относительно большим весом приведенной выше поправкой можно пренебречь, так как выражение, стоящее в скобках, будет мало отличаться от единицы. 

Задача № 85. Цилиндр с внутренним диаметром Решение задач по физике и длиной Решение задач по физике заканчивающийся сверху трубкой с внутренним диаметром Решение задач по физике может свободно двигаться относительно поршня. Первоначально он находится в положении Решение задач по физике (рис. 48). Через трубку наливают Решение задач по физике воды. Определить, на какую высоту над поршнем поднимается верхняя стенка цилиндра, если вес его вместе с трубкой Решение задач по физике 
Дано: 
Решение задач по физике

Решение:
Объем воды в цилиндре составляет Решение задач по физике в трубке Решение задач по физикегде Решение задач по физике — высота столбика воды в трубке (рис. 48, б). Тогда объем воды в цилиндре и трубке будет равен 
Решение задач по физике
Давление воды на верхнюю стенку цилиндра 
Решение задач по физике
где Решение задач по физике — плотность воды. 
Тогда сила, действующая на верхнюю часть поверхности цилиндра, 
равна 

Решение задач по физике
Отсюда 
Решение задач по физике
Подставляя выражение для Решение задач по физике в ранее полученное уравнение, найдем 
Решение задач по физике
Откуда 

Решение задач по физике
Учитывая, что сила Решение задач по физике действующая на верхнюю часть поверхности цилиндра, равна его весу Решение задач по физике после подстановки численных значений получим 
Решение задач по физике

Задача № 86. Определить скорость воды в струе, вытекающей из наконечника, находящегося на высоте Решение задач по физике над полом, в момент падения ее на пол, если начальная скорость вытекания струи под углом Решение задач по физике горизонту Решение задач по физике
Дано: 
Решение задач по физике 

Решение задач по физике
Рис. 49 

Решение:
Скорость выделенной части струи при нахождении ее в точке Решение задач по физике (рис. 49) будет по величине равна начальной скорости Решение задач по физике Следовательно, кинетическая энергия ее при нахождении 
в этой точке будет 
Решение задач по физике
где Решение задач по физике — масса воды выделенной части струи. 
При дальнейшем движении до точки падения эта часть струи, находясь под действием только силы тяжести, опустилась на высоту Решение задач по физике. Работа силы тяжести зависит только от высоты Решение задач по физике падения струи. Поэтому работа силы тяжести будет равна 
Решение задач по физике 
При отсутствии сопротивления воздуха эта работа пошла на увеличение кинетической энергии движущейся части струи по параболе от точки А до точки падения. Поэтому в момент падения  части струи на пол ее кинетическая энергия будет 
Решение задач по физике
или
Решение задач по физике
где Решение задач по физике — скорость воды в струе в момент падения. 
Решая последнее уравнение относительно Решение задач по физике найдем 

Решение задач по физике
Это выражение показывает, что скорость тела, брошенного под углом к горизонту, зависит от начальной скорости тела и от высоты нахождения его в данный момент времени над местом бросания и что она совершенно не зависит от угла бросания. 
Следовательно, предыдущая формула справедлива и для тех случаев, когда брошенное тело находится в точках параболы,  расположенных не ниже, а выше горизонтали, проведенной из точки бросания. Для этих случаев Решение задач по физике необходимо считать величиной  отрицательной. 
Подставляя значения Решение задач по физике найдем, что искомая скорость равна 
Решение задач по физике

Задача № 87. Из вертикальной трубки высыпается песок, причем его струя сохраняет диаметр трубки (рис. 50). Скорость песка в момент высыпания из трубки Решение задач по физике а его средняя плотность Решение задач по физике Какова средняя плотность струи па расстоянии Решение задач по физике от отверстия трубки? 
Дано; 
Решение задач по физике 

Решение задач по физике

Рис.50.

Решение:
Подсчитаем количество песка, вытекающего за единицу времени из трубки. Обозначим через Решение задач по физике площадь сечения трубки. Количество песка, протекающего через любое сечение, должно быть одинаково, в противном случае ои будет накапливаться в каком-либо месте. 
Тогда 
Решение задач по физике 
откуда 

Решение задач по физике

Согласно законам свободного падения (см. задачу 86), 

Решение задач по физике
Тогда 
Решение задач по физике
Подставив численные значения, получим 
Решение задач по физике 

Вязкость жидкости

Задача № 88. Цилиндрический сосуд имеет в боковой стенке два отверстия, расположенные на расстоянии 25 см одно над другим. В сосуд налита вода до уровня на 25 см выше верхнего отверстия. Определить положение точки пересечения струй воды, вытекающих из 
отверстий (рис. 51). 
Дано: 
Решение задач по физике

Решение задач по физике

Рис.51.

Решение:
Скорость вытекания воды из отверстия Решение задач по физике 
Решение задач по физике 

Скорость вытекания воды из отверстия Решение задач по физике 
Решение задач по физике
Пересечение струи из отверстия Решение задач по физике со струей из отверстия Решение задач по физике произойдет на  
расстоянии 

Решение задач по физике
где Решение задач по физике 
Аналогично и для струи, вытекающей из отверстия Решение задач по физике 

Решение задач по физике
Приравнивая правые части двух последних равенств и учитывая 
два первых, получим 

Решение задач по физике
Решение указанного уравнения относительно Решение задач по физике дает 
Решение задач по физике
Тогда 
Решение задач по физике
Определим Решение задач по физике 
Решение задач по физике 

Задача № 89. К водопроводному крану с помощью резиновой трубки присоединена стеклянная трубка длиной Решение задач по физике с внутренним поперечным сечением Решение задач по физике Трубка изогнута снизу (рис 52, а). Определить, на какой угол отклонится трубка, если из -нее вытекает вода со 
скоростью Решение задач по физике а масса трубки Решение задач по физике
Дано: 
Решение задач по физике

Решение:
Вытекающая из отверстия струя жидкости действует на трубку с силой Решение задач по физике (рис. 52, б) и отклоняет ее на угол Решение задач по физике Вес трубки с находящейся в ней жидкостью разложится на силу Решение задач по физике стремящуюся вернуть трубку в положение равновесия, и силу Решение задач по физике ускоряющую движение жидкости. Равновесие наступит тогда, когда моменты сил Решение задач по физике относительно точки Решение задач по физике будут равны: 
Решение задач по физике
откуда 
Решение задач по физике 
Из Решение задач по физике вытекает, что сила Решение задач по физике Тогда 
Решение задач по физике
Если из трубки за время Решение задач по физикевытекает количество воды Решение задач по физике то, согласно второму закону 
Ньютона, Решение задач по физике 
Так как 
Решение задач по физике
(где Решение задач по физике — плотность воды), то 
Решение задач по физике
Откуда 
Решение задач по физике
Вес трубки с жидкостью равен 
Решение задач по физике
Подставляя значения Решение задач по физике в уравнение Решение задач по физике получим 
Решение задач по физике
После подстановки численных значений 
Решение задач по физике

Задача № 90. Под поверхностью реки, скорость течения которой Решение задач по физике установлен гидравлический таран (рис. 53). Принцип действия его следующий; если клапан Решение задач по физике открыт, то по трубе будет протекать вода со скоростью Решение задач по физике Если клапан Решение задач по физике закрыт (он закрывается автоматически при определенной скорости течения воды), то вода в трубе остановится и за счет не кинетической энергии будет выполнена работа по поднятию какого-то количества воды на высоту Решение задач по физике 
Определить, какое количество воды поднимается тараном за сутки на высоту Решение задач по физике если длина трубы тарана Решение задач по физике диаметр ее Решение задач по физике и каждый клапан открывается Решение задач по физике раз в минуту. 
Дано: 
Решение задач по физике 

Решение задач по физике

Рис.53.

Решение:
Процесс действия тарана при одном закрытии клапана Решение задач по физике быть описай уравнением 
может 

Решение задач по физике

где Решение задач по физике — масса движущейся в трубе воды; Решение задач по физике — скорость в трубе; Решение задач по физике — масса порции воды, поднимающейся при закрытии клапана; Решение задач по физике — высота подъема. 
Из вышеприведенного уравнения следует, что 

Решение задач по физике 
Масса воды, поднимаемой тараном за время Решение задач по физике будет равна 
Решение задач по физике
где Решение задач по физике — число закрытий клапана Решение задач по физике в секунду. Но 
Решение задач по физике 
где Решение задач по физике — плотность воды, 
Таким образом, 
Решение задач по физике

Задача № 91. Из бака, наполненного водой, через трубку (рис. 54) в течение Решение задач по физике вытекает Решение задач по физике воды. Определить величину и направление реакции вытекающей жидкости, если площадь сечения трубки Решение задач по физике 
Дано: 
Решение задач по физике

Решение задач по физике

Рис.54.

Решение:
Так как сечение изогнутой трубки везде одно и то же, то при течении жидкости по изогнутой части трубки количество движения жидкости, протекающей за единицу времени через сечение трубки, будет оставаться постоянным по величине, но будет изменяться по направлению. 
Определим это изменение количества движения. 
Пусть за единицу времени через сечение Решение задач по физике (рис. 54) 
втекает в изогнутую часть трубки масса жидкости 
Решение задач по физике
где Решение задач по физике — численное значение скорости течения воды по трубке;Решение задач по физике—плотность воды. 
Эта масса воды приносит с собой количество движения 

Решение задач по физике
где Решение задач по физике — вектор скорости воды в сечении Решение задач по физике 
Через сечение Решение задач по физике за то же время вытекает такая же масса жидкости. Эта масса уносит с собой количество движения 

Решение задач по физике
где Решение задач по физике — вектор скорости в сечении Решение задач по физике
Изменение же количества движения указанной массы воды заРешение задач по физике при протекании по изогнутой части трубки будет равно 
Решение задач по физике
Так как на величину Решение задач по физике количество движения воды изменяется за Решение задач по физике должно быть равно по величине и направлению равнодействующей всех сил, действующих на жидкость со стороны стенок трубки. 
Обозначая эту силу через Решение задач по физике, можем, следовательно, написать 

Решение задач по физике
Если стенки действуют на жидкость с силой Решение задач по физике то по третьему закону Ньютона и жидкость будет действовать на эту часть трубки с той же по величине и противоположной по направлению силой. 
Эта сила носит название силы реакции вытекающей жидкости. 
Сила реакции, согласно сказанному, будет равна 
Решение задач по физике
Для определения величины Решение задач по физике необходимо найти Решение задач по физике так как остальные величины даны в задаче. 
Сначала найдем величину Решение задач по физике Известны объем жидкости Решение задач по физике вытекающей из бака за Решение задач по физике и площадь сечения трубки Решение задач по физике Исходя из этих данных, найдем, что 
Решение задач по физике
Так как численные значения Решение задач по физике равны Решение задач по физике а по направлению они перпендикулярны друг другу, то 
Решение задач по физике
Подставляя вместо Решение задач по физике их значения в уравнение Решение задач по физикеРешение задач по физике получим 

Решение задач по физике
Решая задачу, найдем 

Решение задач по физике

Движение жидкостей и газов

Задача № 8.1. Как вы думаете, справедлив ли закон сообщающихся сосудов (однородная жидкость в сообщающихся сосудах имеет один и тот же уровень), если в одном из сосудов на поверхности жидкости находится некоторый поплавок (капиллярность не учитывать)?

Решение:

Так как поплавок находится на поверхности жидкости в равновесии, то вес его равен весу вытесняемой им жидкости. Поэтому, если бы заменили поплавок той жидкостью,  которой он плавает, то она заняла бы объем, равный объему погруженной части поплавка, и уровень жидкости не изменился бы. Следовательно, закон сообщающихся сосудов не нарушится, если на поверхности жидкости в одном из сосудов плавает поплавок.

Задача № 8.2. Достаточно длинная открытая снизу трубка с плотно пригнанным поршнем, который может все же двигаться по трубке без трения, находится под водой и удерживается с помощью веревки (рис. 72). Верхний конец трубки над поршнем пустой. Как зависит сила натяжения веревки от глубины погружения трубки в воду?

Решение задач по физике

Рис. 72

Решение:

На уровне основания цилиндрической трубки под водой давление всюду одинаково: и вне трубки, и внутри нее. Следовательно, веревку натягивают сила тяжести самой трубки, за вычетом выталкивающей силы (сила тяжести воды в объеме стенок трубки), и сила, с которой вода давит на закрытый конец трубки (сюда входит и сила атмосферного давления). Давление же столба воды пропорционально глубине погружения закрытого конца трубки. В самом деле, если мысленно закрыть открытый конец трубки, то ничего не изменится. Но тогда на веревку будут действовать следующие силы: 1) сила тяжести трубки вместе с тем, что находится в трубке, 2) сила давления столба воды сверху (плюс атмосферное давление), а также сила давления воды снизу, равная силе тяжести столба воды, высота которого равна глубине погружения открытого конца трубки под свободной поверхностью воды (плюс атмосферное давление). Но последняя сила уравновешена водой и поршнем, находящимся внутри трубки.

Задача № 8.3. Ведро, имеющее массу m, вместимость V, вытаскивают с водой из колодца. Плотность материала, из которого сделано ведро, равна Решение задач по физике, плотность воды Решение задач по физикев. Какую силу необходимо приложить для подъема этого ведра, пока оно находится под водой и когда его вытащили из воды? Сопротивление воды движению ведра не учитывать.

Решение:

Когда ведро находится под водой, то для его подъема необходимо приложить силу, равную силе тяжести ведра минус сила тяжести воды, вытесненной стенками ведра. Считая, что плотность воды равна Решение задач по физике  получим Решение задач по физике Когда же ведро вытащили из воды, необходимо приложить силу, равную силе тяжести ведра с водой, т. е. Решение задач по физике

Задача № 8.4. Для того чтобы поднять уровень жидкости в сосуде на высоту h с помощью насоса, надо совершить некоторую работу. Изменится ли необходимая для этой же цели работа, если на поверхности жидкости плавает какое-нибудь тело?

Решение:

Если на поверхности жидкости в сосуде плавает какое-нибудь тело, то, по закону Архимеда, его масса равна массе вытесненной жидкости. Значит, если тело заменить равной по массе жидкостью, то уровень жидкости не изменится, а потому не изменится и та работа, которую необходимо совершить, чтобы поднять уровень жидкости на высоту h.

Задача № 8.5. Изменится ли осадка парохода, перешедшего из северных вод в экваториальные, вследствие изменения ускорения свободного падения с широтой?

Решение:

Изменение ускорения свободного падения с широтой одинаково сказывается как на пароходе, так и на воде, в которой плавает пароход. Поэтому осадка парохода не изменится.

Задача № 8.6. Как зависит подъемная сила аэростата от температуры,  при которой производится его подъем?

Решение:

Подъемная сила аэростата равна разности силы тяжести воздуха и силы тяжести газа, заполняющего аэростат, в объеме аэростата. Иначе говоря, подъемная сила пропорциональна разности плотностей воздуха и газа, заполняющего аэростат. Так как плотность газа обратно пропорциональна температуре, то и разность плотностей обратно пропорциональна температуре, т. е. подъемная сила тем больше, чем ниже температура.

Задача № 8.7. Когда объясняют опыт со взвешиванием воздуха, иногда говорят, что сначала взвешивают колбу с воздухом, а затем, после откачки воздуха из колбы, взвешивают одну колбу. Разность показаний весов в первом и втором случаях составляет массу воздуха в объеме колбы. Правильно ли такое толкование опыта по взвешиванию воздуха?

Решение:

Такое толкование неправильно. Когда на чашку рычажных весов кладут колбу, в которой есть воздух, то находят массу колбы с воздухом, уменьшенную (по Закону Архимеда) на массу занимаемого колбой объема воздуха, т. е. определяют массу только самой колбы (без воздуха). Когда же на чашку весов кладут колбу, из которой воздух удален, то определяют массу колбы (без воздуха), уменьшенную на массу воздуха в объеме колбы. Разность этих двух взвешиваний дает массу воздуха в объеме колбы.

8.8. Можно ли измерить плотность воздуха, взвешивая мягкий воздухонепроницаемый мешок сначала пустой (сжатый), а потом наполненный воздухом? Объем мешка в наполненном состоянии известен.

Решение задач по физике

Рис. 73                                                                               Рис. 74

Решение:

При заполнении мешка воздухом при атмосферном давлении сила тяжести его не изменится. В самом деле, сила тяжести воздуха в мешке уравновешивается подъемной силой, так как взвешивание производится в воздухе, а не в вакууме (объемом материала мешка мы пренебрегаем). Если воздух в мешке сжат, т. е. плотность его больше, чем плотность окружающей атмосферы, то сила тяжести мешка увеличится. Однако в этом случае для определения плотности воздуха необходимо знать не только объем и силу тяжести мешка, но и давление воздуха внутри него. Зная, во сколько раз давление, а значит и плотность воздуха внутри мешка, больше, чем плотность воздуха в окружающей атмосфере, и учитывая подъемную силу, можно найти плотность воздуха как внутри мешка, так и вне его при атмосферном давлении.

Задача № 8.9. К крючку одной из чашек весов подвешена трубка ртутного барометра. Определить массу гири, лежащей на другой чашке, если весы находятся в равновесии (рис. 73).

Решение:

Масса гири на чашке весов при равновесии должна быть равна сумме масс стеклянной трубки и столба ртути внутри трубки над уровнем ртути в чашке. В самом деле, на верхний, закрытый, конец трубки действует атмосферное давление, т. е. давит весь столб воздуха, находящегося сверху, масса которого практически равна массе столба ртути в трубке. Толщину стенок трубки учитывать не нужно, так как на них снизу действует также атмосферное давление. Снизу же на трубку атмосферное давление не действует. (Атмосферное давление вызывает подъем ртути в трубке, но, поскольку ртуть не достигает верхнего конца трубки, на него это давление не передается.) В нашем рассмотрении мы пренебрегаем потерей в массе конца барометрической трубки, погруженного в ртуть в чашке барометра.

Задача № 8.10. Удастся ли опыт Торричелли, если барометрическую трубку со ртутью поставить открытым концом не в чашку со ртутью, а в чашку с водой (рис. 74)?

Решение:

Опыт Торричелли в таком случае не удастся. Хотя в вертикальном положении трубки мы будем иметь жидкости в положении равновесия, но это неустойчивое равновесие, так как их общий центр масс не занимает наинизшего положения. Поэтому ртуть вытечет из трубки, а вода ворвется в нее.

Задача № 8.11. Почему нельзя тушить горящий керосин, заливая его водой (плотность керосина меньше плотности воды)?

Решение:

Вода будет опускаться вниз и не закроет доступа воздуха (который поддерживает горение) к керосину.

Задача № 8.12. Во многих перегонных аппаратах труба, в которой конденсируется пар, окружена другой трубкой (рубашкой), по которой протекает холодная вода (рис. 75). При этом вода подается в рубашку снизу, а не сверху. Почему не наоборот?

Решение задач по физике

Рис. 75                                                          Рис. 76

Решение:

Вода подается в рубашку снизу, а не сверху, чтобы она заполнила всю рубашку. В противном случае часть рубашки оказалась бы заполненной воздухом (рис. 226).

Решение задач по физике

Рис. 226

Задача № 8.13. Было обнаружено, что в запаянной с обоих концов U-образной трубке уровни воды в обоих коленах находятся на одной высоте, когда трубка вертикальна, а также когда она наклонена в вертикальной плоскости (рис. 76). При каком условии это возможно?

Решение:

Уровни воды в обоих коленах не могли бы быть одинаковыми, если бы над поверхностью воды находился воздух, так как тогда давление воздуха, одинаковое при первом положении трубки, было бы неодинаково при втором, и вода не могла бы стоять на одном уровне. Следовательно, до запайки трубка была откачана.

Что касается давления водяных паров, то при наклоне трубки часть пара в одном из колен трубки конденсируется, а в другом соответственно часть воды испаряется и давление паров воды над уровнем жидкости в обоих коленах сохраняется одинаковым. Поэтому в обоих коленах уровни жидкости остаются на одной и той же высоте.

Задача № 8.14. В изогнутой трубке более короткое колено затянуто очень тонкой и мягкой непроницаемой пленкой. Трубка наполнена водородом и поставлена отверстием вниз (рис. 77). Какое положение займет пленка?

Решение задач по физике

Pиc. 77                                     Рис. 78

Решение:

Давление на пленку изнутри равно давлению воздуха у открытого конца трубки, уменьшенному на давление столба водорода с основанием, равным площади сечения трубки, и высотой h (рис. 227). Давление на пленку снаружи равно давлению воздуха у открытого конца, уменьшенному на давление столба воздуха с таким же основанием и высотой h. Так как воздух тяжелее водорода, то давление снаружи уменьшается на большую величину, чем внутри, т. е. наружное давление меньше внутреннего. Поэтому пленка выгнется наружу.

Решение задач по физике

Рис. 227

Задача № 8.15. В сосудах А и В находятся углекислый газ СО2 и водород Н(рис. 78). Манометры М1 и М2 показывают одинаковое давление. В каком направлении потечет газ, если открыть кран К. Что произойдет, если тот же опыт произвести, повернув сосуды манометрами вниз?

Решение:

Давление на кран К с каждой стороны не будет равно давлению, показываемому манометрами. К этому давлению следует прибавить гидростатическое давление столба газа Решение задач по физике Так как плотность СО2 больше плотности Н2 , то давление слева от крана К будет больше, чем справа, и после открытия крана часть углекислого газа перейдет в сосуд, занимаемый водородом.

Во втором опыте манометры будут находиться внизу. Давления у крана будут в этом случае на Решение задач по физике меньше, чем давления, показываемые манометрами. Если показания манометров одинаковы, то давление водорода на кран больше, чем давление углекислого газа, и при открывании крана часть водорода перейдет в сосуд с углекислым газом. 

Задача № 8.16. Почему не устраивают «газонапорные» башни по аналогии с водонапорными?

Решение:

 Роль водонапорной башни — создание гидростатического давления в водопроводе. Истечение жидкости из крана обусловлено гидростатическим давлением, равным разности между давлением столба воды с единичной площадью сечения и высотой h и давлением такого же столба воздуха (рис. 228).

Решение задач по физике

Рис. 228

Посмотрим, может ли «газонапорная» башня создать давление в газовой сети? Пусть в баке находится газ под давлением р, равным давлению окружающей атмосферы. Тогда внизу, у крана, давление будет равно Решение задач по физике rде Решение задач по физике — плотность газа, т. е. будет выше, чем в баке. Однако атмосферное давление внизу будет равно Решение задач по физике и так как плотность воздуха Решение задач по физике больше плотности светильного rаза (если оба находятся примерно под одинаковым внешним давлением), то атмосферное давление внизу будет больше, чем давление в газовой сети. Таким образом, «газонапорная» башня сама по себе не может создать повышенного давления в газовой сети.

Однако если в «газонапорной» башне газ находится под высоким давлением, так что плотность его больше плотность окружающего воздуха, то «газонапорная» башня создает некоторое избыточное давление. Очень большое давление в «газонапорной» башне создать практически трудно и опасно, а получающееся при этом избыточное давление было бы ничтожно мало по сравнению с тем давлением, которое дает водонапорная башня. Поэтому применение «газонапорных» башен практически нецелесообразно.

Течений вязких жидкостей

Задача № 8.17. В некоторых случаях в глубоких скважинах, пробуренных в земле, по трубе, опущенной в скважину, подается сжатый воздух. Будет ли избыточное давление (т. е. разность между давлением внутри трубы и снаружи) внизу трубы таким же, как наверху, или другим? Потерей давления в трубе вследствие течения по ней воздуха пренебречь.

Решение:

Избыточное давление внизу будет больше вследствие тоrо, что столб сжатого воздуха весит больше, чем такой же столб воздуха снаружи трубы. Эта разность весов добавляется к избыточному давлению, существующему вверху.

Задача № 8.18. В некоторых домах газ в подвале горит лучше, чем на верхних этажах. Почему?

Решение:

В покоящемся столбе газа избыточное давление газа над атмосферным в верхних этажах должно быть больше, чем внизу (см. задачу 8.16). Однако при движении по трубам вследствие действия сил сопротивления происходит падение давления. Поэтому давление газа уменьшается по мере удаления от места ввода газа. Это падение давления может оказаться настолько большим, что давление rаза в верхних этажах будет меньше, чем в подвале (так как газ вводится в дом по подземным трубам).

Задача № 8.19. Две одинаковые по массе оболочки шара, сделанные одна из тонкой резины, а другая из прорезиненной ткани, наполнены одинаковым количеством водорода и у земли занимают одинаковый объем. Который из шаров поднимается выше, если водород из них выходить не может?

Решение:

По мере подъема шаров давление и плотность окружающей атмосферы уменьшаются. Следовательно, если объем шара остается постоянным, то подъемная сила, равная силе тяжести воздуха в объеме, занимаемом шаром, уменьшается с высотой. Так как прорезиненная ткань почти не растягивается, то объем шара из такой ткани будет оставаться почти постоянным, а подъемная сила будет падать с высотой. Когда подъемная сила упадет до величины, равной силе тяжести оболочки и содержащегося в ней водорода, дальнейший подъем шара прекратится — он достигнет "потолка". Наоборот, тонкая резина легко растягивается, поэтому по мере уменьшения внешнего давления с высотой шар, сделанный из тонкой резины, будет раздуваться. Благодаря увеличению объема шара подъемная сила не будет заметно уменьшаться с высотой (несмотря на уменьшение плотности воздуха), и шар будет продолжать подниматься. Таким образом, шар из тонкой резины поднимется гораздо выше, чем шар из прорезиненной ткани. Такие шары-пилоты и шары-зонды из тонкой резины, применяемые для метеорологических наблюдений, поднимаются на очень большую высоту: до 30-40 км.

Задача № 8.20. Пузырьковый уровень представляет собой слегка изогнутую трубку, заполненную жидкостью так, что остается очень маленькое пространство — пузырек, в котором находятся пары жидкости. Когда пузырек больше: летом или зимой?

Решение:

Пузырек больше зимой, так как температурный коэффициент расширения жидкости больше, чем стекла. Объем жидкости при охлаждении уменьшается больше, чем объем трубки, и пространство, занятое парами, увеличивается. Летом же, наоборот, жидкость вследствие нагревания расширяется, и так как пары легко сжимаются под давлением, то пространство, · занятое парами, уменьшается.

Это легко можно наблюдать на опыте. Возьмите колбу, наполните ее водой и доведите до кипения. Часть воды при этом уйдет из колбы. Слегка охладите колбу, чтобы можно было закрыть ее пробкой и чтобы вода заполняла колбу целиком. При дальнейшем охлаждении колбы, положенной горизонтально, над водой появится пузырек. Если продолжать охлаждать колбу, размер пузырька увеличивается.

Задача № 8.21. В барометре, изображенном на рис. 79, отверстие А закрыто пробкой, и барометр наполнен ртутью. Что произойдет, если вынуть пробку из отверстия А

Решение задач по физике

Рис. 79                                 Рис. 80                                                     Рис. 81

Решение:

Внутреннее давление на пробку, равное давлению столба ртути над уровнем отверстия, меньше атмосферного давления, уравновешивающего весь столб ртути. Следовательно, ecли открыть пробку, то в трубку будет входить воздух и ртуть будет опускаться до тех пор, пока ее уровень в трубке не сравняется с уровнем ртути в чашке.

Задача № 8.22. В боковой стенке сосуда Мариотта (рис. 80) находятся три небольших отверстия 1, 2 и 3, закрытые пробками. Через пробку в горле сосуда проходит открытая с обоих концов трубка Т. Уровень воды в сосуде Мариотта находится выше отверстия 1. Уровень воды в трубке Т находится у самого основания трубки, расположенного на высоте отверстия 2. Что произойдет, если открыть одно из отверстий: 1, 2 или 3?

Решение:

Очевидно, что на уровне нижнего конца трубки Т, проходящей через горло сосуда, а следовательно на уровне отверстия 2, давление жидкости равно атмосферному. Поэтому, когда открывают отверстие 1, давление снаружи больше, чем изнутри, и воздух будет входить пузырьками в сосуд, вода в сосуде будет опускаться И входить в трубку Т. Когда уровень воды в этой трубке достигнет высоты отверстия 1, воздух перестанет входить в сосуд. Если вместо отверстия 1 открыть отверстие 2, то, из-за равенства давления снаружи и изнутри, будет наблюдаться равновесие, т. е. воздух не будет входить в сосуд, и вода не будет из сосуда выливаться. Если открыть отверстие 3, то вода будет выливаться из сосуда, а воздух — входить в сосуд через трубку Т. При этом давление у нижнего конца этой трубки, независимо от изменения уровня воды в сосуде, будет оставаться равным атмосферному и вода будет выливаться с постоянной скоростью до тех пор, пока уровень ее в сосу де не опустится до нижнего конца трубки Т. После этого вода будет продолжать выливаться, но скорость истечения будет убывать.

Задача № 8.23. Для измерения объема порошка, не реагирующего с воздухом и не поглощающего воздух, пользуются прибором, который носит название волюмометра. Этот прибор изображен на рис. 81. Чашка А с помощью шлифа соединяется с изогнутой стеклянной трубкой, снабженной краном К и оканчивающейся сосудом В, переходящим в прямую трубку. Последняя с помощью длинной резиновой трубки соединена с другой прямой стеклянной трубкой, которую можно перемещать вдоль шкалы вертикально. В верхней и нижней частях на сосуде В нанесены черточки, между которыми его вместимость точно измерена и равна V. Обе прямые трубки и резиновая содержат ртуть, образуя манометр.

Измерение объема поpoшкa производится так. По манометру отмечается атмосферное давление Н. При открытом кране К уровень ртути доводится до верхней черточки пузыря В, и кран закрывается. Затем прямую трубку манометра опускают вдоль шкалы вниз до тех пор, пока уровень ртути в пузыре не дойдет до нижней черточки, и отмечают теперь разность h1 уровней ртути в прямых трубках. После- этого прибор возвращают в начальное положение и кран К открывают.

Снимем чашку А и насыпем в нее порошок, объем которого желательно определить. Снова пришлифуем чашку и при одинаковых уровнях ртути у верхней черточки пузыря закроем кран К. Опустим опять прямую трубку манометра так, чтобы уровень ртути в пузыре В достиг нижней черточки, и измерим h2 — новую разность уровней в прямых трубках. Как, зная V, Н, h1 и h2, найти объем порошка?

Решение:

 Изменения давления в коленах манометра связаны с изменением объема воздуха внутри прибора законом Бойля - Мариотта. Итак, мы имеем:

1) Кран К открыт. Порошка в чашке А нет. Уровень ртути доведен до верхней черточки сосуда В. Воздух в чашке А занимает некоторый объем v. Манометр показывает атмосферное давление Н.

2) Кран К закрыт. Порошка нет. Уровень ртути доведен до нижней черточки сосуда В. Воздух занимает объем v + V. Давление равно Н – h1.

3) Кран К снова открыли, насыпали в чашку А порошок и при положении уровня ртути у верхней черточки сосуда, т. е. при атмосферном давлении Н, закрыли кран К. Очевидно, теперь воздух в чашке и соединительной трубке занимает объем v2, причем v – v2 = v1 есть объем порошка.

4) При закрытом кране довели уровень ртути до нижней черточки, в результате чего объем воздуха стал равным v2 + V, а давление изменилось на h2. По закону Бойля- Мариотта из 1) и 2) имеем

Решение задач по физике

Из 3) и 4) получаем

Решение задач по физике

Отсюда объем порошка равен

Решение задач по физике

Задача № 8.24. Масляный насос, приводимый в действие электродвигателем, работает с постоянным числом оборотов n0 в единицу времени. За каждый оборот он откачивает из сосуда определенный постоянный малый объем Решение задач по физике = coпst. Обозначим р0 первоначальное давление в откачиваемом сосуде, рt —давление через время t после начала действия насоса. Доказать, что Решение задач по физике растет линейно.

Решение:

Поскольку температура остается все время постоянной, изменения объема и давления можно считать подчиняющимися закону Бойля - Мариотта. После первого оборота объем газа становится равным Решение задач по физике(так как насос откачал Решение задач по физике, а оставшийся газ расширился до объема V, а давление — р1. По закону Бойля - Мариотта

Решение задач по физике

При втором обороте насос начинает качать от объема V (объем Решение задач по физике выброшен из сосуда) и от давления р1 ; после второго оборота

Решение задач по физике

и после n оборотов Решение задач по физике

Так как n0 — число оборотов в единицу времени, то

Решение задач по физике

отсюда

Решение задач по физике

или Решение задач по физике

Так как число оборотов в единицу времени n0 и выражение Решение задач по физике постоянны, можно записать

Решение задач по физике

Очевидно, чем больше константа С, тем больше скорость откачки. Зависимость между Решение задач по физике  и временем можно изобразить графически (рис. 229, а). Получается, что логарифм степени разрежения растет линейно со временем, а значит, растет и степень разрежения, т. е. каждый насос может дать любое разрежение. На самом деле для каждого насоса есть некоторое минимальное давление, ниже которого он не может продолжать откачку.

Решение задач по физике

Рис. 229

По мере приближения к этому предельному давлению скорость откачки замедляется, так что график зависимости Решение задач по физике от времени на самом деле выглядит, как на рис. 229, 6.

Задача № 8.25. На рис. 82 изображены графики зависимости логарифма относительного изменения давления насоса от времени (см. предыдущую задачу) для насосов 1 и 2. Который из этих насосов качает быстрее и который создает лучший вакуум?

Решение задач по физике

Рис. 82

Решение:

Первый насос качает быстрее (насыщение достигается за меньшее время t), но создает меньший вакуум, чем второй (предельное значение Решение задач по физике меньше).

Задача № 8.26. Пустой ящик размерами 1 х 1 х 3 м, открытый снизу, погружен в вертикальном положении в воду так, что крышка ящика находится на глубине 18,6 м. Найти выталкивающую силу F, действующую на ящик.

Решение:

При погружении ящика в воду воздух в нем будет сжиматься и вода будет входить в ящик. Объем воздуха в ящике может быть определен из закона Бойля-Мариотта:

Решение задач по физике                                                                              (1)

где соответственно для двух положений ящика (непогруженного и погруженного) h0 и h1 — высоты крышки ящика над уровнем воды в нем, р0 и р1 — давления воздуха в ящике, S — площадь основания. Давление в данном случае удобнее всего выразить в метрах водяного столба. Подставляя h0 = 3 м, р0 = 0,76 · 13,6 м вод. ст.,

р1 = р0 + 18,6 + (3 – h1) = 10,3 + 18,6 + (3 - h1) м вод. ст., получаем из (1) квадратное уравнение для h1:

Решение задач по физике

решая его, находим Решение задач по физике м.

Таким образом, новый (сжатый) объем воздуха в ящике равен приблизительно 1 м3 . Пренебрегая объемом, который занимают стенки ящика, получим, что выталкивающая сила, действующая на ящик, равна силе тяжести воды в объеме 1 м3 , т. е. Решение задач по физике 10 кН.

Задача № 8.27. Какие силы совершают работу по подъему аэростата?

Решение:

На аэростат действуют силы давления воздуха снизу и сверху, но первая больше второй, так как внизу, у основания аэростата, воздух сильнее сжат, чем вверху, у вершины аэростата. Разность этих двух сил направлена вверх, и она совершает работу по подъему аэростата. Значит, аэростат поднимается за счет упругой энергии сжатого воздуха атмосферы.

Задача № 8.28. При определении плотности твердых тел с помощью гидростатических весов сначала взвешивают тело в воздухе (на обычных весах), а потом погрузив его в воду. Как нужно изменить этот метод, если плотность тела меньше плотности воды?

Решение:

Проще всего прикрепить к телу груз с плотностью, большей плотности воды, что заставит тело погрузиться в воду. Взвешивание испытуемого тела с грузом надо произвести сначала в воздухе, а затем в воде. Предварительно нужно определить плотность груза, что можно сделать обычным способом. Зная силу тяжести груза и его плотность, можно найти плотность испытуемого тела с помощью следующего расчета.

Решение задач по физике

Рис. 230

Пусть сила тяжести испытуемого тела F1, сила тяжести груза F2, его плотность равна р2. Сила тяжести испытуемого тела и груза в воде равна F'. Разность между силой тяжести тела и груза в воздухе и воде позволяет определить объем тела с грузом:

Решение задач по физике

где Решение задач по физике — плотность воды. Так как объем V равен сумме объемов испытуемого тела V1 и груза V2 , а

Решение задач по физике

то Решение задач по физике

и искомая плотность тела равна

Решение задач по физике

На одной из физических олимпиад МГУ, где была предложена эта задача, несколько школьников дали другое решение, тоже возможное, хотя и труднее осуществимое. Схема этого решения приведена на рис. 230. В дно сосуда вделан крючок с блоком, и тело с помощью веревки, перекинутой через блок, прикреплено к чашке весов. Тогда на другую чашку весов приходится для равновесия положить гири, сила тяжести которых равна выталкивающей силе минус сила тяжести тела.

Задача № 8.29. В боковой стенке цилиндрического ведра сделано два отверстия, расположенные симметрично с противоположных сторон. Отверстия закрыты пробками, и ведро наполнено водой. Если вынуть пробки, то из отверстий вода будет вытекать струей. Что произойдет, если ведро будет свободно падать?

Решение:

При свободном падении ведра верхние слои воды перестают давить на нижние, а потому давления на боковую стенку не будет и истечение воды прекратится.

Задача № 8.30. Опрокинутая пробирка укреплена неподвижно над сосудом с водой (рис. 83). Как изменится в ней уровень воды, если вся система начнет свободно падать?

Решение задач по физике

Рис. 83

Решение:

Первоначальная разность уровней воды в пробирке и в сосуде h определяется условием

Решение задач по физике

где р0 — внешнее давление, р — давление воздуха внутри пробирки, Решение задач по физике — плотность воды. Свободно падающее тело ведет себя так, как будто оно не обладает весом. Поэтому при свободном падении столб воды не будет оказывать гидростатического давления, следовательно, уровень воды в пробирке будет опускаться до тех пор, пока не установится равенство р = р.

Задача № 8.31. Подводная лодка, опустившись на глинистое или песчаное дно, иногда не может подняться. Как объясняется это явление «присасывания» подводной лодки?

Решение:

На погруженное в воду тело действуют силы давления окружающей воды. Эти силы действуют как на верхнюю, так и на нижнюю части тела. Но так как давление с глубиной растет, то силы, действующие на нижнюю часть тела и направленные вверх, больше сил, действующих на верхнюю его часть и направленных вниз. Разность этих двух сил и обусловливает выталкивающую силу. Когда подводная лодка плотно прижата к мягкому грунту так, что между ней и грунтом нет воды, давление воды на нижнюю часть лодки отсутствует, т. е. отсутствует сила, направленная вверх. Сила же давления воды на верхнюю часть лодки направлена вниз и вместе с силой тяжести лодки прижимает ее к грунту.

Задача № 8.32. Одно колено открытого U-образного ртутного манометра присоединено к колбе с водой, из которой выкачан воздух (рис. 84). Что будет показыnать манометр, если колбу погрузить в сосуд с кипящей водой? Будут ли зависеть показания манометра от высоты над уровнем моря?

Решение задач по физике

Рис. 84

Решение:

Над водой в колбе будет находиться насыщенный водяной пар при такой температуре, при какой кипит вода при данном атмосферном давлении. Условием кипения является равенство давления насыщенного пара и внешнего давления. Поэтому давление на конце манометра, присоединенном к колбе, всегда будет равно внешнему давлению, а следовательно, манометр всегда будет показывать нуль, независимо от высоты над уровнем моря.

Задача № 8.33. Всасывающим насосом можно поднять воду при 0°С на высоту h = 10 м. На какую (большую или меньшую) высоту можно поднять горячую воду при 90°С?

Решение:

Всасывающий насос поднимает воду за счет разности между внешним атмосферным давлением р0 = 1 атм и остаточным давлением в насосе р1 (рис. 231). Высота h, на которую поднимается вода, определяется из условия

Решение задач по физике 

где Рв- плотность воды. Отсюда находим

Решение задач по физике

Решение задач по физике

Рис. 231

В случае горячей воды (+ 90°С) остаточное давление Pt в васосе увеличится за счет упругости паров воды при + 90 ос {приблизительно равной 530 мм рт. ст.). ПоэтомуРешение задач по физике Решение задач по физике Следовательно, горячая вода будет поднята на значительно меньшую высоту. Правда, плотность горячей воды несколько меньше плотности холодной, но это изменение гораздо меньше, чем изменение упругости паров, поэтому на характер ответа это обстоятельство не влияет.

Задача № 8.34. В стакане, наполненном до краев водой, плавает кусок льда (рис. 85). Перельется ли вода через край, когда лед растает? Что произойдет, если в стакане находится не вода, а жидкость более плотная или менее плотная?

Решение:

По закону Архимеда масса плавающего льда равна массе вытесненной им воды. Поэтому объем воды, образовавшийся при таянии льда, будет в точности равен объему вытесненной им воды, и уровень воды в стакане не изменится.

Если в стакане находится жидкость более плотная, чем вода, то объем воды, образовавшейся nосле таяния льда, будет больше, чем объем жидкости, вытесненной льдом, и вода перельется через край. Наоборот, в случае менее плотной жидкости, после того как лед растает, уровень понизится.

Задача № 8.35. В сосуде с водой плавает кусок льда, внутри которого находится кусок свинца (рис. 86). Изменится ли уровень воды в сосуде, когда лед растает? Что будет, если внутри льда находится не свинец, а пузырьки воздуха?

Решение задач по физике

Рис. 85                                                       Рис. 86

Решение:

Так как кусок льда со свинцом имеет массу, большую, чем кусок чистого льда того же объема, то он глубже погружен в воду, чем чистый кусок льда, и вытесняет больший объем воды, чем тот, который займет вода, образовавшаяся при таянии льда (см. задачу 8.34). Поэтому, когда лед растает, уровень воды понизится (кусок свинца при этом упадет на дно, но его объем остается прежним, и он непосредственно уровня воды не изменяется.

При наличии пузырьков воздуха лед имеет массу, меньшую, чем сплошной кусок льда того же объема, и, следовательно, погружен на меньшую глубину, чем сплошной кусок льда того же объема. Однако поскольку массой воздуха можно пренебречь (по сравнению с массой льда), то кусок льда по-прежнему вытесняет воду, масса которой равна массе льда, и когда лед растает, уровень жидкости не изменится (когда лед растает, пузырьки подымутся кверху и уйдут из воды). Поэтому случай с пузырьками воздуха не является обратным случаю льда с куском свинца.

Задача № 8.36. На одной чашке весов находится сосуд с водой, а на другой чашке — штатив, на перекладине которого подвешено на невесомой нити тело (рис. 87). Пока тело не погружено в воду, весы находятся в равновесии. Затем нить удлиняют так, что тело целиком погружается в воду. При этом равновесие нарушается. Что нужно сделать, чтобы восстановить равновесие?

Решение задач по физике

Рис. 87                                                                              Рис. 88.

Решение:

По закону Архимеда на тело, погруженное в жидкость, действует со стороны жидкости выталкивающая сила, равная силе тяжести жидкости в объеме, занимаемом погруженным телом. Эта сила уменьшает натяжение нити, на которой висит тело. Поэтому на правую чашку весов действуют следующие силы: сила тяжести штатива и сила тяжести тела, уменьшении на силу тяжести вытесненной им воды. По третьему закону Ньютона тело, погруженное в жидкость, будет давить на жидкость с силой, равной подъемной силе, и это действие через жидкость в сосуде будет передаваться на левую чашку весов. Указанная добавочная сила Возникает потому, что уровень жидкости при погружении тела поднимается и, следовательно. давление на дно сосуда повышается. Итак, на левую чашку весов будут действовать следующие силы: сила тяжести стакана с водой и сила тяжести объема воды, равного объему погруженного тела. Так как масса штатива с телом тaкая же, как и стакана с водой, то для восстановления равновесия нужно на правую чашку весов положить груз массой, равной удвоенной массе тоrо объема воды, который занимает погруженное тело.

Задача № 8.37. Сосуд с водой уравновешен на весах. Изменится ли равновесие, если опустить палец в воду, не касаясь при этом дна сосуда (рис. 88)?

Решение:

На погруженный в воду палец будет действовать выталкивающая сила со стороны воды. Сила, равная и противодействующая ей, па третьему закону Ньютона будет действовать на дно сосуда (см. задачу 8.36). Равновесие нарушится и чашка весов, па которой стоит сосуд, опустится.

Задача № 8.38. На весах уравновешено тело, погруженное в жидкость (рис. 89). Изменится ли показание весов при нагревании жидкости вместе с погруженным в нее телом?

Решение:

При температуре 0°С сила, действующая на тело, погруженное в жидкость, равна g (m - pV); где m — масса тела, Решение задач по физике — масса жидкости в объеме, который занимает погруженное тело (V — объем тела, Решение задач по физике — плотность жидкости). С нагреванием объем тела увеличивается, плотность жидкости р уменьшается. Масса жидкости в объеме погруженного тела при температуре t равна

Решение задач по физике

Так как температурный коэффициент объемного расширения твердых тел Решение задач по физике обычно меньше температурного коэффициента объемного расширения жидкостей Решение задач по физике то

Решение задач по физике

Поэтому в большинстве случаев выталкивающая сила, действующая со стороны жидкости на тело, уменьшается с повышением температуры,. т. е. весы (рис. 89) отклонятся влево.

Задача № 8.39. Один из многочисленных ошибочных проектов вечных двигателей заключался в следующем: в вырез стенки бака с жидкостью вставлен вал, ось которого лежит в плоскости стенки (рис. 90); вал закрывает собой весь вырез, так что жидкость не выливается. Вал может вращаться на своей оси. На половину его, погруженную в жидкость, действует выталкивающая сила (по закону Архимеда), которая, казалось бы, должна вызвать вращение вала по часовой стрелке. Это вращение, если бы оно возникло, должно было бы продолжаться вечно. В чем здесь ошибка?

Решение задач по физике                  Решение задач по физике

Рис. 89                                                                          Рис. 90

Решение:

Вращения вала не возникает, так как вода давит на боковую поверхность вала во всех точках перпендикулярно к поверхности. т. е. по направлению радиуса. Поскольку эти силы проходят через ось вала, они не могут вызвать его вращения. Все эти силы дают равнодействующую, направленную наружу, под некоторым углом вверх, но проходящую через ось вала и поэтому стремящуюся вытолкнуть вал из бака, а не поворачивать его вокруг оси.

Задача № 8.40. В сосуд с водой погружается открытый стакан: один раз дном вверх, а другой раз дном вниз (рис. 91) на одну и ту же глубину. В каком из этих случаев работа, которую нужно совершить, чтобы погрузить стакан, будет больше? (Вода из сосуда не выливается, и в стакан, погруженный дном вниз, не вливается.)

Решение задач по физике                      Решение задач по физике

Рис. 91                                                                      Рис. 92

При погружении стакана в воду сила, которую надо приложить к стакану для его погружения, будет увеличиваться по мере погружения стакана в воду, так как будет возрастать выталкивающая сила. Однако эта сила в рассматриваемых двух случаях не одинакова: в первом случае воздух внутри стакана сжимается и вода частично входит в стакан. Таким образом, при одинаковом погружении стаканов в воду в первом случае вытесненный объем меньше, значит, и выталкивающая сила меньше, чем во втором случае, а потому меньше и равная ей сила, которую надо приложить, чтобы погрузить стакан, а следовательно, и работа этой силы при одной и той же глубине погружения меньше.

Задача № 8.41. Почему труба сливной раковины присоединяется к вертикальной канализационной трубе посредством сифонной трубки (рис. 92), а не идет прямо?

Решение:

Внутри U-образной части сифонной трубки задерживается некоторое количество воды. Эта вода образует пробку, изолирующую жилое помещение от газов из канализационных труб.

Задача № 8.42. Будет ли работать гидравлический пресс, если его цилиндр заполнять не жидкостью, а газом?

Решение:

Пресс может работать, так как принцип, на котором основано его действие, справедлив как для жидкостей, так и для газов (в газе, как и в жидкости. давление передается по всем направлениям), но к.п.д. пресса будет крайне мал, так как подавляющая часть затрачиваемой работы будет идти на сжатие газа вследствие того, что сжимаемость газа велика — гораздо больше, чем сжимаемость жидкости.

Задача № 8.43. Когда взрывается паровой котел, в котором давление пара составляет 10-15 атм, происходят большие разрушения; когда же взрывается гидравлический пресс, в котором давление гораздо выше, то взрыв значительных разрушений не причиняет. Почему?

Решение:

Разрушения, причиняемые при взрыве, определяются той работой, которую может совершить пар или жидкость, расширяясь от начального объема до того объема, который они должны занимать при атмосферном давлении. Работа же эта зависит как от давления, так и от изменений объема. Пар, равно как и газ, обладает большой сжимаемостью, жидкости же, наоборот, чрезвычайно слабо сжимаемы; поэтому сжатый пар даже при сравнительно небольшом